LSAT 2016

PART I All About the LSAT

CHAPTER 2 LSAT Diagnostic Test

This diagnostic test is designed to closely resemble the real LSAT in types and numbers of questions, in time limits, and in degree of difficulty. Use it to assess your test readiness as you begin your LSAT preparation.

For best results, try to simulate exam conditions as you take this test. Complete the test in a single sitting and follow directions and time limits. At the end of the test, you will find an Answer Key as well as answers and explanations for every question. Score yourself using the Answer Key, then read the explanations, paying particular attention to those for questions that you missed or that were difficult for you to answer. Don’t skip the explanations for questions you answered correctly; you may learn simpler or easier ways to reach the right answer.

Your score on this test will indicate your current level of readiness to take the LSAT. Use it as a starting point to help you plan your preparation program. Which types of questions were easiest for you? Which ones were the most difficult? Were there any that you found confusing or that you could not answer at all? Instructional material on each LSAT question type is presented in the following chapters. As you read through the chapters, focus on the question types that you found most difficult. Pay careful attention to the examples and solution strategies presented. Then sharpen your problem-solving skills by tackling the practice tests at the end of this book. Additional LSAT practice is available on the companion website at MHPracticePlus.com.

You can also take the diagnostic test on your tablet or smartphone as well as your laptop or home computer. See page 7A of the Welcome insert for more information.

Answer Sheet

Directions: Before beginning the test, photocopy this answer sheet or remove it from the book. Mark your answer to each question by filling in the corresponding answer oval in the columns below. If a section has fewer questions than answer spaces, leave the extra spaces blank.

Section I

1Images

2Images

3Images

4Images

5Images

6Images

7Images

8Images

9Images

10Images

11Images

12Images

13Images

14Images

15Images

16Images

17Images

18Images

19Images

20Images

21Images

22Images

23Images

24Images

25Images

26Images

27Images

28Images

29Images

30Images

Section II

1Images

2Images

3Images

4Images

5Images

6Images

7Images

8Images

9Images

10Images

11Images

12Images

13Images

14Images

15Images

16Images

17Images

18Images

19Images

20Images

21Images

22Images

23Images

24Images

25Images

26Images

27Images

28Images

29Images

30Images

Section III

1Images

2Images

3Images

4Images

5Images

6Images

7Images

8Images

9Images

10Images

11Images

12Images

13Images

14Images

15Images

16Images

17Images

18Images

19Images

20Images

21Images

22Images

23Images

24Images

25Images

26Images

27Images

28Images

29Images

30Images

Section IV

1Images

2Images

3Images

4Images

5Images

6Images

7Images

8Images

9Images

10Images

11Images

12Images

13Images

14Images

15Images

16Images

17Images

18Images

19Images

20Images

21Images

22Images

23Images

24Images

25Images

26Images

27Images

28Images

29Images

30Images

SECTION I

Time—35 minutes

26 Questions

Directions: The questions in this section are based on the reasoning contained in brief statements or passages. For some questions, more than one of the choices could conceivably answer the question. However, you are to choose the best answer; that is, the response that most accurately and completely answers the question. You should not make assumptions that are by commonsense standards implausible, superfluous, or incompatible with the passage. After you have chosen the best answer, blacken the corresponding space on your answer sheet.

1. Since David Ellington became chief of police, several of the most decorated officers have resigned, the number of recruits has decreased, the department has instituted fewer community outreach programs, crime is on the rise, and polls show our police force is less popular than ever before. It’s obvious the city council appointed Ellington to undermine our civic order.

The reasoning in the argument is flawed because the argument

(A)  confuses quantitative results with qualitative results

(B)  leaps to a general conclusion based on a few anomalous instances

(C)  ignores that someone can be perceived as incompetent without actually being incompetent

(D)  assumes that because an action is followed by a result that the action was taken to bring about that result

(E)  restates something that has already been proven

2. An apartment complex is installing alarm systems in every apartment due to an increase in crime in the area. The manager reasons that he does not need to install an alarm system in apartment 3B because the tenant is a police officer and can protect himself.

Which one of the following decisions is based on a flawed reasoning that is most similar to the apartment manager’s reasoning?

(A)  A deliveryman has limited time to deliver all his packages before returning to his office. The deliveryman reasons that they should be delivered in order of largest to smallest because there might not be enough time to deliver all of them.

(B)  A candidate for mayor must prioritize the areas of the city where she must focus campaigning efforts. The candidate reasons that she spend all her resources in these areas where voters are less likely to vote for her because the other voters do not need to be convinced to vote for her.

(C)  Members of a jury are given four hours to reach their verdict or the judge will dismiss them and declare a mistrial. The jury reasons they should have the foreman review the evidence and determine a verdict because the foreman is a lawyer.

(D)  The captain of a yacht requires that all his passengers go through a safety training session. He reasons that one passenger did not have to attend because he was a former US naval officer.

(E)  A manager discovers an employee has stolen food from the break room. He reasons that he should force all his employees to go through ethics training to make sure the guilty party never steals food again.

3. Larry:    Some laws passed by Congress are considered to be bad by our nation’s citizens even though they deliver positive results. A law is good only if it delivers positive results. So, some laws considered to be bad by our nation’s citizens are actually good laws.

Carrie:    Although I agree with your conclusion I disagree with your reasons for it. Some good laws actually do not deliver positive results. But no laws that are considered to be bad by our nation’s citizens deliver positive results, so your conclusion, that some laws that are considered bad by our nation’s citizens are actually good, still holds.

Which one of the following correctly describes an error in both Larry’s and Carrie’s reasoning?

(A)  Assuming that if a law’s having a certain quality is necessary for its being a particular type of law, then having that quality is sufficient for being that type of law.

(B)  Assuming that if a particular quality is shared by two types of laws, then that quality is the only quality distinguishing the two types of law from laws of other types.

(C)  Assuming that if most laws of a particular type share a particular quality, then all laws share that quality.

(D)  Assuming that if a particular quality is shared by laws of a particular type among a particular nation, then that quality is shared by laws of that type among all nations.

(E)  Assuming that if a certain quality distinguishes one type of law from another type of law, then that quality is one of many qualities distinguishing the two types of laws.

4. Based on new evidence, anthropologists now believe that during the Neolithic Age there was a “white revolution,” when Middle Eastern milk-drinking farmers conquered the hunter-gatherer tribes of Europe, and set up permanent settlements where the hunter-gatherer communities used to exist. Previously, experts thought that representatives of the Middle Easterners only traveled through Europe and brought their ideas like evangelists, converting the indigenous people to a new way of life. This new evidence finally proves that the transition of Europe to a farming and milk-drinking society was much more violent and transformative than thought before.

Which one of the following, if true, most weakens the argument?

(A)  a discovery of several Middle Eastern villages in northern Europe with tools that show its members ate only vegetables and fruits and never raised cows

(B)  evidence that several hunter-gatherer societies drank milk long before the Middle Eastern farming communities migrated into the region

(C)  the discovery of land scarred from battle of that period as well as mass graves of the Middle Eastern farmers and hunter-gatherers

(D)  evidence that villages of Middle Easterners were established near the hunter-gatherer communities and trade existed between the two types of communities

(E)  the discovery of children with genetic markers from both hunter-gatherer and farming peoples, proving that there was sexual intermingling

5. A rise in the number of young people entering the workforce in a small country between 2000 and 2006 correlates with a rise in the percentage of young people who graduate from high school in that country. Since young people in that small country are generally either high school graduates or high school dropouts, the correlation leads to the conclusion that the country’s ability to employ young people in the workforce depends substantially on the number of people it can graduate from high school.

Which of the following statements, if true, most weakens the argument?

(A)  The percentage of young people who had completed at least two years of college was greater in 2006 than in 2000.

(B)  Between 2000 and 2006 the percentage of high school dropouts hired into the workforce rose sharply.

(C)  Most of the available jobs require using technology and performing tasks that are too complicated for high school dropouts.

(D)  A larger number of high school dropouts in the small country were hired into the workforce in 2006 than in 2000.

(E)  All workers are strongly encouraged to finish at least their high school education before applying for jobs.

6. Speaker:  The government must recognize that contemporary business firms are able to behave as irresponsibly as they want without fear of losing power. In a modern society, Davis and Blomstrom’s Iron Law of Responsibility, which says that “in the long run, those who do not use power in a manner which society considers responsible will tend to lose it,” no longer applies and never did. History shows that in the long run firms are able to obscure their bad acts with effective public relations or just wait out the short attention span of the public. Thus, a business that wants to act irresponsibly can do so without fear of losing power in society.

Which one of the following statements, if true, most weakens the speaker’s argument?

(A)  Some institutions reach a critical mass at which they are too big to fail and will, in a sense, live forever.

(B)  Some businesses that have used power in socially irresponsible ways have remained powerful.

(C)  The power of some institutions grows faster than the power of others, whether they are socially irresponsible or not.

(D)  Government institutions are as immune to the Iron Law of Responsibility as business institutions.

(E)  Public relations often are transparent and fail to mask a corporation’s truly irresponsible actions.

Questions 7–8

A number of serious amateur photographers have tested the new Apheron digital camera. With it, they observed that the camera took photographs with much finer detail both in bright and low light situations than with the 10-megapixel Norwich, even though the Norwich cameras offer a higher resolution at 10 megapixels than the 8-megapixel Apheron cameras. Given these amateur photographers’ observational findings, any serious photographer ought to choose the Apheron if she or he is buying a digital camera for both bright and low light situations.

7. The argument proceeds by

(A)  describing how a testing situation approximates the real-life conditions of ordinary use

(B)  using the claims of a subset of a group to make a recommendation to the larger group

(C)  evaluating the credibility of a recommendation made by a particular group

(D)  distinguishing between the actual reasons why a certain group did a particular thing and the best reasons for doing that thing

(E)  placing an experience in a wider context in order to explain it

8. Which of the following most seriously weakens the argument?

(A)  The lens in the Apheron digital camera is made of a different substance than that used in other digital cameras.

(B)  With digital cameras, the size of the sensor is more important than the number of pixels when it comes to taking photographs in bright or low light.

(C)  The quality of detail in photographs is only one of several factors that, when taken together, should determine whether the camera is worth purchasing.

(D)  The amateur photographers made their observations and comparisons during several different sessions and at different times of the day.

(E)  Many serious amateur photographers have no intention of buying a digital camera to take photographs under such extreme conditions.

9. Economist:  The automobile industry seems to be on the way to recovery from an earlier slump. Recent figures show that the auto companies are purchasing more equipment and spending more on R&D than ever before, indicating that they expect sales to increase in the near future.

That the auto companies are purchasing more equipment and spending more on R&D than ever before figures in the economist’s argument in which one of the following ways?

(A)  It is an inference drawn from the premise that the auto companies expect sales to increase in the near future.

(B)  It is an inference drawn from the premise that the automotive industry is recovering.

(C)  It is the primary evidence from which the argument’s conclusion is drawn.

(D)  Its truth is required in order for the argument’s conclusion to be true.

(E)  It is the phenomenon that the argument seeks to explain.

10. The amount of water consumed in Anderville on any given day in the summer is directly proportional to the heat index on that day. Since the average heat index this July was four points higher than the average heat index last August, it follows that more water was consumed in Anderville this July than last August.

Which one of the following arguments has a pattern of reasoning most similar to the one in the argument above?

(A)  The number of doctors working in the emergency room on any day is directly proportional to the number of services delivered by the emergency room that day and also directly proportional to the number of patients that visit the emergency room. Thus, the number of services offered by the emergency room on any given day is directly proportional to the number of patients that visit the emergency room on that day.

(B)  The number of doctors working in the emergency room on any given day is directly proportional to the number of nurses working in the emergency room on that day. But the emergency room employs the same number of orderlies every day. Hence, there are usually more doctors than orderlies working in the emergency room.

(C)  The bill paid by a patient at the emergency room is directly proportional to the number of medical professionals the patient sees during his or her visit. Since the number of patients going through the emergency room is increasing, it follows that the emergency room is collecting a greater amount in fees paid by patients than it used to.

(D)  The increase in patients at the emergency room is directly proportional to the amount of advertising the hospital has done the previous year. The hospital seeks to attract more patients to its emergency room by increasing the amount the hospital spends on advertising.

(E)  The amount of analgesics prescribed in an emergency room is directly proportional to the number of patients that go through the emergency room on a given day. Since the emergency room handled 15 percent more patients in the last year than in the previous year, more analgesics were prescribed in the emergency room last year than in the previous year.

11. Jason:    The solar cooker provides free energy for cooking, baking, and cleaning water. People who use it avoid indoor smoke inhalation, fire hazards, and injuries from gathering firewood. They also have environmental benefits such as reducing deforestation, lowering energy costs, and reducing CO2 emissions. The solar cooker is a simple solution to a host of problems in the developing world.

Zola:    While I agree with your arguments, I disagree with your conclusion. Solar cookers have all of those benefits. Unfortunately, it takes a long time for them to bring water to a boil—usually between two and three hours—and they require ample sunlight, which is not always available in the rainy season. In practice, they are not a perfect solution by far.

Zola responds to Jason’s argument using which one of the following argumentative techniques?

(A)  She argues that Jason’s conclusion is just a restatement of his fallacious arguments.

(B)  She argues that Jason’s conception of the ills of developing nations is too limited in scope and thus his conclusion is insufficient.

(C)  She questions the integrity of Jason’s research and whether his conclusion is based on a faulty foundation.

(D)  She gives an alternate interpretation of the assumptions that results in a different conclusion altogether.

(E)  She introduces caveats that weaken several of Jason’s assumptions.

12. Joe’s Lumber stocks only two types of 2 × 4 planks, pine and oak. Roberto never uses pine to build his tables because he likes the heavier feel of oak and believes it is more durable. When he built Marion a dining room table he bought the wood from Joe’s Lumber, so the table must be made of oak.

The argument above is most similar in its logical structure to which one of the following?

(A)  Rocky wants to go with three friends to the amusement park. His father owns both a sedan and a compact. The compact holds four people, but even though the sedan’s engine has more horsepower, if Rocky borrows a car from his father he will borrow the compact.

(B)  The Roddick Corporation offers Pedzisai a choice between an assignment in Sweden and an assignment in South Africa. Pedzisai would never accept an assignment in his home country of South Africa if it meant turning down an assignment in Europe, so if he chooses an assignment it must be the assignment in Sweden.

(C)  If Georgia wants to ride her bike from her apartment to work she must either take a thoroughfare or go through a maze of neighborhoods. The neighborhood streets are always full of potholes and the thoroughfare has too much traffic, so she never rides her bike to work.

(D)  Antonio can take a 15-minute break in the morning or an hour and a quarter for lunch later. The haircut Antonio planned takes 30 minutes, but his barber had only morning appointments available, so no matter what Antonio will have to get his haircut another day.

(E)  If the team owner does not fire his coach, his football team will rebel and they will lose more games. Losing the coach will also mean losing more games, so if no alternative solution presents itself, the team will lose more games.

13. A building inspector has been accused of ignoring serious structural defects on a building. Although the records have been lost and the building has since been demolished, his inspections of more recent buildings have been reviewed and found to be flawless. Therefore, the accusation should be dismissed.

Which one of the following contains questionable reasoning that is similar to that in the argument above?

(A)  A computer scientist was accused of spreading a computer virus to several large computers, but the charge should be dismissed since the hard drives of those computers have been erased and no record of his malfeasance exists. Also, since then, his work has been untainted by any such actions.

(B)  A teacher was accused of allowing several students to pass a course when they actually failed. This accusation should be discussed at length because even though those students went on to be successful in later grades, the teacher may have passed other undeserving students.

(C)  Politician A was accused of stealing parts of a speech given by Politician B. This charge is without foundation and should be disregarded because Politician A was not even born when Politician B gave that speech and there exists no recording of the original speech.

(D)  A restaurateur is accused of using ingredients past their expiration dates. The accusation seems justified even though there is no evidence of it, because several cases of food poisoning can be linked back to the restaurateur’s place of business.

(E)  A financial manager is accused of stealing funds from his clients. The charges should be ignored because even though the records show there has been some malfeasance, the identity of the accuser has not been disclosed.

14. Every employee who takes public transportation to and from work eats lunch in the company break room. It follows that some employees who have their own office do not take public transportation to work.

The conclusion of the argument follows logically if which of the following is assumed?

(A)  Some employees who do not have offices eat lunch in the company break room.

(B)  Some employees who eat lunch in the company break room have an office.

(C)  Every employee who eats lunch in the break room has an office.

(D)  Some employees who do not have an office do not eat lunch in the company break room.

(E)  Every employee who eats lunch in the company break room takes public transportation to and from work.

15. Archaeologists have found underwater etchings near the jungle city of Manaus, following a drought in the Brazilian Amazon. Previously, archaeologists studying the Amazon believed that the rainforest was too inhospitable to host a major civilization and that the only civilizations in the area were nomadic. The new discovery proves this theory incorrect and that thousands of years ago the Amazon was home to large civilizations.

Which of the following statements, if true, would most weaken the above argument?

(A)  Remnants of large established villages with well-trodden roads going by or through them were unearthed near the location of the drawings.

(B)  The etchings exhibited an unexpected sophistication for that period, yet they still showed the telltale signs of being of that period.

(C)  Archaeologists proved that a nomadic artist created the etchings and very similar etchings almost 100 miles away.

(D)  The drought that exposed the submerged etchings was the first in that area in more than 2,000 years.

(E)  The etchings included a representation of hunters walking across a plain and people building small domiciles.

16. A new study observed people who watch television and their happiness level. The study indicated that people who have more channels and thus more choices of what to watch are not necessarily happier than those with fewer choices of what to watch. Thus, people in highly industrialized nations who have more choices in all aspects of their daily lives are not happier than those in less developed nations with fewer choices in their daily lives.

The above argument depends on the presupposition that

(A)  it is equally likely that people in less developed nations have as many television channels as people in highly industrialized nations

(B)  the inverse relationship between happiness and the number of television channels is the same among other categories of goods and services regardless of other factors related to where people live in the world

(C)  people in less developed nations are unfamiliar with the number and quality of choices available to those in highly industrialized nations

(D)  people in highly industrialized nations have visited less developed nations and determined that their choices are of lesser quality and less likely to make them happier

(E)  the relationship between the number of television channels and happiness is directly related to the stage of a society’s industrialization and thus an industrialized nation will feel the effects of such disparities more significantly

17. A high school cheerleading squad held its annual bake sale on the first day of school to raise money for the coming year. In order to increase sales this year the squad decided to lower the prices on everything. Despite their strategy, the cheerleaders had lower sales than in any previous year.

Each of the following, if true, contributes to reconciling the apparent discrepancy indicated above EXCEPT:

(A)  Student enrollment at the high school dropped significantly from the previous year to this year.

(B)  Other school groups planned their fund-raisers for the same day, which had not been done in previous years.

(C)  Because of the increase in prices for sugar and flour, the cost of making the baked goods increased this year.

(D)  The faculty handed out gift baskets on the first day of school that included cupcakes and other sweet items.

(E)  Due to construction on a new auditorium the cheerleading squad was not able to locate their bake sale tables in the same well-known location as in previous years.

18. The Redville Community Center, a nonprofit organization, was facing a budgetary crisis due to rising utility costs and expensive building repairs that were necessary to keep the center open. The board of directors decided to raise the membership fees and assess a building fee to raise the necessary funds. Toward the end of the year the board discovered that the costs were less than they thought and the fee changes actually gave the center a budget surplus. The board decided to use the surplus funds to buy a new van to shuttle people from the nearby nursing home to the center even though the old van was still serviceable. But before doing so, the board should obtain permission from the members who paid the additional fees.

Which one of the following policies, if put into effect, would most justify the position advocated above and yet place the least restriction on the use of funds by community centers?

(A)  Beneficiaries of a nonprofit organization with a vested fee-based interest should be considered to be placing their trust in the directors of those organizations to use the money wisely according to whatever circumstance might arise.

(B)  Beneficiaries of a nonprofit organization with a vested fee-based interest cannot delegate to the directors of those organizations the responsibility of allocating the funds received for various purposes consonant with the purposes of the organization as the directors of the organization see fit.

(C)  Directors of nonprofit organizations cannot allocate assessments to any purposes for which the directors had not specifically earmarked the funds in advance.

(D)  Fees assessed by a nonprofit organization for a specific purpose should only be used for that purpose, or if that is unable to happen then should only be used according to the express wishes of those who pay the assessments.

(E)  Directors of nonprofit organizations who assess additional fees for specific purposes must return funds received from such assessments if more money is received than can practicably be used for the specified purpose of the assessment.

19. Scientist:    My peers have said that my theory regarding molecular cohesion in glass is based on sheer conjecture and has no experimental basis. This is simply not true. I’ve based my theory on several findings that have been published recently by reputable research organizations. Even though I have not reviewed every detail of their experiments, I am confident in their work. Besides, you may recall that I was asked to develop theories on molecular cohesion in wood fibers five years ago and I used findings from the same labs then as well.

The scientist’s argument is LEAST vulnerable to which one of the following criticisms?

(A)  It bases a conclusion about the scientific findings of the research organizations on uncertain recollections.

(B)  It assumes that the experiments done by the research organizations are unaffected by bias or human error.

(C)  It assumes that the experiments done by the research organizations are the only work necessary to develop a scientific theory.

(D)  It hastily concludes that the experiments done by the research organizations are accurate, without having studied them in detail.

(E)  It assumes that having in the past used the experiments done by the research organizations as a basis to develop a scientific theory justifies using them to develop the current theory.

Questions 20–21

The city performed its annual testing of lead levels in the drinking water and discovered that about 14 percent of the samples taken exceeded federal standards for allowable lead levels. Regardless of these findings, the lead levels, even in the ones that exceeded federal standards, were still too low to pose any immediate health threat. So, it’s perfectly safe for the city’s residents to drink all the tap water they want.

20. Which one of the following is an assumption on which the argument depends?

(A)  The city’s residents often disregard federal guidelines when it comes to public health issues because the standards are overly stringent.

(B)  The most dangerous contaminant in drinking water is lead.

(C)  Statistical sampling is not the most accurate basis for determining health threats even though the federal government and the city must depend on them.

(D)  Lead levels even slightly above federal standards do not have long-term effects on people’s health.

(E)  People feel safer when they heed warnings from the federal government even if they are not entirely accurate.

21. Which of the following, if true, most strengthens the argument?

(A)  Most lead in city drinking water is absorbed by the lining of the pipes.

(B)  Due to budgetary cuts the city had to test water sourced from one central location instead of sampling randomly throughout the city as in past years.

(C)  Studies show that children who absorb levels of lead above federal standards experience adverse long-term effects.

(D)  The local hospital has had an increase in lead-poisoning cases among people who drink large quantities of water on a regular basis.

(E)  Several medical associations have lobbied for higher standards and harsher punishments on cities that do not take actions to reduce lead contamination in their drinking water.

Questions 22–23

Jaime:    The life span for both African and Asian elephants is much shorter in zoos than in the national reserve parks. Studies have proven this by comparing the life spans of elephants in zoos to the life spans of elephants in nature reserves in both Kenya and Myanmar. Thus, our government should take a serious look at how zoos treat their elephants and possibly implement a temporary ban on the acquisition of new elephants until this issue is resolved.

Eleanor:    But both the reserves you mention are protected areas where the animals do not face the threat of poaching and other unnatural dangers. Also, the data in those studies go back more than 40 years and do not take into account more recent advances in captive animal care that will most definitely extend the life spans of elephants currently in zoos.

22. Which one of the following is an assumption on which Eleanor’s argument depends?

(A)  There are genetic differences between African and Asian elephants that effect variance of life spans between those particular species.

(B)  Predators and diseases within the confines of a nature reserve affect the life span of an elephant to a lesser extent than threats an elephant faces outside the parks.

(C)  The training of zookeepers and those who manage nature reserves is exactly the same; thus, the care of the elephants should be exactly the same.

(D)  A zookeeper in 1970 was unable to help extend the lives of elephants in captivity due to budgetary restraints that plagued zoos at that time.

(E)  The professionals that manage nature reserves are more aware of the threat of poachers than zookeepers; thus, they do a better job of protecting their elephants from them.

23. Eleanor’s rejection of Jaime’s conclusion employs which one of the following techniques of argumentation?

(A)  Producing a single contradicting example that establishes that a stated generalization is false

(B)  Questioning the validity of a conclusion because the supporting statements cannot be experimentally verified

(C)  Pointing out that potentially more favorable evidence exists that has been neglected

(D)  Rejecting a problematic correlation and a potential variance shift with relation to the supporting statements that underlie the conclusion

(E)  Reanalyzing the underlying assumptions and pursuing a wider data sampling that underlie the conclusion

24. A teacher decided to challenge 10 of her students to do their mathematics work without a calculator or computer for three months, while the rest of her class was able to use whatever tools they had at their disposal. She discovered that the 10 students’ scores improved on their homework assignments and their exams. On the basis of this experiment the teacher determined that students perform better when forced to complete their work manually rather than depending on machines.

Which of the following would, if valid, most weaken the reasoning above?

(A)  The 10 students were performing poorly in the teacher’s course before the experiment.

(B)  The 10 students were being tutored by other students who did have the advantage of a calculator or computer.

(C)  The 10 students happened to be the best students at mathematics in the class.

(D)  During the three months of the experiment, the class studied mathematics that only the 10 students had studied previously.

(E)  The teacher coached the students on how to do their homework without the help of a calculator or computer.

25. A recent survey showed that 60 percent of employers polled believe that an employee should be fired from the job if the employee has been charged with a crime, whereas 40 percent believe that an employee should be fired only if the employee has been convicted of a crime. Therefore, more employers believe that an employee should be fired if charged of a crime than believe they should be fired if convicted.

The reasoning above is flawed because it

(A)  uses a sample of the population to draw a conclusion about the general population

(B)  bases a conclusion on an ambiguous term in the supporting statements

(C)  uses two different beliefs to draws a conclusion about a different belief altogether

(D)  is based on premises that cannot all be true

(E)  confuses a sufficient condition with a required condition

26. Editorial:    History has shown that even though politicians promise that the profits from a lottery will be devoted to educational purposes, the funds are inevitably redirected to other purposes that have nothing to do with education. Therefore, our representatives in the state legislature should not support the proposed lottery because they will inevitably break their promise to use the profits to fund education and direct the funds elsewhere.

The reasoning above is flawed because it

(A)  draws a conclusion about a specific population based on a study of a larger population

(B)  uses historical data to draw incorrect conclusions about a similar situation in the future

(C)  presupposes that a pattern of behavior under one set of conditions will recur under a completely different set of conditions

(D)  rejects a proposed solution based on the track record of an altogether different solution

(E)  assails a proposed lottery by attacking those who support it rather than the merits of the proposed lottery

STOP

IF YOU FINISH BEFORE TIME RUNS OUT, CHECK YOUR WORK ON THIS
SECTION ONLY. DO NOT GO ON TO ANY OTHER TEST SECTION.

SECTION II

Time—35 minutes

24 Questions

Directions: The questions in this section are based on the reasoning contained in brief statements or passages. For some questions, more than one of the choices could conceivably answer the question. However, you are to choose the best answer; that is, the response that most accurately and completely answers the question. You should not make assumptions that are by commonsense standards implausible, superfluous, or incompatible with the passage. After you have chosen the best answer, blacken the corresponding space on your answer sheet.

1. Chancellor of Mayfield Academy:    Mayfield Academy must grow if it is to survive, so as we have agreed, efforts should be made to attract students in all grades K–12. The best strategy for attracting students is to build new facilities for athletics, computer science, and the arts, including a new auditorium. Parents demand that their children receive the very best facilities to explore their interests to the fullest. Also, as other schools are investing in similar facilities, we must keep up with the competition. Therefore, parents will move their kids to our school as our facilities match or surpass other institutions.

The argument leads to the conclusion that

(A)  the Mayfield Academy should attract more students

(B)  parents who want the best facilities for their children’s interests should choose Mayfield Academy

(C)  parents should consider public as well as private institutions

(D)  the Mayfield Academy should invest in new facilities

(E)  parents who have children in public school should transfer them to Mayfield Academy

2. Business executive:    Attempting to create an ethical company by teaching ethics to our employees is a waste of time and money because the corporate structure at its foundation is inherently neither ethical nor unethical. No matter what we do, people will inevitably act in an unethical manner. All we can do is create monitoring systems to prevent problems from occurring and to protect the company when they do.

Ethicist:    To claim that we should not train employees in ethics because they will inevitably act unethically makes about as much sense as arguing that we should not spend money on driver’s education because all drivers will inevitably cause an accident.

The method the ethicist uses to object to the business executive’s argument is to

(A)  argue that there are problems that time and money, no matter how judiciously spent, cannot solve

(B)  attack the character of the business executive rather than the position the business executive is taking

(C)  show that the executive’s line of reasoning would lead to an unacceptable conclusion if applied to a different situation

(D)  show that the executive must present more evidence to substantiate the business executive’s position

(E)  explicate a dilemma that is central to the business executive’s argument

3. Renfield should not be promoted to the management position. His performance as a member of our staff, while exemplary, has shown little of his management ability. Everything he has said and done up to this point has been directed toward obtaining this promotion rather than showing us that he has the ability to manage other people. Therefore, we cannot trust that he will be an effective manager once in the position.

Which one of the following is an assumption on which the argument relies?

(A)  The duties of a staff member do not in and of themselves prove that the person is capable of managing people.

(B)  Renfield cannot be trusted even with his current duties as a member of the staff.

(C)  When Renfield obtains the promotion, he does not intend to act in the best interest of the staff under his management.

(D)  The staff will not follow Renfield’s lead as a manager once he receives the promotion.

(E)  Managers rarely are promoted from the staff because they are unable to handle the higher level of responsibility.

4. Real progress in society is not something that can happen without significant and transformative action. It only comes when those who are in power have the courage to overcome significant opposition and overturn previously accepted norms that are detrimental to society.

The reasoning in the argument is fallacious because the argument

(A)  undermines its own premise that a particular attribute is present in all instances of a particular phenomenon

(B)  concludes that, because an influence is the paramount influence on a particular phenomenon, that influence is the only influence on that phenomenon

(C)  denies that the observation that a particular pattern is common to phenomena within society might contribute to observing a causal explanation of the phenomenon

(D)  concludes that the characteristic of a type of phenomena in society occurring at one time is characteristic of similar phenomena at all times

(E)  selects one influence on a particular phenomenon in society as indicating that its influence outweighs any other influence on those phenomena

5. Educator:    Scholars are concerned that the increasing popularity of online universities and low-residency degree programs will render the classroom experience obscure. They object to this trend, saying that online courses do not offer the level of collaboration and support that the traditional classroom does. At Plymouth Online, however, we offer a fully interactive experience with web video, web-based collaboration software, online chat, and video conferencing sessions. It is therefore possible to join our online university and experience a very close approximation of the in-classroom experience, and in some situations, the experience is a richer and more satisfying one.

The educator’s argument proceeds by

(A)  referring to a scholarly authority to challenge a widely held belief

(B)  questioning the accuracy of evidence given in support of an opposing position

(C)  offering a counterexample to a prevalent belief among experts in the field

(D)  proposing an alternative sociological explanation for a pedagogical practice

(E)  making a distinction between instructional approaches

6. Until recently it was believed that weight training did not help children but in fact did harm and possibly stunted their growth. A new study has determined that weight training among children between ages 6 and 18 helped them grow stronger even though they did not gain muscle mass like adults. Therefore, our government should require weight training for all children in public schools who are between the ages of 6 and 18.

Which one of the following statements, if true, most seriously weakens the argument?

(A)  The school year is only nine months of the year, and with children, weight training must be done on a consistent basis all year round. A home-based program is required for success.

(B)  Since the children do not gain muscle mass, measuring their progress will be difficult without the specialized instruments used in the study.

(C)  The study was performed with subjects from only one geographic location where weight training is more culturally acceptable.

(D)  The gain in strength over a nine-month school year will be minimal. It takes at least a year for the children to show significant results from weight training.

(E)  Children do not respond positively to government requirements and low morale may adversely affect results of any government-instituted weight training requirements.

7. According to a study, an exercise program combining aerobics and weight lifting can help people with type 2 diabetes lower their blood sugar. Such a combined program offers greater benefits than doing either exercise type alone. Therefore, all people with type 2 diabetes should undergo a physical training program that includes both aerobics and weight lifting.

Which of the following statements would most weaken the argument?

(A)  People with type 2 diabetes are less likely to be able to physically manage both aerobics and weight lifting.

(B)  Aerobics and weight lifting are mutually exclusive and thus impossible to be done together.

(C)  The physical demands of a combined program could have negative physical effects that negate the benefits of lowering the person’s blood sugar.

(D)  Some individuals with type 2 diabetes, due to their particular physical requirements, need only one of the exercise activities to obtain the benefits to their blood sugar.

(E)  The study took into account all important indicators, including blood sugar, that may be affected by exercise.

8. During the first 140 million years of existence on earth, mammals were relatively small, ranging from 2 to 22 pounds, but once the dinosaurs became extinct, mammals grew in size up to 1,000 times bigger and as large as 17 tons. Once the herbivorous dinosaurs were gone the mammals no longer needed to compete for space to roam and vegetation for food. They could eat all the food they wanted, and the larger mammals were better able to fend off predators. This is why we have our hippopotamuses and giraffes of today.

If the statements above are true, which one of the following must also be true on the basis of them?

(A)  If the dinosaurs had lived longer, mammals today would be much smaller yet still herbivorous.

(B)  If the hippopotamus we know today was a 1,000 times smaller, other mammals would also be significantly smaller.

(C)  If other larger nonmammals had survived the event that made the dinosaurs extinct, modern mammals may have been much smaller due to competition for resources.

(D)  If mammals had been larger during the time of the dinosaurs, the resulting competition would have made both extinct.

(E)  If hippopotamuses and giraffes had evolved as carnivores and not herbivores, large mammals most likely would not have competed with the dinosaurs for space and food.

9. At an auction, nobody wants to buy the statue by Alberto Giacometti more than Jody, but Jody will not be participating in the bidding, so no matter how much the auctioneers lower the minimum bid not one person will bid on the statue.

The flawed reasoning in the argument above most closely parallels that in which of the following?

(A)  The jockey who most wants to ride the horse Black Lightning in the next race is Cornwall, but he suddenly told the owner he could not ride in the next race. This means the other jockeys who had given up hope of riding Black Lightning will double their efforts.

(B)  Better than anyone, Larry can spot a forgery of a Renaissance painting, but he has yet to find any flaws in Renny’s painting. So there must be a forgery among the other paintings in his collection.

(C)  If anyone can translate this ancient text it is Professor Ricardo, but he is currently engaged in a project to translate a much more significant text and will probably not be interested in doing this smaller project. So we will have to hope that we can find someone of equal stature to take on the translation.

(D)  Even though Emilio is the most intent of anyone to obtain the sales position, he is not applying for the position. It follows that nobody else will apply for the sales position no matter how high a salary is being offered.

(E)  Sherry wanted to join an extra-credit group project for her science class, but each time they called a meeting she was too busy with cheerleading practice to join them. So, the closer she gets to becoming head cheerleader, the less time she can devote to her studies.

10. If the conductor does not want to participate in the New York competition, then we should consider other competitions. If the orchestra does not want to participate in the New York competition, then we should skip the competitions altogether. And, it is bound to be the case that either the conductor or the orchestra does not want to participate in the New York competition.

If the statements above are true, which one of the following must also be true?

(A)  If the orchestra agrees to participate in the New York competition, then we should skip the competitions altogether.

(B)  We should consider other competitions only if it makes it more likely that both the conductor and the orchestra will participate.

(C)  We should attempt to convince both the conductor and the orchestra to participate in the New York competition.

(D)  If the conductor agrees to participate in the New York competition, then we should skip the competitions altogether.

(E)  We should consider other competitions only if the conductor is more likely to participate.

11. It is ridiculous to claim that unless a newspaper covers an event it must not be newsworthy, as one can see by rephrasing the statement to read: no event is newsworthy unless it appears in a newspaper.

The pattern of reasoning in which one of the following is most similar to that in the argument above?

(A)  The claim that every politician who is elected to Congress will become corrupt is absurd, as one can see by rewording it: No politician is elected to Congress without becoming corrupt.

(B)  The claim that every politician who is elected to Congress will become corrupt is absurd, as one can see by rewording it: No politician who failed to be elected to Congress will become corrupt.

(C)  The claim that every politician who is elected to Congress will become corrupt is absurd, as one can see by rewording it: No politician can become corrupt unless she is elected to Congress.

(D)  It is ridiculous to claim that a student who has never been accused of cheating has never cheated, as one can see by rewording it: Every student who cheats gets accused of cheating.

(E)  It is ridiculous to claim that a student who has never been accused of cheating has never cheated, as one can see by rewording it: Every student who is accused of cheating has cheated.

12. Philosopher:    Pragmatism is the view that meaning or worth is determined by practical consequences. If a course of action has the desired effect, then it is good. So if a theory works in practice, it is right. If it does not work, then it must be wrong. Pragmatism, however, is a dangerous philosophy because it is impossible to develop a general truth that applies to everyone or gives guidance for making moral or ethical decisions. What is “good” for one person may not be for someone else.

The philosopher’s reasoning is most vulnerable to criticism on which one of the following grounds?

(A)  Discrepancies between perceptions of the practical and good are a matter of people joining together and deciding what actions and inactions are practical and good at a societal level.

(B)  The individual cannot always successfully determine what is practical and good. Only through experience can one develop better judgment. Thus experience is the great equalizer and resolves the discrepancies between how a course of action is viewed.

(C)  While it is true that one person may believe an action is practical and good, the action may be in conflict with another person’s action only because the other person has a poor sense of what is right and wrong.

(D)  There is a possibility that a person’s action can be practical but wrong when another person in proximity observes that the action is in conflict with the values of society.

(E)  Other philosophies have failed to adequately determine a general truth for human action, so pragmatism is no different from other philosophies.

13. Press release:    A committee of physicians analyzed nearly 1,000 publications and determined that the high level of vitamin D and calcium recommended by physicians and testing labs to sustain bone health is unnecessary. Food producers are adding vitamin D and calcium to foods people eat every day, not to mention that people get vitamin D from exposure to sunlight. Therefore, everyone should stop taking high levels of vitamin D and calcium immediately.

Which one of the following points to a weakness in the reasoning in the press release’s argument?

(A)  Vitamin D and calcium work together rather than separately to sustain bone health.

(B)  The physicians analyzed other publications instead of developing their own study and analyzing a direct study of patients.

(C)  Physicians recommend high levels of vitamin D only to those who have tested for poor bone health.

(D)  High levels of vitamin D and calcium have benefits other than sustaining bone health.

(E)  Many of the foods with added vitamin D and calcium are breakfast foods and people tend to skip breakfast on a regular basis.

Questions 14–15

The board of directors of Company X has decided to sell its underperforming divisions that have been dragging down the stock price. To some analysts this seems like a drastic move, but the president of Company X has long maintained that among the troubled divisions are some with antiquated systems and low-quality products. He has argued that it would be too costly to improve their operations. Hence, the board’s action would not hurt but would actually help Company X’s stock price.

14. The conclusion drawn depends on which one of the following assumptions?

(A)  All the companies being considered for sale are 100 percent owned by Company X.

(B)  Only a very savvy buyer will be willing to buy such underperforming divisions from Company X.

(C)  Company X will be able to sell its underperforming units only after it has improved their operations.

(D)  All the companies the board decides to sell will include those recommended by the president for sale.

(E)  Buyers of companies are unable to recognize an underperforming division.

15. Which one of the following, if true, most weakens the argument?

(A)  The stock price is determined not just by the company’s highest-performing divisions but by the interaction among all its divisions.

(B)  The better-performing divisions command a much higher price than the underperforming units.

(C)  The underperforming divisions have always been thought to be poor investments by Company X.

(D)  Buyers who buy underperforming divisions in order to sell them later at a higher price care little about the resulting inflation in the marketplace, but large companies like Company X are highly concerned about the resulting inflation.

(E)  The directors are currently conducting research to find a means other than selling off its underperforming divisions that would improve the company’s stock price.

16. A new restaurant that survives beyond six months is popular either among its local patrons or among the food critics. Last year, all the new restaurants that were popular with the food critics were also very popular with its local patrons. Therefore, every new restaurant that survived beyond six months last year was popular among the food critics.

The pattern of reasoning in which one of the following arguments is most similar to that in the argument above?

(A)  All auto service garages in the Caedmon Township will do maintenance on both foreign and domestic automobiles. Larry’s Auto Shop is an auto service garage that will not perform maintenance on foreign automobiles. Hence, Larry’s Auto Shop is not in the Caedmon Township.

(B)  In their second year, all apprentices at the Willow Construction Company study dry wall or cabinetry. This year, all the apprentices who are studying dry wall are also studying cabinetry. Therefore, every apprentice at Willow is studying dry wall.

(C)  Former members of Congress either go on to teach at their hometown university or write a book about their years in politics. Mary Seldon is a former congresswoman who is teaching at Riverside University in her hometown of Riverside. Therefore, Seldon is not writing a book about her years in politics.

(D)  Every bestselling novel published last year is both well written and has a suspenseful plot. The novel Jacob Rain published last year is well written and has a suspenseful plot. Therefore, Jacob Rain’s novel is a bestseller.

(E)  In order to succeed, most new movies require either a big advertising campaign or a cast with big stars. But most movies with a cast of big stars automatically get a big advertising campaign. Hence, a movie that has a cast of big stars is guaranteed to succeed.

17. It has been discovered that a rice plant has protein molecules that recognize and bind to specific molecules on invading organisms, signaling the plant to mount an immune response and fend off microbial infection and disease. This immune response is not at all dissimilar to a human immune system’s method of warding off diseases. It will not be long before we can discover how plants fight off the common cold.

Which one of the following is an assumption on which the argument depends?

(A)  The same diseases that attack humans also attack plants.

(B)  The common cold is a disease that the human immune system fails to ward off.

(C)  Because the immune system of plants and humans is similar, diseases attack both in the same manner.

(D)  The immune system of a rice plant is the same as the immune system of other plants.

(E)  Protein molecules bind to the invading molecules in plants in the same configuration that they do in humans.

18. Joseph:    Evening customer service representatives at a computer company handle complaints 20 percent faster than day customer service representatives on average, yet they offer similar quality service when handling complaints. So, the evening representatives work more efficiently.

Davis:    Your conclusion is unfair. Different times of the day bring different demands on the customer service representatives. Since the evening calls are more often about home computer problems, they are less complex problems than those during the day when more business-oriented calls come into the customer service center.

The issue in dispute between Joseph and Davis is

(A)  why the evening customer service representatives are able to remain on the late shift as opposed to the day shift

(B)  the relationship between the time of day and the time it takes for a customer service representative to handle a customer complaint

(C)  how the company may decide to move customer service representatives back and forth between the evening and day shift to increase efficiency at handling complaints

(D)  the accuracy of the figure of 20 percent with regard to the difference in speed of handling complaints between evening and day customer representatives

(E)  the reason why evening customer service representatives are able to offer equal quality service as day customer representatives while spending less time with each customer on average

19. In a legislature much of the legislation is written by industry experts who also serve as lobbyists on behalf of their respective industries that will ultimately be affected by the legislation they draw up. Since these experts know the industry much better than the legislators, they can better design legislation that will do what the legislators require without harming the companies or entities that must abide by the resulting regulations. But clearly this strategy is based on poor reasoning. After all, as lobbyists, industry experts are paid to represent the interests of their employer and any legislation they write will most likely be weakened in the favor of their employer’s self-interest and not in the best interest of the legislature’s constituents.

The point made by the author is that the most common way of creating legislation might not be in the best interest of the legislature’s constituents because

(A)  many industry experts might let their self-interest as industry lobbyists affect their role, as writers of legislation, in acting on behalf of elected officials

(B)  most industry experts, thanks to outside lobbying contracts, are heavily influenced financially to write unfair legislation

(C)  most legislators would be less corrupt in writing legislation than industry experts would be

(D)  many industry experts create weakened legislation in order to obtain lucrative jobs in their respective industries some day

(E)  many industry experts are paid generously and want to keep the lobbying jobs by creating weak legislation that favors their respective industries

20. All diamonds cut by Richman are over three carats, and all its rubies are less than three carats. Most of the precious stones cut by Richman have a clarity rating below SI2. All the diamonds and rubies cut by Allister have a clarity rating above SI2 and are less than three carats. Ellington Jewelry, which only buys its diamonds and rubies from these two companies, only purchases stones that are less than 3 carats. Ellington is currently purchasing a large diamond shipment.

If the statements above are true, which of the following must be true on the basis of them?

(A)  Ellington buys only diamonds.

(B)  All the stones from Allister have a lower clarity than those from Richman.

(C)  The diamond shipment being purchased has a clarity rating above SI2.

(D)  The diamond shipment being purchased belongs to Richman.

(E)  Ellington does not purchase stones from Allister.

Questions 21–22

Coal plant manager:    No matter what technical advances come over the next 30 years, coal will remain the most cost effective source of energy. Despite what some people claim, the cost of alternative sources of power like solar and wind technologies will never drop to a level that will compete with coal. Coal is here to stay.

Environmentalist:    I disagree. There have been significant improvements in solar cells and wind turbine technologies over the last 10 years. They are more efficient than ever. These new technologies continue to advance while the coal industry has been slow to adopt the latest clean coal technologies. Eventually, coal will face obsolescence.

21. The reasoning in the environmentalist’s argument is flawed because the argument

(A)  ignores the length of time that it will take for alternative sources of energy to become competitive with coal

(B)  fails to acknowledge the cost advantage of coal over alternative sources of energy like solar and wind technologies

(C)  mistakes the coal plant manager’s discussion of technical advances as a statement that the coal industry intends to adopt clean coal technologies

(D)  overlooks the possibility that coal and alternative technologies might collaborate to create a cleaner energy future

(E)  assumes that costs of alternatives to coal will drop with the introduction of more efficient solar and wind technologies

22. Which one of the following, if true, most strongly supports the environmentalist’s counterargument?

(A)  Engineers have discovered limitations to technological advancement in the coal industry that do not exist in the solar and wind power industries.

(B)  Thirty years is too small a time window to judge whether one energy technology will win out over another.

(C)  A direct relationship has been determined between technological advancements in the energy sector and the cost-adoption factors.

(D)  A coal company in one state is currently converting its facilities over to clean coal technology.

(E)  Improvements in solar and wind technologies have accelerated but costs have only slightly improved over the last 10 years.

23. Randy’s boss asked him to write a management report concerning the training of new financial advisors coming into the company, but because of other projects assigned to him he put it off until the day before he is supposed to make a presentation based on the report to his boss. Randy could still complete the report by the deadline, but only if he works on it all evening without interruption. However, one of his clients invited him to a dinner party and requested that Randy remain afterward to discuss the client’s portfolio in detail. Thus, Randy will be forced to choose between satisfying his boss and his client obligations.

The argument proceeds by

(A)  providing one version of events and an alternative version of events where both exhibit an incompatibility among the factors involved

(B)  explaining the inherent difficulties in a situation by using another situation with equivalent difficulties

(C)  showing how one set of responsibilities with its own conditions that need satisfying is incompatible with another set of responsibilities and creates for the individual a conundrum

(D)  exhibiting the struggles involved as one set of events creates another set of events that leaves the subject in a quandary

(E)  delving into how one person’s failure to act in a situation leads to a difficult conundrum and how the person’s choices in that situation can lead to inevitable harm to the other parties involved

24. Bumblebees have been discovered to visit and pollinate red or striped snapdragons more often than white or pink flowers. Bumblebees are the most important pollinator of the snapdragon because the pollen carried by the bumblebee is necessary for the flower to open its petals. Thus, in order to increase the population of bumblebees, nurseries are being encouraged to grow more of the striped and darkly pigmented varieties of snapdragons.

Which of the following can be properly inferred from the passage?

(A)  The bumblebee population is in danger from a lack of striped or darkly pigmented varieties of snapdragons.

(B)  If there are not enough striped or darkly pigmented snapdragons, then all snapdragons will fail to open their petals.

(C)  If there are more striped and darkly pigmented snapdragons, then other insects will be forced to open the petals of the white and pink varieties.

(D)  The bumblebee population is smaller than desired, and more striped and darkly pigmented snapdragons will help increase their numbers.

(E)  The red and striped varieties of snapdragons are endangered, and more bumblebees are needed to increase their numbers.

STOP

IF YOU FINISH BEFORE TIME RUNS OUT, CHECK YOUR WORK ON THIS
SECTION ONLY. DO NOT GO ON TO ANY OTHER TEST SECTION.

SECTION III

Time—35 minutes

26 Questions

Directions: Each group of questions in this section is based on a set of conditions. In answering some of the questions, it may be useful to draw a rough diagram. Choose the response that most accurately and completely answers each question and blacken the corresponding space on your answer sheet.

Questions 1–6

A mechanic is considering using coils D, E, G, H, I, J, and K to replace old ones in a machine. The mechanic has devised a test to determine their quality, but the mechanic has only so much time to test the coils. Only those coils that are tested can be used in the machine. The selection process must meet the following requirements:

If G is tested, I is tested.

If E is tested, G is tested.

D is tested.

H is not used unless J is tested.

D is not used unless H is tested.

If J is used and I is tested, K is used.

1. Which of the following could be a complete and accurate list of the coils that are tested?

(A)  D, E, G

(B)  D, E, I

(C)  D, E

(D)  D, I

(E)  D, G

2. Which one of the following could be true?

(A)  E and three other coils are the only coils tested.

(B)  E and two other coils are the only coils tested.

(C)  E and one other coil are the only coils tested.

(D)  D, G, and H are the only coils tested.

(E)  I and J are the only coils tested.

3. If J is not tested, which one of the following must be true?

(A)  D is not used.

(B)  H is not tested.

(C)  D is tested but H is not used.

(D)  D is used but H is not used.

(E)  H is tested but not used.

4. If E and five other coils are the only coils tested, and if exactly three coils are used in the machine, then which one of the following could be an accurate list of the coils used?

(A)  D, I, J

(B)  D, H, J

(C)  E, G, I

(D)  H, I, K

(E)  E, G, J

5. If every coil that is tested is used, and if I is used, then each of the following coils must be tested EXCEPT:

(A)  K

(B)  H

(C)  J

(D)  D

(E)  G

6. If K is not used, and if exactly four coils are used, then which one of the following must be false?

(A)  I is used.

(B)  J is used.

(C)  G is tested.

(D)  H is tested.

(E)  E is tested.

Questions 7–13

Exactly seven children—Barry, Ezra, Jaime, Karly, Pakhi, Sharon, and Usef—go to a craft class at the same time. During class, they have a choice of three activities: ceramics, drawing, and origami. Each child participates in exactly one of the activities. The activities occur only once and one activity at a time. The following restrictions must apply:

Exactly twice as many of the children choose drawing as choose ceramics.

Sharon and Usef participate in the same activity as each other.

Ezra and Karly do not participate in the same activity as each other.

Barry and Pakhi do not participate in the same activity as each other.

Barry participates in either ceramics or origami.

Jaime participates in drawing.

7. Which of the following could be a correct matching of children to activities?

(A)  Karly-drawing; Pakhi-drawing; Usef-origami

(B)  Karly-origami; Pakhi-origami; Sharon-origami

(C)  Ezra-drawing; Pakhi-drawing; Sharon-ceramics

(D)  Barry-origami; Ezra-ceramics; Sharon-ceramics

(E)  Barry-drawing; Ezra-origami; Karly-drawing

8. Which one of the following could be a complete and accurate list of the children who do not choose drawing?

(A)  Barry, Ezra, Pakhi

(B)  Barry, Sharon, Usef

(C)  Barry, Karly, Usef

(D)  Barry, Karly

(E)  Barry, Pakhi

9. Each of the following must be false EXCEPT:

(A)  Exactly two children choose origami.

(B)  Usef is the only child to choose origami.

(C)  Exactly three children choose drawing.

(D)  Pakhi is the only child to choose drawing.

(E)  Pakhi is the only child to choose ceramics.

10. If exactly one of the children chooses origami, then which one of the following must be true?

(A)  Karly chooses ceramics.

(B)  Sharon chooses drawing.

(C)  Barry chooses ceramics.

(D)  Pakhi chooses drawing.

(E)  Ezra chooses ceramics.

11. If Sharon chooses the same activity as Barry does, then which of the following could be true?

(A)  Ezra chooses drawing.

(B)  Pakhi chooses origami.

(C)  Barry chooses ceramics.

(D)  Usef chooses ceramics.

(E)  Sharon chooses drawing.

12. Each of the following could be a complete and accurate list of the children who choose ceramics EXCEPT:

(A)  Barry, Karly

(B)  Karly, Pakhi

(C)  Ezra, Pakhi

(D)  Barry, Ezra

(E)  Sharon, Usef

13. Which one of the following must be true?

(A)  Ezra chooses a different activity than Sharon does.

(B)  Ezra, Jaime, and Sharon do not all choose the same activity.

(C)  Ezra chooses a different activity than Jaime does.

(D)  Barry, Jaime, and Karly do not all choose the same activity.

(E)  Barry chooses a different activity than Ezra does.

Questions 14–20

A scientist must create two test groups—group 1 and group 2—from six of seven lab rats—three brown rats named Abby, Carl, and Dennis; and four white rats named Elle, Fern, Horn, and Iris. Each group will have three rats. No rat can be in more than one group. Each group must have a least one brown rat and one white rat. The composition of the groups must conform to the following conditions:

Neither group includes both Abby and Iris.

Neither group includes both Elle and Horn.

If a group includes Dennis, it includes neither Horn nor Iris.

If group 1 includes Carl, group 2 includes Horn.

14. Which of the following could be the makeup of the two groups?

(A)  Group 1: Abby, Fern, Horn

Group 2: Carl, Elle, Iris

(B)  Group 1: Carl, Dennis, Iris

Group 2: Abby, Elle, Fern

(C)  Group 1: Carl, Dennis, Elle

Group 2: Fern, Horn, Iris

(D)  Group 1: Abby, Elle, Iris

Group 2: Carl, Fern, Horn

(E)  Group 1: Abby, Dennis, Fern

Group 2: Carl, Elle, Horn

15. If Carl is in group 1, which one of the following pairs must be in group 2 together?

(A)  Fern and Iris

(B)  Abby and Horn

(C)  Horn and Iris

(D)  Elle and Fern

(E)  Dennis and Fern

16. If Elle is in group 1, which one of the following pair of rats could be in group 1 together with Elle?

(A)  Fern and Iris

(B)  Carl and Fern

(C)  Carl and Horn

(D)  Abby and Carl

(E)  Carl and Iris

17. If Abby is in the same group as Dennis, which one of the following could be true?

(A)  Both Elle and Fern are in group 2.

(B)  Horn is in group 1.

(C)  Carl is in group 1.

(D)  Both Elle and Fern are in group 1.

(E)  Iris is in group 2.

18. Each of the following pairs of rats could be in group 2 together EXCEPT:

(A)  Carl and Horn

(B)  Abby and Horn

(C)  Abby and Carl

(D)  Abby and Dennis

(E)  Carl and Dennis

19. Which one of the following could be true?

(A)  Carl is in group 1 and Elle is in group 2.

(B)  Abby is not in any group and Carl is in group 1.

(C)  Abby is in group 1 and Elle is in group 2.

(D)  Abby is in group 2 and Dennis is in group 2.

(E)  Abby is in group 2 and Carl is not in any group.

20. If Dennis is in group 2, which one of the following must also be in group 2?

(A)  Horn

(B)  Fern

(C)  Iris

(D)  Carl

(E)  Abby

Questions 21–26

Of eight cargoes—L, N, O, P, R, S, T, and W—only six can be loaded on the three available trucks—Truck 1, Truck 2, and Truck 3. Each cargo must be loaded into one of two holds—front and back. The loads must be loaded in order, front first, then back, under the following conditions:

L can only go in Truck 2.

T and W cannot go in the back hold of a truck.

If S is loaded on a truck, then N and O go on the next truck, unless S is loaded on Truck 3.

21. Which one of the following could be the loading register for the cargo?

(A)  Truck 1: Front, S; Back, N

Truck 2: Front, W; Back, R

Truck 3: Front, T; Back, R

(B)  Truck 1: Front, N; Back, W

Truck 2: Front, T; Back, O

Truck 3: Front, R; Back, P

(C)  Truck 1: Front, T; Back, S

Truck 2: Front, O; Back, N

Truck 3: Front, P; Back, L

(D)  Truck 1: Front, R; Back, N

Truck 2: Front, L; Back, P

Truck 3: Front, W; Back, O

(E) Truck 1: Front, O; Back, T

Truck 2: Front, N; Back, P

Truck 3: Front, S; Back, R

22. If P and R are not loaded, then the front sections of Truck 1, Truck 2, and Truck 3 could carry the following cargo:

(A)  T, W, and O

(B)  O, W, and N

(C)  W, L, and N

(D)  N, L, and S

(E)  S, N, and T

23. Which one of the following is a pair of cargoes, if loaded on the same truck, that must go on Truck 3?

(A)  O and W

(B)  L and R

(C)  N and S

(D)  T and P

(E)  P and S

24. If L, S, and W are loaded in different trucks from each other, which one of the following could be true?

(A)  T is on Truck 1.

(B)  S is on Truck 2.

(C)  N is on Truck 3.

(D)  W is on Truck 3.

(E)  S is on Truck 1.

25. If N, P, and R, not necessarily in that order, are loaded in the fronts of the three trucks, which of the following must be true.

(A)  P is on Truck 3.

(B)  O is on Truck 3.

(C)  P is on Truck 2.

(D)  N is on Truck 1.

(E)  O is on Truck 1.

26. If P is on the back of Truck 2, and N is on the back of Truck 3, which one of the following could be the list of cargoes in the fronts of Truck 1, Truck 2, and Truck 3, respectively?

(A)  W, O, and R

(B)  W, L, and O

(C)  R, L, and O

(D)  O, R, and S

(E)  S, O, and R

STOP

IF YOU FINISH BEFORE TIME RUNS OUT, CHECK YOUR WORK ON THIS
SECTION ONLY. DO NOT GO ON TO ANY OTHER TEST SECTION.

SECTION IV

Time—35 minutes

25 Questions

Directions: Each passage in this section is followed by a group of questions to be answered on the basis of what is stated or implied in the passage. For some of the questions, more than one of the choices could conceivably answer the question. However, you are to choose the best answer; that is, the response that most accurately and completely answers the question, and blacken the corresponding space on your answer sheet.

Images

Images

Images

Images

1. The primary purpose of the passage is to provide an answer to which one of the following questions?

(A)  How has the Supreme Court failed to protect the “one person, one vote” precedent and how have voters taken action to correct its failure?

(B)  What mistakes are states making by trying to solve a national problem on a state level and how will their solutions hinder democracy and create unfairness in congressional elections?

(C)  What are California and Florida’s unique electoral redistricting problems and what efforts have voters taken to solve them?

(D)  What legislation have voters in two states approved as solutions to a perceived political problem and what major opposing arguments might be used in future such efforts as exemplified by a particular case?

(E)  How have reform-minded citizens misinterpreted the legislative apportionment problem and enacted costly and unwise solutions?

2. The passage implies that before 1964, which of the following may have been true?

(A)  Political parties had no say or control in legislative apportionment.

(B)  Some citizens’ votes counted for less than a full vote.

(C)  A district could be designed to contain more constituents than another.

(D)  Gerrymandering was allowed for some states and not others.

(E)  The Supreme Court could overrule any redistricting based on the “one person, one vote” requirement.

3. The discussion of the Supreme Court Wesberry v. Sanders case in lines 18–26 is intended primarily to

(A)  show that the Supreme Court failed to solve the “one person, one vote” problem that has plagued the electoral system since the early beginnings of the nation

(B)  explain the legal underpinnings of the proposed solutions to the legislative apportionment issue

(C)  explain that while one branch of the nation’s government was trying to solve the problem another branch was undermining that effort

(D)  give a historical perspective to a problem with democratic representation that has yet to be resolved through legislative reapportionment

(E)  establish that the Supreme Court is in fact the original cause of gerrymandering and other interference by political parties in the electoral mapping process

4. The discussion of Proposition 27 in lines 53–59 implies that which of the following was true before the passage of Proposition 20?

(A)  The Citizens Redistricting Commission, though a good idea, was an impracticable institution and needed to be eliminated for redistricting to work properly.

(B)  The politicians wanted to create confusion by having both propositions on the same ballot, hoping that voters would favor Proposition 27 over Proposition 20.

(C)  The Citizens Redistricting Commission existed before the election to redistrict the state legislative map but lacked the authority on the congressional map.

(D)  Voters rejected Proposition 27 because it conflicted with Proposition 20 but would have approved it if a compromise had been struck.

(E)  The Citizens Redistricting Commission was unconstitutional and if the voters did not eliminate it the courts would have to do it.

5. Which of the following, if true, would most undermine the arguments for California’s Proposition 20 discussed in the third paragraph?

(A)  It is proven that a significant majority of voters reelect their representatives because they are happy with their performance.

(B)  Several bills were passed recently because both Republican and Democratic congressmen were able to come together and achieve compromises on the legislation.

(C)  The proposition would deny the commission the right to use income, race, or gender as factors in determining the shape and size of the district.

(D)  It is discovered that one of the members under consideration for the commission is actually a member of a political party.

(E)  Population movements among districts are so fluid from year to year that it is almost impossible to predict voting patterns for a particular district in any election.

6. Which of the following accurately expresses the meaning of the word “entrenched” as it is used in line 38 of the passage?

(A)  Unelectable

(B)  Deep-seated

(C)  Unshakeable

(D)  Stubborn

(E)  Inexorable

PASSAGE A

Images

Images

PASSAGE B

Images

Images

7. Which of the following best describes the relationship between passage A and passage B?

(A)  Passage A and passage B both present similar solutions to the same problem.

(B)  Passage A presents a problem and passage B presents a solution to that problem.

(C)  Passage B explicitly contradicts a set of implicit assumptions made by the author of passage A.

(D)  Passage A and passage B both describe similar movements arising from the same circumstances.

(E)  Passage B discusses a refinement of an idea that passage A presents as a broad outline.

8. Which of the following can be inferred from passage A about theater in the early twentieth century?

(A)  The advent of agit-prop theatre was a wholly new concept of political theatre intended to provoke its audience to take action.

(B)  The political ideas advocated by plays such as the push for organized labor was the result of people like Lee Strasburg importing ideas from Russians like Stanislavski.

(C)  Broadway was more likely to feature a musical or lighthearted comedy in 1920 than a tragic play portraying a struggling family.

(D)  Audiences lost interest in “serious drama” after the end of the Depression.

(E)  Playwrights and actors resented the political nature of the plays and, when the Depression ended, forced the theater managers to go back to the pre-Depression types of theater.

9. Each of the following is mentioned as being a point of difference between the Group Theatre and the Shock Troupe EXCEPT

(A)  the use of untrained actors

(B)  the depiction of societal ills

(C)  the physical location in which the play is staged

(D)  the names of the characters in the plays

(E)  the avocation of societal change

10. According to passage B, the Shock Troupe would least likely perform a play that includes which the following:

(A)  a staged argument between “Freed-Slave Man” and “White Slave Owner” about the lingering effects of slavery and racism in America

(B)  a public re-enactment of the workers walking off the production line at a local car manufacturing plant in protest of poor working conditions

(C)  a dramatic argument between Jack, an unemployed ironworker, and his wife, Mary, about her long hours working as an assistant to a corporate executive, who Jack believes has romantic intentions toward her

(D)  an unemployed man sells his last belongings to a bartender in exchange for a place to sleep in a storeroom and then giving a speech about how the American free market system stripped him of his dignity

(E)  an impromptu performance before a miners’ union meeting that through dramatic effect calls on workers to rise up and speak out about low wages and dangerous working conditions

11. Which of the following would the author of passage B most likely believe about the Group Theatre as it is described in passage A?

(A)  The Group Theatre’s use of trained professional actors allowed it to better to connect with audiences.

(B)  The Group Theatre was better known than the Shock Troupe with theater audiences of the 1930s.

(C)  Actors working with the Shock Troupe were less idealistic than the actors associated with the Group Theatre.

(D)  The Group Theatre’s production of Waiting for Lefty was ultimately more popular with contemporary audiences than the Shock Troupe’s production of Newsboy.

(E)  “The Method,” as employed by actors in the Group Theatre, is the most effective means of teaching acting.

12. Which of the following is the meaning of the term “agit-prop” as implied by the context of both passages?

(A)  Agriculture-Proposition

(B)  Agitation-Proposal

(C)  Agitation-Propaganda

(D)  Agitation-Propagation

(E)  Agitation-Proper

Images

Images

Images

Images

13. Which one of the following best states the main point of the passage?

(A)  People are happier under a universal welfare state than under a partial welfare state and thus the United States as a partial welfare state will have difficulty making its people happy.

(B)  People are happier under a universal welfare state but happiness is a poor basis for judging a society and thus the recent studies on the relationship between the welfare state and happiness are flawed.

(C)  While the United States is engaged in a battle between those for or against welfare programs, the rest of the world has forged ahead at establishing welfare states that have achieved a high level of societal happiness.

(D)  While politicians, social scientists, and economists argue about the welfare state using arguments unsupported by data, Rothstein, Pacek, and Radcliff offer unique support for it through analysis of well-researched data.

(E)  Happiness as a basis for analyzing the success of a political state is valid but the SWB measurement is flawed. Only through improved data collection and a more accurate happiness index can study of the welfare state be accepted by the thinkers of today.

14. Which of the following best describes the organization of the passage?

(A)  A social issue is discussed. An alternative method of viewing that issue is discussed. The alternative method is assessed. The social issue is discussed in relation to the alternative method.

(B)  A social problem is presented. An approach to solving the problem is discussed. The approach is evaluated. The approach is discussed in relation to one specific nation that is experiencing the social problem.

(C)  Alternative views of a social problem are presented. An approach to resolving the problem is presented. A critique is presented that discredits that approach. A particular case study is given to show that the approach is not feasible.

(D)  A social problem is analyzed. A solution to the problem is proposed. The strengths and weaknesses of the solution are discussed. The practical difficulties of delivering the solution are presented.

(E)  A social problem is discussed. A unique approach to the problem is discussed. The validity of the approach’s underlying assumption is discussed. A particular case is discussed in relation to the approach.

15. The author of the passage refers to Rothstein’s bias in line 32 because

(A)  Rothstein is a well-known and outspoken advocate for social welfare programs

(B)  Rothstein was an unnamed author of the study with Pacek and Radcliff and thus has a vested interest in bringing attention to their ideas

(C)  Rothstein lives in a northern European country that happens to be a long-standing universal welfare state

(D)  Rothstein invented the SWB measurement and wants it to be used in future studies

(E)  Rothstein is currently running for political office in his country and retaining the welfare state is his central reason for running

16. Which of the following statements most undermines Rothstein’s argument regarding the “arbiter” in lines 48 to 52?

(A)  Political scientists and public policy experts experience happiness and unhappiness due to social welfare programs just as much as anyone, so their criteria are not much different from those of the widespread public.

(B)  Happiness or unhappiness is influenced by factors outside the political spectrum such as the availability of companionship, community support, and social opportunities, while the experts filter out those issues and focus on the specific problems.

(C)  Political scientists and public policy experts tend to see problems from a more theoretical perspective and don’t see the very basic elements that might affect whether a social welfare state is successful or not.

(D)  The widespread public tends to be swayed by waves of economic ups and downs that affect happiness while the political scientists and public policy experts are all wealthy individuals who are unaffected by economic changes.

(E)  The SWB measurement has been found to be very accurate in predicting the success of welfare programs, while simulation models created by political scientists and public policy experts have rarely been successful at making such predictions.

17. Based on the first paragraph of the passage, which of the following, if true, would support an argument made by advocates for a more universal welfare program in the United States?

(A)  The results of a study show that unemployment benefits reduce the number of applicants for jobs.

(B)  A survey of welfare recipients shows that a large majority save the money and do not spend it.

(C)  Research determines that the number of registered voters increased significantly during the 10 years after certain welfare programs were instituted.

(D)  A survey of welfare program participants revealed that a large percentage expressed substantial resentment toward the government.

(E)  The results of a study show that a large portion of the population is happier due to the existence of welfare programs.

18. Which of the following best describes the author’s attitude toward Rothstein’s approach to happiness and the welfare state?

(A)  cautious neutrality

(B)  strong condemnation

(C)  moderate advocacy

(D)  moderate skepticism

(E)  grudging acceptance

19. Which of these claims would Rothstein be least likely to accept?

(A)  The ability of its people to retain family ties and establish new ones is a factor that might cloud the relationship between SWB and the establishment of a universal welfare state.

(B)  The addition of a program to protect children from the effects of poverty will make a nation’s people happier.

(C)  Northern European states have a higher SWB than the United States and other states with nonuniversal welfare programs.

(D)  A long-standing program that all people of the nation have come to depend on and know will be there when they need it will increase the SWB for that nation.

(E)  The discovery that a large portion of welfare program funds was misappropriated to a politician’s pet project proves that the welfare state is a failure.

Images

Images

Images

Images

20. Which one of the following most accurately expresses the main idea of the passage?

(A)  Corporations have found a way to integrate philanthropy into their operations and sales to avoid the stigma associated with cash donations, which are seen as cheap marketing tactics. Why they participate in philanthropy is a more complicated question.

(B)  Corporations are increasingly integrating philanthropy into their operations, but the financial effect on customers, shareholders, and employees is no different from a corporation making a cash donation to a charity. Why corporations do it is a more complicated question.

(C)  Regardless of how corporations’ actions are perceived, as an integral part of society corporations have a moral obligation to not only make charitable donations but also to integrate philanthropic values into their operations.

(D)  Integrating philanthropic activities into the company’s operations is the equivalent of extortion, forcing customers to participate in an unwanted philanthropic effort. Companies would do better to just make a charitable donation. Why they participate in philanthropy is a more complicated question.

(E)  Corporations are increasing their commitment to philanthropy. As a result, they find themselves in more complicated relationships with their customers, shareholders, and employees. Why corporations participate in philanthropy in the first place is a more complicated question.

21. Which one of the following most accurately describes the author’s attitude toward the corporate philanthropy model discussed in lines 8–13?

(A)  confident that it offers corporations a new and better way to be philanthropic without negatively affecting profits and shareholder value

(B)  certain that it will give customers more power over their relationship to the philanthropic transaction implied by the purchase of the company’s products

(C)  convinced that it offers little new in the way of a financial equation or explanation for corporate philanthropic efforts

(D)  satisfied that it comes up short at doing much better than individuals could if they had control of the funds directed to philanthropic causes

(E)  pleased that corporations have found a more creative way to commit financial resources to philanthropic causes than just cutting a check

22. Which one of the following sentences would most logically begin a paragraph immediately following the end of the passage?

(A)  Logically, we must move on to discuss how a company that is committed to philanthropy can truly know that its suppliers are abiding by the fair trade rules and whether its production processes actually create more waste rather than eliminate it.

(B)  Thus, the question becomes, are the billions of dollars sent from companies toward public good worth it, and if so, which philanthropic model is more effective at allowing a company to maximize the benefit from that contribution?

(C)  It is impossible to know why a corporation engages in philanthropy because corporations are more complex than individuals. Instead, the question to ask is whether the government should stop corporations from participating in this unprofitable activity and leave philanthropy to individuals.

(D)  Therefore, corporations should be limited to the philanthropic model that involves donating cash and not the one that hides the donation within the cost of goods sold and results in inflating the price the consumer pays.

(E)  In an effort to understand why corporations engage in philanthropy, we must survey the guidelines for morals and ethics established by corporations as guiding principles for employees. After all, a corporation is just the sum of its employees.

23. The relationship of the information contained in the two sentences at lines 35–42 to that in the sentence at lines 8–13 can most accurately be described as

(A)  no significant relationship because they represent two unrelated factual statements

(B)  the author’s opinion agreeing with another opinion reported by the author in the earlier lines

(C)  a hypothetical situation clarifying a statement reported by the author in the earlier lines

(D)  agreement in general with the earlier position but disagreement over the particulars

(E)  essentially equivalent assertions with the latter being an explicit clarification of the earlier lines

24. It can be inferred from the passage that the author holds that a corporation that engages in philanthropy should

(A)  inform the customer that all funds are tax deductible

(B)  allow customers to opt out of paying the portion of the price that goes toward philanthropic causes

(C)  inform its customers, employees, and shareholders that price, profit, and shareholder value will be affected by its philanthropy

(D)  educate the public on the corporate misuse of philanthropic funds

(E)  establish a moral code and publish it for its customers, employees, and shareholders to see

25. The primary purpose of the passage is to

(A)  question the reasons for a corporate behavior, give countering reasons for that behavior to cease, and synthesize the opposing views by posing another question

(B)  argue that a particular corporate behavior has no moral and ethical justification within society and should be stopped

(C)  discuss the nature of a corporate behavior and pose a moral reason that undercuts its justification

(D)  explain a trend in corporate behavior, compare it to previous similar behavior, and pose a question directed at the foundation of that behavior

(E)  reveal how corporations manipulate the relationship with their customers, employees, and shareholders to justify an unethical behavior

STOP

IF YOU FINISH BEFORE TIME RUNS OUT, CHECK YOUR WORK ON THIS
SECTION ONLY. DO NOT WORK ON ANY OTHER TEST SECTION.

LSAT Diagnostic Test Answer Key

Section I

1. D

2. D

3. A

4. D

5. B

6. E

7. B

8. C

9. C

10. E

11. E

12. B

13. A

14. B

15. C

16. B

17. C

18. D

19. A

20. D

21. A

22. B

23. D

24. D

25. E

26. A

Section II

1. D

2. C

3. A

4. E

5. C

6. A

7. D

8. C

9. D

10. D

11. D

12. B

13. D

14. D

15. A

16. B

17. A

18. E

19. A

20. C

21. B

22. C

23. C

24. D

Section III

1. D

2. A

3. C

4. C

5. E

6. B

7. B

8. A

9. E

10. B

11. A

12. E

13. D

14. A

15. B

16. E

17. E

18. D

19. C

20. D

21. D

22. A

23. C

24. C

25. E

26. B

Section IV

1. D

2. C

3. D

4. C

5. E

6. B

7. D

8. C

9. B

10. C

11. A

12. C

13. B

14. E

15. C

16. B

17. C

18. D

19. E

20. B

21. C

22. B

23. E

24. C

25. D

Calculate Your Score

 Complete the following table.

Your Raw Score

Images

Your Approximate Scaled Score

It is impossible to say with complete precision what raw score will translate to what scaled score on future LSATs, but here is a rough estimation.

Images

LSAT Diagnostic Test Answers and Explanations

SECTION I

1. Answer: D

STEP 1:  Read the question and identify your task.

This is a Flaw question. You are looking for a flaw in the argument or something illogical in the argument that calls into question its conclusion.

STEP 2:  Read the argument with your task in mind.

The argument makes an unqualified leap in logic. Instead of attributing the civic disorder to David Ellington, it says that the city council intended to undermine civic order by hiring the incompetent Ellington.

STEP 3:  Know what you’re looking for.

The correct answer will point out that flawed logic.

STEP 4:  Read every word of every answer choice.

When you read each answer closely, certain words help you eliminate answers. Answer A does not resemble what you are looking for at all. It focuses on “quantitative” results versus “qualitative” results, but the argument does not include any quantitative results, so it cannot be the correct answer. Answer B also does not work. Yes, the conclusion is fairly general, but the descriptor “anomalous” is inaccurate. Answer C describes Ellington’s incompetence as “perceived,” but specific results are discussed, and it does not address the argument’s conclusion. Answer D says the flaw is the assumption that an action leads to a result (hiring of Ellington leads to civic disorder), and that the action was taken to bring about that result (hiring Ellington was intended to bring about the civic disorder). This fits your understanding of the flaw perfectly and would seem to be the right answer. Before you make a decision, you must finish reviewing all the answers. Answer E says the flaw is a “restatement,” but no part of the argument serves as a restatement, so this cannot be the correct answer. The correct choice is answer D..

2. Answer: D

STEP 1:  Read the question and identify your task.

This is a Parallel question. The question asks that you match the reasoning in the answer to the one in the statement, so you are looking for a similar pattern of thinking between the two.

STEP 2:  Read the argument with your task in mind.

The argument tells of a maintenance man who makes an exception based on a judgment that a police officer, due to his special skills, can protect himself.

STEP 3:  Know what you’re looking for.

The correct answer will include a judgment and an exception to that judgment that matches the kind made in the argument.

STEP 4:  Read every word of every answer choice.

In answer A the deliveryman makes a judgment to organize his deliveries, but he makes no exceptions, so this does not resemble your pattern of thinking and is not the correct answer. Answers B and C both fail your test because both the candidate and jury make a judgment regarding certain areas or one person to the exclusion of all others, not the other way around as your maintenance man does. In answer D the captain of the yacht judges that the US naval officer has special skills that enable him or her to handle an emergency situation, so the captain excludes the officer from the training. This resembles your required pattern of thinking and is most likely the correct answer, but you must complete your review of the answers. Answer E says the manager makes no exceptions at all, forcing everyone to participate, and this does not fit your pattern, sothe correct choice is answer D.

3. Answer: A

STEP 1:  Read the question and identify your task.

This is a Describe question. It asks you to describe the error in both Larry and Carrie’s reasoning.

STEP 2:  Read the argument with your task in mind.

Larry and Carrie discuss the relationship between positive results and how this quality determines whether the law is bad or good. Larry and Carrie incorrectly assume that because a law has positive results it is a good law, and therefore, because a law shows positive results, such evidence is sufficient to consider it a good law.

STEP 3:  Know what you’re looking for.

The correct answer will identify the shared weakness in the exchange between Larry and Carrie. Pay close attention to the words in the argument—what is considered good or bad by the citizenry versus what is good or bad. In order for both their arguments to hold up, they have to presuppose that a law is good if it has positive results and vice versa.

STEP 4:  Read every word of every answer choice.

Answer A describes Larry and Carrie’s assumption almost exactly, that a law having a certain quality (positive results) is necessary for it to be a particular type (good or bad) of law and that having that quality is sufficient for being that type of law. But, let’s review the remainder of the answers to make sure, paying close attention to specific words. The quality is a determinant of the type of law, not a “shared” attribute and they never say the quality is the “only” quality distinguishing between the two types of laws, so answer B is incorrect. Their reasoning includes no extrapolation from “most” laws to “all” laws, so answer C is incorrect. Answer D is incorrect because Larry and Carrie make no comparison between a “particular nation” and “all nations.” Finally, answer E is incorrect because they do not extrapolate one distinguishing quality to mean anything with regard to other qualities of the laws. The correct choice is answer A.

4. Answer: D

STEP 1:  Read the question and identify your task.

This is a Weaken question. It tells you that you must pay close attention to the bases for the argument and discover how a counterargument might weaken it.

STEP 2:  Read the argument with your task in mind.

This is an anthropological argument saying that there is a new belief that the “white revolution” occurred due to one society conquering another in a “violent and transformative” manner, while it was previously thought to be a more peaceful process.

STEP 3:  Know what you’re looking for.

A weakening statement would contradict or discredit this argument, proving that in fact it was as previously thought, not violent but a peaceful transition to a milk-based society.

STEP 4:  Read every word of every answer choice.

Answer A talks specifically of northern Europe and not the wider continent. Also, milk may have still been part of the Middle Eastern diet even if someone else raised the cows. But more importantly, the transition in Europe still could have been violent. For answer B, just because the Europeans drank milk before the Middle Easterners arrived does not mean that their society did not go through the later violent conversion mentioned in the argument, so this answer fails to weaken the argument. Answer C actually strengthens the argument, proving that violence occurred at the time, and you are looking for a statement that weakens the argument. Answer D says that the Middle Easterners had settlements next to the hunter-gatherers, and they engaged in peaceful trade. Such coexistence and peaceful trade definitely weakens the argument that the transition was violent and transformative. And finally, the evidence of “sexual intermingling” mentioned in answer E does not rule out the possibility that the hunter-gatherer society went through the cataclysmic conversion, so the correct choice is answer D.

5. Answer: B

STEP 1:  Read the question and identify your task.

This is a Weaken question. You are looking for a statement that weakens the argument, so you read the argument looking for critical facts or bases that make the argument work.

STEP 2:  Read the argument with your task in mind.

The argument bases its conclusion on a trend over six years during which there was a rise in young people entering the workforce at the same time there was a rise in percentage of young people graduating from high school. It uses this basis to state that increasing graduates will increase employment of young people.

STEP 3:  Know what you’re looking for.

A weakening argument might show a caveat or alternate interpretation that will alter that logical step. The correct answer will satisfy that caveat or alternate interpretation.

STEP 4:  Read every word of every answer choice.

Answer A discusses only completion of years in college, while the argument discusses graduation and dropout rates. Since it is dealing with different terms altogether, it cannot be the right answer. Answer B says there was also a rise in percentage of high school dropouts hired between 2000 and 2006, indicating that the rise in employment benefited all young people, graduates and non-graduates. This weakens the correlation between graduation and employment and thus weakens the reverse correlation between employment and graduation. This would seem to weaken the argument considerably, but let’s continue. Answer C speaks to the jobs and their difficulty, but says nothing about the relationship between graduate or nongraduate status and employment. Also, it is possible that the jobs are too complicated for graduates as well. You would not know because the statement doesn’t say anything about that. Answer D tries to confuse you by using the term “number” versus “percentage,” and even though a larger number of dropouts were hired, there might still be a larger percentage of graduates who were hired, so this cannot be your weakening argument. The argument discusses percentages in its premises, and the weakening statement must do so as well. Answer E may be a common practice, but it has nothing to do with actual results of who was hired or not. The correct choice is answer B.

6. Answer: E

STEP 1:  Read the question and identify your task.

This is a Weaken question. You need to find a statement that weakens the speaker’s statement.

STEP 2:  Read the argument with your task in mind.

The speaker talks about how corporations can use public relations to gloss over any malfeasance on their part and ultimately get away with anything as long as they wait out the “short attention span of the public.”

STEP 3:  Know what you’re looking for.

You seek in your answer a statement that contradicts or undermines this idea.

STEP 4:  Read every word of every answer choice.

Answer A may be interpreted as strengthening the argument, saying that a company can become too big to fail despite any socially irresponsible actions. Answer B essentially restates the argument in simpler terms, and it speaks in past tense and does not address what the case might be in the future. Answer C also restates the argument, but with the caveat of faster and slower growth among the corporations. Answer D adds government institutions to the argument, which does not weaken it. Answer E says that public relations are often seen through by the public and fail to mask a corporation’s irresponsible actions. While this may not address the short attention span part of the argument, it does weaken the argument and qualifies it much better than the other answers. The correct choice is answer E.

7. Answer: B

STEP 1:  Read the question and identify your task.

This is a variation of a Describe question—a Describe How the Argument Proceeds question. It asks you to find the answer that describes how the argument “proceeds,” that is, how it comes to its conclusion.

STEP 2:  Read the argument with your task in mind.

Based on the experience of “a number” of amateur photographers with a particular camera, the argument urges “any” ( = all) such photographers to choose that camera.

STEP 3:  Know what you’re looking for.

The correct answer will describe how the argument applied the assertions of a small group (the amateur photographers with the Apheron camera) to make a suggestion to a larger group (all amateur photographers).

STEP 4:  Read every word of every answer choice.

Answer A talks about testing conditions, while the argument makes no mention of testing conditions, so it is off the mark. Answer B says that the argument uses a subset to make a recommendation for a larger group, and this meets your criteria. The amateur photographers are the subset and the “any” photographers are your larger group. With answer C the credibility of the amateur photographers is accepted as a given and is not part of the argument. For answer D, the motivations for buying the camera are never discussed in the argument. The experience of the smaller group of photographers is not given any wider context. It is only used to recommend the camera to a larger group, so answer E is incorrect. The correct choice is answer B.

8. Answer: C

STEP 1:  Read the question and identify your task.

This is a Weaken question. It asks you to find an answer that weakens the argument.

STEP 2:  Read the argument with your task in mind.

Look back at the basis for the camera recommendation, in this case that the camera offers finer detail despite its having a lower resolution than the other competing Norwich cameras.

STEP 3:  Know what you’re looking for.

The correct answer will cast doubt on the relationship between the premises and the conclusion.

STEP 4:  Read every word of every answer choice.

With answer A the makeup of the lens does not change the results discussed in the argument, so this does not weaken the basis for the recommendation. Answer B is not much different from answer A, except with regard to sensor size instead of lens substance. For answer C, the criterion under which the argument makes its recommendation is the finer detail the camera is able to achieve in both bright and low light situations. If other criteria exist for amateur photographers when considering a camera, then the argument is most definitely weakened and this would seem to be your correct answer. Answer D describes conditions under which the testing results seem more credible rather than weaker. Answer E may weaken another argument, one that argues against digital cameras, but it does not weaken the recommendation that if photographers do consider a digital camera, they should consider the Apheron over others. The correct choice is answer C.

9. Answer: C

STEP 1:  Read the question and identify your task.

This is a Describe question. It asks you to describe how a certain fact is being used within the argument.

STEP 2:  Read the argument with your task in mind.

When you read through the argument, you pay close attention to this fact and how it functions. In this case, that the auto companies are purchasing more equipment and spending more on R&D seems to be a justification or evidence supporting the conclusion of the argument. This argument actually has the conclusion in the first sentence.

STEP 3:  Know what you’re looking for.

The correct answer discusses the fact as a justification or evidence.

STEP 4:  Read every word of every answer choice.

Answer A states that the fact is an inference from a premise when, in fact, it is the exact opposite. That companies expect sales to increase is the inference from the fact under consideration. Answer B is incorrect for the same reason; your fact is not an inference but a premise or basis for inference. Answer C states that the fact is the primary evidence for the conclusion, and this seems like your answer because the economist concludes that industry is on the way to recovery based on the fact under consideration, as you formulated from reading the question and the argument. Answer D states that the fact is required for the conclusion to be true, but the conclusion may be true based on other facts that you do not have on hand, so this is not the function of the facts under consideration. Finally, the argument does not explain why the companies are spending more. It merely makes a judgment based on that fact, so answer E cannot be correct. The correct choice is answer C.

10. Answer: E

STEP 1:  Read the question and identify your task.

This is a Parallel question. It asks you to find an argument that follows the same logical pattern.

STEP 2:  Read the argument with your task in mind.

The given argument says that because there is a directly proportional relationship between a higher use of water and a higher heat index (increase leads to increase), then because the average heat index is four points higher, more water will be used.

STEP 3:  Know what you’re looking for.

You must look for a similar relationship in the answers.

STEP 4:  Read every word of every answer choice.

Answer A creates two proportionalities (doctors to services, doctors to patients) in relation to the first statement (doctors). Then it tries to create a relationship between the two conditionals (services to patients). This is a much more complicated relationship than you are looking for and thus not your answer. Answer B starts out with the right kind of proportionality between two terms (doctors and nurses), but then it adds an unrelated statement about orderlies and draws an unsupportable relationship between the doctors and orderlies, so it is also not your answer. Answer C starts out well with a directly proportional relationship, higher bill to higher number of medical professionals seen, but the conclusion is based on the number of patients and not the number of medical professionals the patients see during their visit, which is a break from the premise. Thus answer C cannot be your answer. Answer D makes a recommendation based on the premise but does not draw a conclusion. Answer E describes a directly proportional relationship. More analgesics are prescribed when there are more patients, and since the number of patients this year is up 15 percent, then the number of analgesics must be up as well. Thus, the correct choice is answer E.

11. Answer: E

STEP 1:  Read the question and identify your task.

This is a Describe question. The question asks that you describe what argumentative method Zola uses to respond to Jason.

STEP 2:  Read the argument with your task in mind.

Reading their arguments, you discover that Zola argues that the long cooking time and the ample sunlight necessary to operate the solar cooker are problems, meaning that the solar cooker is not such a perfect answer to “a host of problems in developing nations.” She sees the arguments on which the conclusion is based as sound and logical, but she feels the conclusion doesn’t take other factors into account.

STEP 3:  Know what you’re looking for.

You will look for something similar in your answer options.

STEP 4:  Read every word of every answer choice.

Zola actually agrees with Jason’s arguments and does not believe them to be “fallacious,” so answer A cannot be correct. While Zola disagrees with Jason’s conclusion, her problem is specific to the cooker, not his insufficient definition of the problems in developing nations, so answer B cannot be correct. Answer C has the same problem as answer A. She does not disagree with his arguments or their basis. With answer D, Zola is not reinterpreting anything, so this answer cannot be correct. Zola gives caveats that weaken several of Jason’s assumptions, and this is what you find in answer E. Thus, the correct choice is answer E.

12. Answer: B

STEP 1:  Read the question and identify your task.

This is a Parallel question. It asks you to look for a set of statements that follow a similar logical pattern to the one given in the passage.

STEP 2:  Read the argument with your task in mind.

In this case, the argument states that a lumberyard sells only two types of lumber, pine and oak, and Joe never uses pine to build tables. Thus, when he builds a table for Marion, it must be made of oak. The key word never restricts Joe’s choice.

STEP 3:  Know what you’re looking for.

You must look for a similar pattern in the answers.

STEP 4:  Read every word of every answer choice.

In answer A there is no definitive “only” statement that restricts Rocky’s choice and determines the conclusion. He just makes a judgment call as to which car works for him. In answer B you see the right pattern. The Roddick Corporation offers Pedzisai only two choices, Sweden and South Africa. Pedzisai would never choose South Africa if given a choice between the two. Thus, when choosing an assignment, he will choose Sweden. The word never is used in almost exactly the same way, as a restricting factor and one that determines the concluding statement. Answer C actually sets up conditions under which Georgia will choose neither of the options at her disposal. Answer D has a conclusion that has nothing to do with the choice between the break and a longer lunch. Rather, it has to do with a third element, whether to get a haircut or not. The pattern is not the same. In answer E, the owner has equally bad choices and the conclusion gives no indication of what choice is made. Rather, it just describes the conditions regardless of what choice the owner makes, unless of course a third alternative presents itself. The correct choice is answer B.

13. Answer: A

STEP 1:  Read the question and identify your task.

This is a Parallel question. This question is asking you to find the answer whose logic most resembles the given statements. That the reasoning is “questionable” is irrelevant to your task of just finding the answer that follows the same logic.

STEP 2:  Read the argument with your task in mind.

In this case, a building inspector has been accused of a crime, the evidence has been lost or wiped out by an action, and thus, the argument recommends that the accusation be retracted.

STEP 3:  Know what you’re looking for.

Again, that the reasoning is “questionable” is irrelevant to your task of just finding the answer that follows the same logic, questionable or not.

STEP 4:  Read every word of every answer choice.

Answer A switches the statements around, giving the recommendation in the second statement, but the logic is essentially the same. The computer scientist has been accused of a crime and the accusation should be dismissed because all evidence of his crime (hard drives, records) has been eliminated. Answer B recommends that the teacher be further investigated and the evidence is still available, so this situation is very different from your inspector’s. Answer C is close but still not the best answer because the lack of evidence described is insufficient. People still may have heard the speech and be familiar with its content. Answer D argues to sustain the accusation that is supportable by some evidence, which is a completely different situation. Answer E argues that the accusation be retracted based on the lack of identity of the accuser rather than any insufficiency of evidence against the financial manager. The correct choice is answer A.

14. Answer: B

STEP 1:  Read the question and identify your task.

This is an Assumption question. The question asks which answer makes the logic of the argument successful.

STEP 2:  Read the argument with your task in mind.

In the argument, despite the claim that the second statement “follows” from the first, it does not actually do so.

STEP 3:  Know what you’re looking for.

Therefore, you must find the assumption that, when inserted between the first and second statements, will make the conclusion work.

STEP 4:  Read every word of every answer choice.

Answer A ignores the condition of having or not having an office and thus does not help the conclusion. Answer B seems to meet your needs. Since every employee who takes public transportation eats lunch in the company break room, it follows that if some employees who eat in the company break room have an office (as stated in answer B), then there are some employees with an office who do not eat in the break room and thus do not take public transportation. Answer C results in every employee with an office taking public transportation, which contradicts the conclusion. Answer D does not lead to any conclusion involving those who have an office or whether they take or do not take public transportation. Answer E tells you something about those who eat in the break room but nothing about those who have an office or whether they might or might not take public transportation. Thus, the correct choice is answer B.

15. Answer: C

STEP 1:  Read the question and identify your task.

This is a Weaken question. The question prompts you to find the statement among the answers that most weakens the argument.

STEP 2:  Read the argument with your task in mind.

You need to read the argument and discover the central basis for its conclusion, then formulate what kind of answer will weaken that basis. In this case, the argument reveals a discovery that leads archaeologists to believe that a larger, more permanent civilization existed in the Brazilian Amazon.

STEP 3:  Know what you’re looking for.

A weakening statement may indicate that a smaller, less permanent civilization was responsible for the etchings.

STEP 4:  Read every word of every answer choice.

Answer A uses words like large and established to strengthen the conclusion that there was a well-developed civilization in the Amazon, so this cannot be the correct answer. Answer B offers irrelevant facts and no insight into the nomadic or permanent nature of the civilization. It onlyrelates to the sophistication of either form. Answer C reveals a discovery of similar etchings 100 miles away by a “nomadic” artist. You can infer that the artist was a member of a nomadic civilization as was originally thought before the discovery. This weakens the argument and would seem to be your answer. Regarding answer D, the argument does not say the civilizations in question did not exist before the 2,000-year flood window, and this answer also does not address the nomadic or permanent nature of the civilization. Answer E is interesting but does not weaken the argument as well as Answer C. While the content of the etchings is relevant, they do not necessarily prove the nature of the civilization since they could be depictions of previous civilizations or other civilizations. The correct choice is answer C.

16. Answer: B

STEP 1:  Read the question and identify your task.

This is an Assumption question. It is asking you to find a statement upon which the logic of the passage depends.

STEP 2:  Read the argument with your task in mind.

The conclusion is that people in industrialized nations with more choices “in all aspects of their life” are not happier than people in less developed nations with fewer choices.

STEP 3:  Know what you’re looking for.

The conclusion extrapolates from television to all aspects of people’s lives, so you expect that the correct answer will support the conclusion and most likely the extrapolation.

STEP 4:  Read every word of every answer choice.

Answer A focuses only on the television aspect of daily life and not other aspects of their lives. It also somewhat weakens the comparison of nations. Answer B states an inverse relationship—that more channels equals less happiness—also exists among “other categories.” If the same relationship exists among other goods and services regardless of other factors related to where people live in the world, then people in nations with more choices are less happy than people in nations with fewer choices. Thus answer B would seem to give strong support to the argument and to be your answer. For answer C, the argument says nothing about awareness between nations, only that the number of choices determines happiness. For answer D, how each nation judges each other’s choices is irrelevant to their own experience. Like answer A, answer E discusses only the television aspect of daily life. The correct choice is answer B.

17. Answer: C

STEP 1:  Read the question and identify your task.

This is a Paradox question. Reading the question, you discover that it wants you to find the one answer that does not explain the phenomenon described in the argument.

STEP 2:  Read the argument with your task in mind.

In the argument you learn that the cheerleading squad experienced lower sales than usual for its bake sale after it decided to lower prices.

STEP 3:  Know what you’re looking for.

Therefore, you must find the one answer that does nothing to explain the lower sales.

STEP 4:  Read every word of every answer choice.

For answer A, a lower population of potential buyers would very likely hurt sales. For answer B, competing fund-raisers would most likely decrease funds students have to spend on baked goods. In answer C, the cost of sugar and flour may affect profitability but has nothing to do with sales. Since answer C does not contribute to reconciling the apparent discrepancy, it would seem to be your answer, but you should continue to review the rest of your answers to make sure. For answer D, students probably would not buy more baked goods if they already had free cupcakes from the teachers, so that would definitely hurt sales. Finally, for answer E, location could very likely affect sales, especially if students had come to expect their sale table to be located in a traditional location. The correct choice is answer C.

18. Answer: D

STEP 1:  Read the question and identify your task.

This is a Strengthen question. It asks you to find the policy that fits the situation.

STEP 2:  Read the argument with your task in mind.

In this case, you need a policy that allows the community center to use the additional funds as it desires but also satisfies the interests of those who donated the money.

STEP 3:  Know what you’re looking for.

You will look for something similar in your answer options.

STEP 4:  Read every word of every answer choice.

Answer A gives the center complete freedom to do what it wants with all the funds but ignores the interests of those who donated the money. Answer B states that the center can do nothing with the money except what the donors approve, which ties the hands of the center while giving too much power to the donors. Answer C hamstrings the center by saying the directors must approve the purpose for all spending in advance, but it does not address funding overages and thus is not helpful in the situation described in the argument. Answer D states that the center should use the money for the intended purpose unless those funds cannot be used for that purpose, in which case the donors should be consulted. This policy fits the situation perfectly. It allows the center to use the money as it sees fit until the allocations are fulfilled. Then, they must consult the donors. Answer E requires that the center return the money, which is in complete contradiction to the situation. Therefore, the correct choice is answer D.

19. Answer: A

STEP 1:  Read the question and identify your task.

This is a Strengthen question. It asks you to find the answer that weakens the argument least, or in other words, most strengthens the argument.

STEP 2:  Read the argument with your task in mind.

The scientist argues that his peers’ statements that his theories are “based on sheer conjecture and have no experimental basis” are wrong because his theories are based on recent, reputable data. Although the scientist admits that he hasn’t read “every detail” of the information, he trusts the sources.

STEP 3:  Know what you’re looking for.

The correct answer is a criticism that has little or no effect on discrediting the argument. As you review your answer choices, a process of elimination works best in this situation.

STEP 4:  Read every word of every answer choice.

Answer A describes the basis of the conclusion as “uncertain recollections” even though the recollection does not seem uncertain at all. He refers to a specific study and use of the same research organizations. There is nothing uncertain about those recollections. Answer A seems like a good option. Considering answer B, if the experiments are affected by human error, then such a criticism would definitely weaken the scientist’s argument that the research supported the theory. Since the scientist argues that the experiments done by the research organizations are all that are necessary to support his theory, answer C would seriously weaken the argument by saying more support is necessary. Answer D calls into question the quality of the research, and since the scientist uses these studies as the sole support for the theory, this criticism most definitely weakens the scientist’s argument. Finally, if the logical pattern of using the lab’s results to support a theory does not work for the current theory, then the scientist’s argument is again weakened. Since answers B, C, D, and E all render the scientist’s argument vulnerable to further scrutiny, they can be eliminated, which means answer A is the correct choice.

20. Answer: D

STEP 1:  Read the question and identify your task.

This is an Assumption question. For this first question you must find the assumption that the argument depends on in order to hold true.

STEP 2:  Read the argument with your task in mind.

The argument is about annual tests of the lead levels in drinking water. The conclusion is that the water is safe to drink, even though some samples exceeded federal standards, because the levels were still too low to pose any risks.

STEP 3:  Know what you’re looking for.

The correct answer will address the relationship between the federal standards and the amount of lead that causes health problems.

STEP 4:  Read every word of every answer choice.

Answer A says the standards are too stringent, but the lead levels still may be harmful to people’s health, and whether they disregard the guidelines or not, they take a risk by drinking the water. Answer B seems to weaken rather than support the argument, and that is not what the question is asking. Answer C calls into question the method of testing and leaves open the option that the situation might be worse than stated, possibly leading to the opposite recommendation, that people should not drink the water. For answer D, if lead levels slightly above federal standards are still not harmful to people’s health, then it is valid to say that drinking the water is still safe for the city’s residents. This very much supports the argument and is critical for the argument’s conclusion. Answer E has nothing to do with the lead levels or drinking the water. Rather, it gives an irrelevant observation about human behavior. Therefore, answer D is the correct choice.

21. Answer: A

STEP 1:  Read the question and identify your task.

This is a Strengthen question. This question is asking you to find the answer that actually bolsters the claim in the argument.

STEP 2:  Read the argument with your task in mind.

The argument is about annual tests of the lead levels in drinking water. The conclusion is that the water is safe to drink, even though some samples exceeded federal standards, because the levels were still too low to pose any risks.

STEP 3:  Know what you’re looking for.

The correct answer will best support the conclusion that the water is safe to drink.

STEP 4:  Read every word of every answer choice.

Answer A says that most lead never makes it to the faucet as it is absorbed by the lining of the pipes, thereby lowering the threat of people being exposed to the lead and being harmed. This seems to give great strength to the argument and is most likely the correct answer. Answer B creates doubts about the testing methods, but it does not address the actual results. Answer C says children exposed to lead levels above the federal standards will be harmed, the opposite of what you are looking for. Answer D is anecdotal at best since the hospital was dealing with people who happened to drink large quantities of water, not the normal amount. Answer E does indicate a problem with the current federal standards, but it does not say on what basis the medical associations make their argument, and it is still not as strong an answer as A. Answer A is the correct choice.

22. Answer: B

STEP 1:  Read the question and identify your task.

This is an Assumption question. The first question asks that you find the one answer upon which Eleanor’s argument is based. The word depends requires that the statement be central to holding up the argument to scrutiny.

STEP 2:  Read the argument with your task in mind.

The subject of this conversation between Jaime and Eleanor is the life span of elephants in captivity. Jaime’s conclusion is based on studies that show that elephants living in two protected reserve parks in Kenya and Myanmar live longer than those that live in zoos. Eleanor’s argument is that zoos are painted in a bad light because the elephants in parks don’t face the same dangers as elephants in the wild and that the studies that Jaime cites on zoos are outdated.

STEP 3:  Know what you’re looking for.

Eleanor assumes that poaching and unnatural dangers are the more dominant threat to animals outside protected reserve parks. The correct answer will address this issue.

STEP 4:  Read every word of every answer choice.

Answer A may be true concerning the genetic differences between African and Asian elephants, but says nothing about what happens when either type is kept in a zoo versus an unprotected reserve. Answer B states that predators and diseases affect the life span of an elephant to a lesser extent than other threats beyond the protected parks. This would seem to be central to Eleanor’s argument since she indicates that the threat of poaching and unnatural dangers affect the life span of elephants much more than any other threats inside protected parks. Answer C contradicts the second part of Eleanor’s argument and is thus incorrect. Answer D says why an elephant’s life span in a zoo was shortened in 1970 but nothing to support Eleanor’s claim that care is better today. Also, it says that the poorer care then was due to budgetary reasons. Eleanor indicates that the science of care advanced, not funding. Answer E actually contradicts Eleanor’s argument by saying that elephants are safer in reserves from poachers and unnatural threats. The correct choice is answer B.

23. Answer: D

STEP 1:  Read the question and identify your task.

This is a Describe question. It asks you to find the answer that accurately describes the argumentation technique that Eleanor uses to make her case.

STEP 2:  Read the argument with your task in mind.

The subject of this conversation between Jaime and Eleanor is the life span of elephants in captivity. Jaime’s conclusion is based on studies that show that elephants living in two protected reserve parks in Kenya and Myanmar live longer than those that live in zoos. Eleanor’s argument is (1) that zoos are painted in a bad light because the elephants in parks don’t face the same dangers as elephants in the wild and (2) that the studies that Jaime cites on zoos are outdated.

STEP 3:  Know what you’re looking for.

Eleanor discards the premises that Jaime uses to reach his conclusion—namely, the link between living in a reserve park and a longer life, as well as the studies he cites, which she says are obsolete.

STEP 4:  Read every word of every answer choice.

Regarding answer A, Eleanor does not offer such an example, so it cannot be your answer. For answer B, Eleanor does not question the conclusion, only how Jaime reached it. Also, she does not question the verity of his data, only his choice of data population. Answer C says that Eleanor is trying to bolster Jaime’s argument, which is definitely not the case. Answer D states that Eleanor rejects the correlation between the zoos and nature reserves that Jaime chose to make his argument and that changing his data pool will affect Jaime’s supporting statements that led to his conclusion. These both seem to be exactly what Eleanor is doing with her argument. Answer E is partially correct in that she is questioning his assumptions, but she is not pursuing a wider data sampling but a different data sample altogether. Therefore, answer D is the correct choice.

24. Answer: D

STEP 1:  Read the question and identify your task.

This is a Weaken question. You need to find the answer that most weakens the argument.

STEP 2:  Read the argument with your task in mind.

What is the basis on which the teacher makes the argument? In this case, based on an experiment with 10 of her students, the teacher concludes that all her students would be better off doing mathematics manually than with machines.

STEP 3:  Know what you’re looking for.

Most likely, the weakening statement will undermine the experiment in some fashion.

STEP 4:  Read every word of every answer choice.

Answer A actually strengthens the teacher’s reasoning by making the students’ achievement even more impressive, so this option does not qualify. Answer B seems to indicate that the 10 students were receiving help that might have improved their performance. This could weaken the argument, but it does not indicate whether the other students were receiving help as well, so this option may not be as strong as another answer. Answer C only indicates how the 10 students compared to the other students but does not affect the judgment that they showed improvement from their own past performance. Thus, it does not weaken the argument. Answer D says that the students undergoing the experiment had seen the material before while the other students had not. This indicates that they had an advantage that improved their performance, and the lack of a calculator may have had nothing to do with it. This very much undermines the teacher’s reasoning and may be your strongest candidate. Answer E indicates only that the teacher gave them a methodology for working without a calculator, but this does not mean that such training enhanced their performance and does not necessarily affect the teacher’s reasoning. Answer D is the correct choice.

25. Answer: E

STEP 1:  Read the question and identify your task.

This is a Flaw question. The question is looking for a description of the logical flaw, rather than the flaw itself.

STEP 2:  Read the argument with your task in mind.

The argument concludes that there are more employers who believe one idea (people charged with a crime should be fired) over another (only people convicted of a crime should be fired), using the result of a recent survey.

STEP 3:  Know what you’re looking for.

The correct answer will describe why the statistics are misleading, specifically why one or both of the survey statistics may be double counting or undercounting the number who believe a person should be fired in either case.

STEP 4:  Read every word of every answer choice.

Answer A is a true description of what the statement does but does not necessarily describe a flaw. Most polls extrapolate from a sample to draw a conclusion about the general population. For answer B, there is no ambiguous term used in the supporting statements, nor is any such term used as a basis for the conclusion, so this one cannot be correct. Regarding answer C, the conclusion compares the two beliefs mentioned in the supporting statements and makes no mention of a third belief. For answer D, there is no doubt that all the premises can be true. Finally, regarding answer E, in order to be convicted, one must be charged, so it is very likely that more employers believe that an employee should be fired if convicted of a crime than those who believe the employee should be fired if just charged with a crime. A sufficient condition is being confused with a required condition. Thus the poll and the reasoning are flawed. The correct choice is answer E.

26. Answer: A

STEP 1:  Read the question and identify your task.

This is a Flaw question. It is asking for the flaw in a complete-the-sentence form.

STEP 2:  Read the argument with your task in mind.

Find the logical flaw that leads the editorialist to believe that the politicians will redirect the funds away from education.

STEP 3:  Know what you’re looking for.

The statement asserts that because politicians in general have betrayed the purpose of the lottery that the current legislature will do the same. You will look for something similar in your answer options.

STEP 4:  Read every word of every answer choice.

Answer A states that the argument draws a conclusion about a specific population (“our representatives in the state legislature”) based on a study of a larger population (“politicians”). This would seem to describe the editorialist’s mistake perfectly, but you should read through the rest of the answers to make sure. Answer B might look good, but the statements do not mention any historical data. It only makes general historical statements. Answer A remains the stronger answer. Answer C is incorrect because you do not know the conditions, historical or current, but even so, the current option on the table, a public lottery to support education, is the same. Both the historical politicians and the current legislature are considering the same solution, so answer D cannot be correct. None of the statements attacks the supporters or the merits of the lottery, only the resulting malfeasance once it is approved, so answer E cannot be the correct choice. The correct choice is answer A.

SECTION II

1. Answer: D

STEP 1:  Read the question and identify your task.

This is a Conclusion question. It is asking you to find among the answers a logical conclusion that can be reached as a result of the chancellor’s argument.

STEP 2:  Read the argument with your task in mind.

Thus, you read the chancellor’s statement with the expectation that it will lead you to an inevitable conclusion or assumption. Put simply, the chancellor argues that the school needs to invest in new facilities in order to attract new students and beat out the competition.

STEP 3:  Know what you’re looking for.

The correct answer will sum up the argument. In this case, the chancellor argues that Mayfield Academy must grow to survive, and building new facilities is a way to generate that growth, so you must look for a conclusion that the Mayfield Academy must invest in new facilities in order to grow.

STEP 4:  Read every word of every answer choice.

Answer A seems like it might work, but it only mentions attracting more students and not how that should be achieved. Answer B seems like something parents might say when or if the school actually agrees and executes the specifics of the chancellor’s recommendation, but this is not exactly a conclusion you can take from the argument. Answer C discusses a consideration of the parents and something unrelated to the school’s need to attract more students. Answer D states that the academy should invest in new facilities, a conclusion that the school might very well come to based on the chancellor’s argument. This seems like your answer, but you should review the final answer to be sure. Similar to answer A, answer E states what the school would like to be the inevitable decision parents make, to move their children to Mayfield, but it does not discuss how the Mayfield Academy can achieve that result, which is the subject of the argument. The correct choice is answer D.

2. Answer: C

STEP 1:  Read the question and identify your task.

This is a Describe question. The question asks you to describe the method the ethicist uses to respond to the business executive’s argument

STEP 2:  Read the argument with your task in mind.

In essence, the business executive argues that teaching ethics to employees is a waste of time because they will inevitably act unethically.

STEP 3:  Know what you’re looking for.

First you notice by the tone that the ethicist indicates disagreement with the executive (“makes as much sense as … ”) and that the ethicist uses an analogous situation (“spending money on driver’s education”) and that the analogous situation has what the ethicist considers an absurd justification (“all drivers will inevitably cause an accident”), indicating that such thinking will have an equally bad result. You will look for something similar in your answer options.

STEP 4:  Read every word of every answer choice.

Answer A seems closer to a description of the business executive’s thinking than the ethicist’s. Your question asks for the ethicist’s method, not the executive’s. With answer B, the ethicist makes no attack on the executive’s character, only on his argument, so this is not the correct answer. For answer C, the ethicist does use another (analogous) situation to show the executive’s reasoning is flawed and would lead to a bad result, so this answer seems like your best option, but let’s continue to review the rest of the options. Answer D does not seem right because the ethicist makes no demands for further evidence. For answer E, the ethicist does not think there is any dilemma to explicate. For the ethicist, there is no dilemma at all, as you see by his use of an analogous situation that is fairly black-and-white. The correct choice is answer C.

3. Answer: A

STEP 1:  Read the question and identify your task.

This is an Assumption question. You must describe the assumption upon which the argument depends.

STEP 2:  Read the argument with your task in mind.

In this case, the argument makes a case that Renfield is not qualified for a management position despite his qualifications and based on certain disqualifications.

STEP 3:  Know what you’re looking for.

The correct answer will most likely discuss either the qualifications or disqualifications, since they are the basis for the conclusion that Renfield should be a manager.

STEP 4:  Read every word of every answer choice.

Answer A describes the argument’s assumption with regard to Renfield’s one qualification, that “his performance as a member of your staff, while exemplary” does not in and of itself prove that the person can be a manager. This is definitely an assumption upon which the arguer makes the case, and it is very likely your answer, but you should read the rest of the answers to make sure. Answer B says Renfield cannot be trusted even with his regular duties, something the argument actually contradicts by saying that his performance has been “exemplary.” With regard to answer C, nothing in the statement is based on whose interest Renfield will represent once he is a manager. Rather, it is based on his management abilities. Answer D deals with the staff’s behavior and not Renfield’s, so this does not qualify for your answer. Finally, answer E is a very general statement regarding the general practices of the organization. It says nothing of the specifics of Renfield’s case and thus cannot be your answer. The correct choice is answer A.

4. Answer: E

STEP 1:  Read the question and identify your task.

This is a Flaw question. You need to figure out why the argument is false, but it structures the question in complete-the-sentence form, so the answer will most likely be a description of the logical flaw rather than the flaw itself.

STEP 2:  Read the argument with your task in mind.

In this case, the statement makes a simple argument—overturning accepted norms is the only way to make real progress.

STEP 3:  Know what you’re looking for.

The argument singles out one specific influence on the issue of real progress—overturning accepted norms—and it doesn’t allow for the possibility that another factor could have an impact. You will look for something similar in your answer options.

STEP 4:  Read every word of every answer choice.

Answer A seems to be describing another argument altogether since the argument does not undermine its own premise and the term attribute seems to be a fairly inaccurate description of what is discussed. Answer B might seem like the correct answer to the question, but the problematic word is because since there is no causal relationship being offered between the first statement and the second statement. Answer C is problematic with the first word, denies, because the argument is actually affirming such a contribution, not denying it. Answer D does not work because the argument does not isolate its observation to a particular time. It is more general than that. Answer E is your last remaining possibility. The argument makes the mistake of saying that one influence outweighs all other influences on progress. The correct choice is answer E.

5. Answer: C

STEP 1:  Read the question and identify your task.

This is a variation of a Describe question—a Describe How the Argument Proceeds question. The question is in complete-the-sentence form, and thus the answer will be a description of how the argument is formed and not necessarily anything specific within the argument.

STEP 2:  Read the argument with your task in mind.

The educator describes what the critics say about online universities and low-residency degree programs and then discusses how Plymouth Online is the exception to those objections.

STEP 3:  Know what you’re looking for.

You will look for something similar in your answer options.

STEP 4:  Read every word of every answer choice.

Answer A is incorrect because the educator makes no reference to any scholarly authority during the challenge. Answer B is incorrect because the educator does not, in fact, question the validity of the objections the critics make. The educator merely states how Plymouth Online overcomes those issues. Answer C may be your answer because the educator does use Plymouth Online as a counterexample to what the scholars say are the prevalent beliefs about the distance programs. You must review the rest of the answers to be sure whether you are finished. Answer D is off the mark because the term sociological is an inaccurate description of the educator’s discussion. The educator makes no sociological observations as to why distance programs are used. Also, this answer fails to mention the educator’s use of his or her own program as an example. Finally, answer E is not accurate because the educator does not compare two approaches. Also, this answer ignores the dynamic between the critical scholars and the educator’s discussion of Plymouth Online. The correct choice is answer C.

6. Answer: A

STEP 1:  Read the question and identify your task.

This is a Weaken question. You must find among the answers the statement that “most seriously” weakens the argument.

STEP 2:  Read the argument with your task in mind.

The statement argues that the government should require weight training in public schools for all children between ages 6 and 18 based on a new study.

STEP 3:  Know what you’re looking for.

You expect that the correct answer will be a statement that seriously undermines some aspect of this recommendation to the government.

STEP 4:  Read every word of every answer choice.

Answer A looks very likely to be your answer since it says that a school-based program will be ineffective and that a home-based program is what is required for real success. You need to read through the remainder of the answers to be sure. Answer B puts a damper on the ability to measure progress, but it does not undermine the benefits of the program or the recommendation. Answer C adds facts about the study, but the particulars are not the kind that would undermine the results of the study. Answer D only speaks to the time it takes to notice measurable results, which does not change the fact that such weight training is beneficial and thus does not weaken the argument. Finally, answer E might give you pause. Yes, there might be some health risks to weight training at such a young age and adding those tests to the study might have been helpful, but the results might have been positive as well. You do not have enough information to know and thus, this answer does not weaken the argument more than answer A. Answer A is the correct choice.

7. Answer: D

STEP 1:  Read the question and identify your task.

This is a Weaken question. This question asks for a statement that weakens the argument.

STEP 2:  Read the argument with your task in mind.

The statement is arguing that, based on a study, all people with type 2 diabetes should train in both aerobics and weight lifting. The key terms are the health problem type 2 diabetes and the recommendation for both exercise types.

STEP 3:  Know what you’re looking for.

The correct answer will cast doubt on the relationship between the premises and the conclusion.

STEP 4:  Read every word of every answer choice.

Answer A looks good, but just because people with this condition are “less likely” to manage it physically does not mean that ultimately they can’t manage it or that it will not benefit them as the study indicates. Answer B is an absurd statement because even if they cannot be done at the same time it does not mean that a training program cannot be designed to alternate between them. Also, the statement does not mention the condition type 2 diabetes. Answer C looks good, but pay close attention to the words that hurt its chances. It says that the physical demands “could” have negative physical effects. This is speculative and contradicts the study mentioned in the argument. If the answer said another study indicated such a detriment, then this answer might have qualified as your choice. Answer D is a strong candidate because if some people with type 2 diabetes need only one of the activities to get the same benefits, then the recommendation for both activities is weakened. Finally, answer E does not cast doubt on the relationship between the premises and the conclusion; in fact, it offers support for the results of the study. The correct choice is answer D.

8. Answer: C

STEP 1:  Read the question and identify your task.

This is a Deduction question. It asks you to find a statement that must be true on the basis of the given statements.

STEP 2:  Read the argument with your task in mind.

The argument gives you a paleontological discussion concerning the survival of large mammals like the hippopotamuses and giraffes after the extinction of the dinosaurs.

STEP 3:  Know what you’re looking for.

The correct answer will most likely be a conclusion you can come to based on how such survival occurred, mainly the competition for living space and food.

STEP 4:  Read every word of every answer choice.

Answer A tries to relate a later timing of extinction to the smaller size of the surviving mammals, but nothing in the argument leads you to believe there is a relationship between the two, only that when it happened their size changed. The only thing you can assume is that the increase in size would have been delayed. With regard to answer B, the statements give no size relationship between the large mammals like the hippopotamus and other mammals. It tells only of large mammals and how they grew in size. Answer C is a very good option for your question. The statements discuss how the large mammals grew in size because they no longer had to compete for space and food with dinosaurs. Thus, you can assume that if larger nonmammals had survived the extinction event, they would have been competitors for space and food and the larger mammals would have been smaller. Answer D may be true, but nothing in the statements leads you to believe that the size of the dinosaurs had anything to do with their extinction. Answer E states something that cannot be verified by the statements. It is possible they would have competed with carnivorous dinosaurs and the same evolutionary process would have occurred. Answer C is the correct choice.

9. Answer: D

STEP 1:  Read the question and identify your task.

This is a Parallel question. It asks that you find among the answers a situation that “parallels” the one given.

STEP 2:  Read the argument with your task in mind.

The given situation is that of an auction that Jody is not participating in; because nobody wants to buy the Giacometti statue more than she does, then “not one person” will bid on the statue no matter how low the bidding starts.

STEP 3:  Know what you’re looking for.

Most likely the correct answer will use similar extreme or all-or-nothing terms like “most,” “nobody,” or “not one person” and will create an equally absurd notion. The correct answer will have a similar flawed logic.

STEP 4:  Read every word of every answer choice.

Answer A starts out well, saying that one jockey wants something the “most,” but it breaks down later in the statement because it says the other jockeys will just double their efforts. This is not the all-or-nothing statement you need. Answer B starts off with Larry being the most qualified to spot a forgery, but the rest of the statement is wrong because it says that since one item has no flaws, others must be forgeries. This is not an all-or-nothing result of the first statement. Answer C does not say Professor Ricardo is the only one who can translate the text. It leaves open the possibility that there are others, definitely not an all-or-nothing result. In answer D, Emilio is the “most” intent to get the sales position, and because he is not applying, nobody else will apply no matter how high the salary goes. You have your extreme (“most”) and all-or-nothing result (“nobody else”) words and the statement is equally flawed in logic. This would seem to be your answer, but you must read your last option. In answer E, Sherry is not the most motivated to join the group, and her timing conflicts result in only a gradually worsening situation, not an all-or-nothing situation. The correct choice is answer D.

10. Answer: D

STEP 1:  Read the question and identify your task.

This is a Deduction question. It asks you to find among the answers the one statement that must be true assuming the statements in the passage are true.

STEP 2:  Read the argument with your task in mind.

The argument gives a set of conditionals, one for the conductor, one for the orchestra, and one that binds them together. Use shorthand to note these conditions: if C not NY, then other competitions. If O not NY, then no competitions. Lastly, C or O not NY. You can infer from the last statement that if C yes NY, then O not NY, or if O yes NY, then C not NY.

STEP 3:  Know what you’re looking for.

The correct answer will most likely test that you understand how these conditionals work together. As you begin to read the answers, you notice that they are a series of “if … then” statements. You should attempt the same shorthand with each to evaluate them.

STEP 4:  Read every word of every answer choice.

Answer A says O yes NY, then no competitions. This contradicts your second conditional altogether, so this is not your answer. Answer B cannot be done in shorthand because it discusses a probability (“more likely”), and your given conditionals are not based on likelihoods. They are certainties, so this cannot be your answer. Answer C also cannot be done in shorthand because it is a recommendation, not a statement. The word should is key to recognizing the problem with this option. Answer D says C yes NY, then no competitions. You know that if C yes NY, then O not NY. Thus, if O not NY, then no competitions. This would seem to be your answer. Answer E, similar to answer B, deals in probabilities (“likely”) and disqualifies this option. The correct choice is answer D.

11. Answer: D

STEP 1:  Read the question and identify your task.

This is a Parallel question. You must find the answer that follows the same pattern of reasoning as the statement.

STEP 2:  Read the argument with your task in mind.

In this case, the statement puts you through some mental gymnastics over newspaper coverage of an event, or lack thereof, and whether that determines the event’s newsworthiness. Altogether, the argument says that whichever way it’s worded it is a ridiculous statement. The statements use convoluted language to somewhat confuse the issue but can be simplified to “if … then” statements. The first statement is “if a newspaper does not cover an event, then it is not newsworthy.” The second version is just a rephrasing, but it could be reworded as “if an event isnewsworthy, it does appear in a newspaper.”

STEP 3:  Know what you’re looking for.

The key is to look for a similar pattern, including the use of logical words (negative-negative, positive-positive) and the ordering of phrases (newspaper-newsworthy, newsworthy-newspaper).

STEP 4:  Read every word of every answer choice.

Answer A says “if a politician IS elected to Congress, then he or she WILL BE corrupt.” You don’t have to go to the second sentence because the first pattern does not match. The key words are in the positive and not the negative. Answers B and C start the same as answer A, so you can quickly eliminate them as well. Answer D states “every student who is NEVER accused of cheating has NEVER cheated” and “every student who DOES cheat IS accused of cheating.” This seems very close to your pattern. The logical words follow the same pattern (negative-negative, positive-positive) and the sentences do the same reversal of phrases (accused-cheated, cheated-accused), but you should review the last option to be sure. Answer E starts out the same as answer D, but the second statement does not reverse the phrases. Instead it says, “every student who is accused of cheating has cheated” (accused-cheated, accused-cheated). You are looking for the option that most resembles the argument and in this case the correct choice is answer D.

12. Answer: B

STEP 1:  Read the question and identify your task.

This is a Weaken question. You are to find a basis upon which the argument loses its effectiveness.

STEP 2:  Read the argument with your task in mind.

The philosopher is arguing that pragmatism leads to relativism, which makes it difficult to develop a general guidance for moral and ethical decisions.

STEP 3:  Know what you’re looking for.

This question requires you to look through the answers and make a judgment as to which does the best job of weakening the argument, but most likely the correct answer will eliminate the problem of relativism and create some basis for general guidance for moral and ethical decisions.

STEP 4:  Read every word of every answer choice.

Answer A goes beyond the scope of the argument and says that people should get together and decide what is right for the entire society. This cannot be your correct answer. Answer B argues that the individual’s judgment is suspect and that experience in the world eliminates relativism (“the great equalizer”) and resolves the discrepancies to develop better judgment. This option seems very promising because it eliminates relativism and creates a basis for general judgment of actions, but you should review the remaining answers. Answer C actually strengthens the argument that pragmatism results in no general truth and individuals who disagree on what is practical and good, so C cannot be your answer. Answer D, much like answer C, is making a case for supporting the philosopher’s statement. Answer E is a “let’s just throw our hands up in the air and give up” statement that has nothing to do with the specifics of the philosopher’s argument. The correct choice is answer B.

13. Answer: D

STEP 1:  Read the question and identify your task.

This is a Weaken question. You must find the answer that points to a weakness in the argument.

STEP 2:  Read the argument with your task in mind.

The argument states that people should stop taking vitamin D and calcium supplements because they get enough of the vitamins from their diet and from sun exposure.

STEP 3:  Know what you’re looking for.

The correct answer will cast doubt on the relationship between the premises and the conclusion.

STEP 4:  Read every word of every answer choice.

Answer A is an interesting fact about the interaction between vitamin D and calcium, but it has nothing to do with people’s intake of the two nutrients. Answer B may very well weaken the basis for the argument, but this statement asks you to make an unfounded judgment that analyzing other publications is not as valid as developing a study and performing a direct study of patients. After all, 1,000 publications is still very impressive. You continue to look for a stronger answer. Answer C gives an interesting fact about a common practice of physicians, but this does not have any relevance to the committee’s analysis. Answer D says that there might be other benefits, beyond bone health, to taking the supplements, and this may very well weaken the argument since the basis for the study was to assess whether the supplements helped sustain bone health. This could be your answer. Answer E is too minor an objection since you do not know whether the non-breakfast foods with added vitamin D and calcium have enough of the nutrients to make up for missing breakfast. Thus, the correct choice is answer D.

14. Answer: D

STEP 1:  Read the question and identify your task.

This is an Assumption question. It asks you to find a statement upon which the conclusion depends, so you are looking for something that must be true for the conclusion to work.

STEP 2:  Read the argument with your task in mind.

In this case, the argument says that Company X is going to sell underperforming divisions that are dragging down the stock price. The analysts are concerned. As evidence that this is a good move, you are told that the president has long recommended that some divisions are outdated and are too expensive to bring up to date. The conclusion is that the board’s move will help the stock price.

STEP 3:  Know what you’re looking for.

The correct answer will be something that allows you to believe that, given the facts, the sale will actually benefit the stock price.

STEP 4:  Read every word of every answer choice.

Answer A is an interesting fact but is not necessarily critical to the conclusion. Answer B relates to the chances of success or failure of the sale but not the conclusion that once they are sold, that will help the stock value. Answer C may mean a delay in the sale, but whether the sale helps the stock price does not depend on when the sale goes through, rather that it goes through at all. Answer D seems rather important. The argument bases its conclusion on the statement that the president has long maintained that certain companies should be sold. Thus, those companies must be part of the sale or the stock price improvement may not happen. This seems to be a promising option. Answer E is about the buyers, and whether the buyers recognize an underperforming division or not would seem irrelevant to the sale and its effect on the stock price. The correct choice is answer D.

15. Answer: A

STEP 1:  Read the question and identify your task.

This is a Weaken question. It asks that you find among the answers a statement that weakens the argument.

STEP 2:  Read the argument with your task in mind.

In this case, the argument says that Company X is going to sell underperforming divisions that are dragging down the stock price. The analysts are concerned. As evidence that this is a good move, you are told that the president has long recommended that some divisions are outdated and are too expensive to bring up to date. The conclusion is that the board’s move will help the stock price.

STEP 3:  Know what you’re looking for.

You are looking for an answer that undermines the recommendation to sell the underperforming divisions.

STEP 4:  Read every word of every answer choice.

Answer A suggests that all divisions are integral to maintaining the stock price. If all divisions must stay together, then the sale of some of the divisions might hurt the “interaction” between all of the divisions and, therefore, the stock price. This is a strong candidate for your selection since it indicates that selling some divisions might actually drag the stock price down further rather than bolstering it. You must read through the other answers to make sure it is the strongest among them. Answer B tells you an interesting fact about selling different types of divisions, but since the monies received for the underperforming divisions is of little consequence to the conclusion, this cannot be your selection. Answer C tells you something of the history of the company, but again, the past “thought” concerning the divisions is of little consequence to the conclusion. Answer D is about inflation, and inflation is not even discussed in the argument. Answer E says the company is looking for alternative ways Company X can bolster its stock price. Some alternate way may exist, but we don’t know if they work. And either way, it does not say that the sale of the divisions is not still a good option or even a preferable option. Thus, the best choice is answer A.

16. Answer: B

STEP 1:  Read the question and identify your task.

This is a Parallel question. It asks you to find among the answers a statement that follows the same pattern of reasoning as the argument.

STEP 2:  Read the argument with your task in mind.

As you read the argument, pay close attention to the structure of the argument. The argument structure can be simplified as follows: IF restaurants over six months, THEN popular with patrons OR food critics. Last year, IF restaurants popular with food critics, THEN popular with patrons. THEREFORE, last year, IF a restaurant was over six months, THEN it was popular with food critics.

STEP 3:  Know what you’re looking for.

This is a fairly complicated pattern of logic but it should be repeated in the answer.

STEP 4:  Read every word of every answer choice.

Answer A is IF garages are in Caedmon, THEN they do maintenance on both foreign AND domestic automobiles. You can stop right there since the first sentence does not include the “or” construction. This cannot be your selection. Answer B is IF apprentices are at Willow, THEN apprentices study dry wall OR cabinetry. This year, IF apprentices study dry wall, THEN they study cabinetry. THEREFORE, this year, IF an apprentice is at Willow, THEN the apprentice is studying dry wall. This is exactly the pattern and is probably your answer. Check the rest of the answers to be sure. Answer C starts out well with IF a congressperson is no longer in Congress, THEN the congressperson teaches OR writes a book. This answer then gives a specific example of Mary Seldon, and this is not in the pattern of your argument that stays general in its terms. Thus this cannot be your selection. Answer D makes the same mistake as answer C in that in the second part it goes into a specific example. Answer E is problematic from the start because it speaks in terms of “most” new movies when your argument talks in absolutes (“all” or “every”). Thus, the correct choice is answer B.

17. Answer: A

STEP 1:  Read the question and identify your task.

This is an Assumption question. It asks you to find the assumption upon which the argument depends.

STEP 2:  Read the argument with your task in mind.

In this case, the argument makes a claim that a plant’s immune system behaves similarly to the human immune system and then makes an outlandish claim that you can discover how plants fight off the common cold.

STEP 3:  Know what you’re looking for.

You read through the answers for the one statement that seems to uphold this comparison.

STEP 4:  Read every word of every answer choice.

Answer A says the same diseases that attack humans also attack plants. This is deceptively simple, and you might think it too simple to be the correct answer. Nevertheless, this may be your best option, since the outlandish conclusion is based on the very idea that the common cold attacks plants as well as humans. You must review the rest of the options to be sure. Answer B is definitely true, but it is not a claim upon which the comparison between plants and humans depends. It also says nothing about plants at all. Answer C is close to saying the same thing as answer A, but it creates a causal relationship (“because”) that does not necessarily serve as a foundation to the argument. It also creates unnecessary complications by discussing how the disease attacks either one. Still, if answer A was not so effective, you might consider this option. Answer D is definitely important to the conclusion of the argument that makes the generalization about all plants based on the rice plant, but this option says nothing about the common cold or disease and is still not as strong as answer A. Answer E enhances the comparison between the immune systems of plants and humans, but this does not support the entire argument concerning the common cold and disease. Therefore, the best choice is answer A.

18. Answer: E

STEP 1:  Read the question and identify your task.

This is a variation of a Describe question—an Identify a Point of Disagreement question. You need to figure out the central disagreement between two people.

STEP 2:  Read the argument with your task in mind.

Joseph makes a value judgment that the evening customer service representatives are more efficient than the morning representatives based on certain performance statistics. Davis disagrees, arguing that each shift has different demands and the representatives face different kinds of problems.

STEP 3:  Know what you’re looking for.

You can expect that the issue in dispute will describe how Joseph and Davis differ on their definition of efficiency.

STEP 4:  Read every word of every answer choice.

Answer A says they disagree about why the evening shift is able to remain on that shift and not on the day shift. This is totally unrelated to the efficiency issue and cannot be the correct answer. Answer B says the disagreement is about the relationship between the time of day and the amount of time it takes to handle a complaint. This is close to being a good description, but it seems somewhat inadequate. You should read the rest of your answers to see if there is a better option. Answer C cannot be your answer because neither of them discusses whether the company will move representatives back and forth between shifts. Answer D is incorrect because they do not disagree on the accuracy statistic itself. Rather, they disagree about why the statistical discrepancy exists at all and whether that should be used to judge their efficiency. Answer E says the disagreement is about why the two shifts can offer quality service but one shift is faster than the other. This seems to be a perfect description of the disagreement and much better than answer B, your second best answer, which focuses only on one aspect, how the time of day relates to the time it takes to handle a complaint. The correct choice is answer E.

19. Answer: A

STEP 1:  Read the question and identify your task.

This is a Conclusion question. In essence, the question is asking you to identify an answer that states the reason legislation is not working in the best interest of constituents.

STEP 2:  Read the argument with your task in mind.

The first part of the argument is a description of the current way legislation is drawn up. Starting with “But clearly this strategy …,” you learn what the problem is, which is that the industry experts, as lobbyists, are paid by the industry they represent and thus act in the industry’s interest, not the constituents’.

STEP 3:  Know what you’re looking for.

You are looking for something similar among the answers.

STEP 4:  Read every word of every answer choice.

Answer A says that the industry experts will let their self-interest as industry lobbyists affect their writing of legislation. This is almost exactly your expected answer, but you should read through the remaining options to be sure. In answer B, several words work against it, especially “heavily influenced financially” and “unfair.” The argument says they are paid, but you have no idea how much their compensation influences them. Also, the argument says only that the legislation is weakened, but there is no indication of how unfair it is. These terms make it difficult to choose this option. Answer C discusses the legislators being less corrupt in writing legislation. Although this is a corollary of the argument, this is not the reason the current method is not in the best interest of constituents. Answer D may be true, but it is constructing new information and attributing a motive to the industry experts that is not even mentioned in the argument. You have no idea whether they expect to obtain lucrative jobs in their respective industries at a later date. This cannot be your answer. Similar to answer B, answer E contains words that disqualify it as your choice. The words “generously” and “want to keep” give information not evident in the argument. There is no mention of how much they are paid or that their jobs are at risk in this relationship. Thus, the correct choice is answer A.

20. Answer: C

STEP 1:  Read the question and identify your task.

This is a Deduction question. The question asks that you find which answer must be true based on the statements in the argument.

STEP 2:  Read the argument with your task in mind.

The argument gives a set of conditionals, so you can simplify them into basic logical statements. The first sentence says Diamonds(R) > 3 carats and Rubies(R) < 3 carats. The second sentence: Most Diamonds(R) and most Rubies(R) < SI2. The third sentence: Diamonds(A) and Rubies(A) > SI2. Also, Diamonds(A) and Rubies(A) < 3 carats. Because Ellington only buys stones < 3 carats, you can figure out that E can buy Rubies(R), Rubies(A), and Diamonds(A). In the final sentence you learn that Ellington is buying only a diamond shipment. Thus, Diamonds(A) is the only option. Allister is the only source for that shipment.

STEP 3:  Know what you’re looking for.

You read through the answers with this in mind.

STEP 4:  Read every word of every answer choice.

Answer A is too absolute and cannot be true. Ellington may be buying only diamonds currently, but there is nothing telling us that the company never buys rubies and only buys diamonds. Answer B says the opposite of what is given in the statements. In fact, Allister sells higher clarity stones than Richman. Answer C says the diamond shipment has a clarity rating above SI2. This works. Since Ellington must buy this shipment from Allister and Allister only sells stones with a clarity rating above SI2, then the diamond shipment must be rated above SI2. As for the other answers, answer D cannot be true since Ellington only buys stones < 3 carats and you know that Diamonds(R) > 3 carats. Answer E cannot be true because Allister is the only source that Ellington can buy the diamond shipment from. The correct choice is answer C.

21. Answer: B

STEP 1:  Read the question and identify your task.

This is a Flaw question. The first question asks that you find the flaw in the environmentalist’s response to the coal plant manager.

STEP 2:  Read the argument with your task in mind.

The coal plant manager claims that because of its cost effectiveness, coal will remain a dominant source of energy. The environmentalist argues that technology will improve the efficiency of alternative energy sources and this will enable them to beat out coal.

STEP 3:  Know what you’re looking for.

You can see that the environmentalist discusses technology and its efficiency benefits, but does not address directly the manager’s discussion of the cost benefits of coal over alternatives. You will look for something similar in your answer options.

STEP 4:  Read every word of every answer choice.

With answer A, the environmentalist does fail to address the length of time it would take to become competitive, but time is not central to the discussion in the first place and this is a weaker option. Answer B says the environmentalist fails to acknowledge the cost advantage of coal over alternatives. This matches what you noticed about the environmentalist’s statement and is most likely your answer. You should review the remaining options to be sure. Answer C is incorrect because the environmentalist makes no such statement. In fact, the environmentalist says that the coal industry has been slow to make such an adoption of clean coal. Answer D discusses a scenario that neither the coal manager nor the environmentalist discusses and thus cannot be your answer. You might be able to infer answer E from the environmentalist’s statements, but because the environmentalist makes no mention of an eventual cost advantage, you cannot choose this one. The correct choice is answer B.

22. Answer: C

STEP 1:  Read the question and identify your task.

This is a Strengthen question. This question asks you to find among the answers a statement that supports the environmentalist’s argument.

STEP 2:  Read the argument with your task in mind.

The environmentalist’s argument is based on technology and efficiency and the coal industries’ slow adoption of new technologies

STEP 3:  Know what you’re looking for.

The correct answer will be along those terms, and you should expect that it helps the environmentalist overcome the weakness discovered in the previous question, addressing the cost effectiveness issue.

STEP 4:  Read every word of every answer choice.

Answer A seems to contradict the environmentalist’s statement with regard to clean coal, and it does not help the environmentalist overcome the cost effectiveness issue. Answer B weakens the coal plant manager’s argument but does not strengthen the environmentalist’s argument. Answer C says there is a direct relationship between technological advancement and cost-adoption factors. This matches your requirements by saying that the environmentalist’s focus, technological advancement, affects cost effectiveness. You should review the remainder of the answers. Answer D discusses only one company, and the experience of one company does not necessarily weaken or bolster either argument. Lastly, answer E actually seems to weaken the environmentalist’s argument, saying that cost benefits have been minimal despite technological advances. Therefore, the correct choice is answer C.

23. Answer: C

STEP 1:  Read the question and identify your task.

This is a Describe question. The question is asking you to find the answer that describes the method by which the argument is made.

STEP 2:  Read the argument with your task in mind.

In this case, the argument gives a scenario. In essence, Randy must work on a report all evening for his boss, but a client has asked him to dinner and a discussion afterward that same evening. The final statement says that he has a difficult choice between satisfying his boss and satisfying his client, but he cannot do both.

STEP 3:  Know what you’re looking for.

You must look for a similar pattern in the answers.

STEP 4:  Read every word of every answer choice.

Answer A cannot be correct because the argument does not give alternative versions of Randy’s evening. It speaks of choices. Answer B cannot be correct because the argument does not give another situation for comparison. There is just the one situation that Randy faces. Answer C looks like it is your answer. The argument does talk about one set of responsibilities (the boss) and shows how it is incompatible with another set of responsibilities (the client), which results in a conundrum (cannot do both). This would seem to be your answer, but you should read through the remaining options to be sure. Answer D cannot be correct because the boss’s requirements do not lead to the client’s requirements. They are concurrent circumstances. Answer E starts off well. Randy did fail to work on the report in a timely manner, which led to the conundrum, but nothing in the arguments indicates that harm will inevitably come to any of the parties involved. The correct choice is answer C.

24. Answer: D

STEP 1:  Read the question and identify your task.

This is a Deduction question. Reading the question, you learn that you must choose an answer that can be inferred or derived from the content of the argument.

STEP 2:  Read the argument with your task in mind.

The argument tells you that bumblebees prefer red or striped snapdragons. Then, you are told that the bumblebee is critical to the survival of the snapdragon. Lastly, the nursery grows more striped and darkly pigmented snapdragons to encourage growth in the bumblebee population.

STEP 3:  Know what you’re looking for.

You expect your correct answer to discuss the relationship between snapdragons and the bumblebee population.

STEP 4:  Read every word of every answer choice.

Answer A cannot be inferred because nothing in the argument indicates that the bumblebee population is in danger from a lack of a particular snapdragon. The argument just suggests that growing the snapdragons will help increase the population. Answer B cannot be inferred because the argument says that the bumblebee visits the striped or darkly pigmented snapdragons more often, but it does not say that without that type it will stop pollinating snapdragons altogether. Answer C cannot be inferred because the argument gives no relationship between the bumblebee’s behavior and that of other insects. Answer D seems promising. It is evident from the argument that the bumblebee population needs to grow, so it is not much of a reach to infer that the population is smaller than desired. Also, you can infer that more striped and darkly pigmented snapdragons will help increase their numbers because the nurseries would not execute such a strategy if they did not believe it would do so. Read the last option just to be sure it is not better. Answer E says the opposite of what the argument is stating, that the bumblebees are necessary to save the snapdragons, which is not the case. Answer D is the correct choice.

SECTION III

Questions 1–6

As with all logic games you follow the six-step process.

STEP 1:  Identify the Game Type.

The wording of this logic game is tricky. It asks that you maintain an order: a coil cannot be used unless it is tested first. It also asks that you group the coils into untested, tested, and used. The game also does not tell you how many of the coils are in each group, and a coil can be in both the tested and used groups. This is one of the rare hybrid games that cannot be easily classified. As part of your test-taking strategy, you might choose to leave this game to last when taking the exam. Proceed to step 2 under that assumption.

STEP 2:  Begin Your Diagram.

You visualize a process of coils moving from left to right, from untested to tested to unused to used. Here is your diagram.

Images

STEP 3:  Symbolize the Clues.

In symbolizing the clues, you will pair the coil with its status. Techniques for symbolizing are explained in Chapter 3

The first clue says “If G is tested, I is tested.” This is a simple “if … then” statement. You will use the equals sign to say that a coil is in a particular stage of the process. This clue can be symbolized as follows:

G = T → I = T

The second clue is “If E is tested, G is tested,” which is another simple “if … then” statement. You use the same format as the previous clue:

E = T → G = T

The third clue is a straight definition and can be symbolized simply as:

D = T

The next two clues are worded as complex conditional statements that need to be translated into “if … then” language. They are:

H is not used (~H), unless J is tested (J)

D is not used (~D), unless H is tested (H)

You negate the first terms (~H and ~D) and put the later terms (J and H) after the arrows. The end results are as follows:

Images

Finally, you are given the clue “If J is used, and I is tested, K is used.” This is a compound “if … then” statement that can be symbolized as follows:

Images

This completes your symbolization, and you move on to step 4.

STEP 4:  Double-check your symbolizations.

To double-check your symbolizations, translate your symbolized clues back into normal English and see whether they match the original language of each clue.

Your page should look like the following:

Images

STEP 5:  Make deductions.

Finally, before you tackle the questions, see if you can make any deductions based on the setup of the game and the clues. Go through each type of deductions.

1. Can’t-be-first-or-last deductions

This is not an ordering game, but it has some ordering elements in it. Based on what you know, you can make the following deductions:

If G is tested, then I is tested, so G can never be tested without I also being tested.

If E is tested, then G is tested, so E can never be tested without G also being tested.

If H is used, then J is tested, so H can never be used without J also being tested (J will always precede H).

If D is used, then H is tested, so D can never be used without H also being tested (H will always precede D).

If J is used and I is tested, then K is used, so you can deduce that when J is used and I is tested then J and K will both be used together.

2. Repeated-element deductions

You see immediately that the first two “if … then” statements share the term G = T (G is tested). You see that E = T → G = T and G = T → I = T. Thus, using the reflexive law (if a = b and b = c, then a = c), you see that E = T → I = T.

Thus, you deduce that if E is tested, then G and I are tested. You can also add the “if … then” statement:

E = T → G = T & I = T

There are no other repeated elements that can be used to make further deductions.

3. Down-to-two deductions

There are no limitations on the number that can be in each stage, and the deductions do not put any limiting factors with words like “only” or “must have.” Therefore, you have no down-to-two deductions.

4. Block-splitting deductions

Notice that a coil can appear in the tested and the used column.

STEP 6:  Answer the questions in the smartest order.

On test day, answer the questions in this order:

1. Answer the Complete and Accurate List question.

2. Answer questions that give you more information to work with.

3. Answer the remaining questions.

In this example, questions 1 and 4 are Complete and Accurate List questions and should be done first. Questions that give you more information include:

Question 3 (“J is not tested”)

Question 4 (“E and five other coils are the only coils tested, and if exactly three coils are used in the machine … ”)

Question 5 (“Every coil that is tested is used, and if I is used … ”)

Question 6 (“K is not used, and if exactly four coils are used … ”)

Once you have answered these questions, that leaves just question 2 to answer.

THE ANSWERS

1. Answer: D

This question asks which of the answers could be a complete and accurate list of the coils that are tested. You are concerned only with the tested coils. Since you are dealing with only one group, you can solve this question without your visual. It is best in this case to look through the answers one by one and check whether they follow the rules. You know that D is tested. Every answer includes D, so disqualification will have to come from the other clues. In answer A, E is tested and you know that if E is tested, then G must be tested. So far so good, since G is the next one listed. But, thanks to your deductions, you also know that if E is tested, I must also be tested. Since this answer does not include I in the list, it can be eliminated. Answer B has a similar problem. It has D and E and I, but it excludes G. Answer C is even worse. It includes E, but both G and I are missing. Answer D says that the mechanic tests D and I. There is no clue that requires that D or I be tested with any other coil, so this could very well be your complete and accurate list. Answer E has D and G, but you know that if G is tested, I must also be tested, and I is not listed in answer E, so you can eliminate it as your choice. The correct choice is answer D.

2. Answer: A

This question is asking which statement could be true. The key is the word could, which tells you that the answer must be “possible” given the game setup and clues. The other answers will be impossible given the same facts. The way to answer this question is to review the answers and test them against your clues and deductions. Scanning all the answers, you can quickly see that all the answers are about which coils are tested, so you are not concerned about the ones that are used. You know that D is tested no matter what, so this will figure into your considerations. Answer A says that E and three other coils are used. You use the one certainty, that coil E is tested, to figure out whether this answer could be true. You run through the clues related to coil E. You know that if E is tested, then G is tested. That leaves one last coil to figure out. Through your deduction you know that if E is tested, I is also tested. That gives you three other coils (D, G, and I). This scenario is not only possible but also necessary when given that E is tested. You look through the other clues and see that none of them makes it necessary that another coil be tested. You could stop right there, but let’s review the other answer choices. Answer B is not possible because you have already proved that with E there must be three other coils tested. Answer C has the same problem; with E there must be three other coils tested, so it cannot be possible that E and one other coil are tested. Answer D says that D, G, and H are the only coils tested. The key word is only, which means only these three coils are tested and no others. This cannot be true because your rules say that if G is tested, then I must be tested. Since I is not in the list, this answer can be eliminated. Finally, answer E says that I and J are the only coils tested. Again the key word is only. This cannot be the case since your rules say that D is tested and this answer does not include D in its list. The correct choice is answer A.

3. Answer: C

This question gives you new information to work with. It says J is not tested. The question asks you to figure out which answer must be true. The key word is must, which means that because J is not tested, a certain requirement is created for other coils. You look at your clues and deductions and see what that condition might be. The J coil appears in the first clue as follows:

H = U → J = T

Because your given fact is the negative of the term on the right side of the arrow, it might be useful to develop the contrapositive of this clue. You flip sides:

J = T → H = U

Then, you negate both sides:

J ≠ T → H ≠ U

And now you have a new version of your clue that fits this question. It says that if J is not tested, then H is not used. You can add this to your list of clues in case it helps you with later questions. The last clue also involves J and can be symbolized this way:

J = U & I = T → K = U

This clue is based on J being used, but your question asks about J not being tested, so this clue cannot help you.

With your new information you evaluate your options. Answer A says D is not used. In your given situation, J not being tested only tells you about other coils being tested or not being tested. It tells you nothing about what coils are used. You cannot know whether D is used or not, so this cannot be your answer. Answer B says H is not tested. The only thing you can derive from your clues is that because J is not tested, H is not used. H may or may not be tested. You cannot be sure given your clues, so answer B is not your choice. Answer C says that D is tested, but H is not used. The first part is given by your clues. D is tested regardless of the scenario. The second part is confirmed by the second part of your contrapositive above. This appears to be your answer, but you should go through the final two answers just to be sure. Answer D states that D is used, but that does not have to be true, and the question asks what must be true. Answer E says H is tested but not used. The second part of this answer is confirmed by your contrapositive, but the first part is not. You do not know whether H is tested or not, given the clues. The correct choice is answer C.

4. Answer: C

The question tells you that coil E and five other coils are the only coils tested and exactly three coils are used in the machine. The key words are only and exactly. These words create a constraint that will most likely help you determine your answer. Since this question requires an accounting of both tested and used coils, you will use your visual tools to represent the various scenarios. You also notice that it asks what “could be” an accurate list. This means you need to determine only what is possible, not what must be true. You know that D is tested, and you are given that E is tested by the question. Put both in the tested column of your diagram. The question says that there are five other coils that are tested in addition to E. D counts as one, so you need to figure out four more.

Images

Using the clues to the right of your diagram, you know that if E is tested, then G is tested as well. You can also use your deduction to derive that if E is tested, then I is tested. You add G and I to your tested column. As represented below, this leaves H, J, and K as options to fill out the remaining two tested slots. One of the three will remain untested and also remain unused, since only those tested can be used.

Images

There is nothing more you can do to fill out the grid, so you look at your answers and see whether any of them work under the scenario you have built. Answer A says D, I, and J are used. You test this scenario. If D is used, then H is tested, which leaves you one more test slot for J or K. Then, you have I and J to contend with. Coil I works since it’s already tested. J must be tested in order for it to be used, so the last two test slots are taken by H and J, leaving K untested and unused. You have a problem, though. If J is used and I is tested, then your last clue tells you that K must be used. K cannot be used because only three coils can be used, and including K would make it four. That is enough to disqualify this answer, but you can also say that if K is used then it must be tested, and you have run out of test slots for K, which is a double disqualification. This scenario is represented like this:

Images

Answer B says that D, H, and J are the three coils used. As before, if D is used, then H gets one of your two remaining test slots. Another clue says that if H is used, then J is tested, so J takes the last remaining test slot. Again, you have a problem, because if J is used and I is tested, then K is used and K would make it four coils used, which is impossible under the conditions of the question. Answer C says that E, G, and I are the three coils used. You already know that E, G, and I are tested, so they can be used. None of these triggers any conditions that break your limitations, so this is very likely your answer. You will review the rest of the answers to be sure. Answer D says that H, I, and K are the coils used. First, if H is used, then it must have been tested, so it takes one of your two remaining available test slots. From the clues, you know that if H is used, then J is tested and it takes the last of your test slots. You have no more test slots available, which is a problem. If K is used, it must be tested as well and there is no test slot available for it. This cannot be your answer. Answer E says that E, G, and J are the three coils used. You know that E and G are tested, so they can be used. If J is used, then it triggers your last clue and K must be used, but that is not possible because only three coils can be used, and once again, K would make four. The correct choice is answer C.

5. Answer: E

This question says that every coil that is tested is used and coil I is used. It also asks which coil does not have to be tested. In other words, four out of the five options must be tested and one does not. You must find the one exception. You use your visual again. Since every coil that is tested is used, and you know that a coil must be tested in order to be used, both D and I occupy your tested and used columns.

Images

Based on what you know already, you review your clues to see whether they tell you anything more. According to your fifth clue, if D is used, then H must be tested. So, answer B is eliminated. Since H is tested, it will be used because, according to the question, all coils tested are used. From clue 4, if H is used, then J must be tested. Thus, answer C is eliminated. Accordingly, since J is tested, it will be used. From clue 6 you know that if J is used and I is tested, then K is used and therefore must be tested. This eliminates answer A. You are left with answers D and E. Thanks to your clues you know that coil D must be tested, so answer D is eliminated. This leaves answer E and it must be your answer. Of course, you can go through the clues and see that G does not necessarily have to be tested or used. The second clue is never invoked. E is never tested, so G does not need to be tested. The correct choice is answer E.

6. Answer: B

This question tells you that K is not used and exactly four coils are used. The word exactly is important for it will be a constraining factor that will help you eliminate possibilities and determine the correct answer. Also, the question asks you to find the answer that is false. You must eliminate the true answers to find your choice. You use your visual to help you.

Images

The term “K is not used” is the negative of the last term in the sixth clue, so you should create the contrapositive of that statement to help you with this question. First you flip sides:

K = U → J = U & I = T

Then you flip signs:

K ≠ U → J ≠ U & I ≠ T

Finally, you flip connectors:

K ≠ U → J ≠ U / I ≠ T

You add this new contrapositive to your list of clues alongside your visual.

Images

Thanks to your new clue you know that because K is not used, either J is not used or I is not tested. You evaluate your answers based on the given situation. Answer A says I is used. If I is used, it must have been tested. According to your new contrapositive clue either I is not tested or J is not used. Therefore, if I is tested, then it must be the case that J is not used. That leaves D, E, G, and H to fill the remaining three slots (I takes the first). There is nothing in your clues that keeps you from using any of those coils to fill the remaining three slots. Answer A is not false and cannot be your selection. Answer B says J is used. According to your new contrapositive clue, if J is used, then it must be the case that I is not tested. This is the negative of a term in the first clue, and creating another contrapositive might be helpful. We will not go through all the steps here, but this is the result:

I ≠ T → G ≠ T

This contrapositive leads to another contrapositive based on the second clue:

G ≠ T → E ≠ T

These tell you that if I is not tested, then G cannot be tested, and if G cannot be tested, then E cannot be tested. This leads also to the conclusion that if I, G, and E cannot be tested, they also cannot be used. That leaves only D and H, only two coils, to fill the final three slots, and that is not enough. Answer B has to be false and therefore is your answer to this question. The last three answers are about certain coils being tested. With regard to testing, the only coil affected by K not being used is I, which is not in answer C, D, or E. Also, just because a coil is tested does not mean that coil is used and your constraint exists only in the used column. Thus, none of these answers must be false. The correct choice is answer B.

Questions 7–13

As with all logic games you follow the six-step process.

STEP 1:  Identify the game type.

This is a grouping game. You know it is a grouping game because it is asking you to sort the seven children into the three activities. It also says that a child can participate in only one of the activities and the activities occur only once.

STEP 2:  Begin your diagram.

Create a grid with three areas for each group—ceramics, drawing, and origami. Then shorten the names of the children to their first initials (B, E, J, K, P, S, and U) and put them in the upper right corner. The groups can also be shorthanded to C, D, and O.

Images

STEP 3:  Symbolize the clues.

The game description does not give you any information to work on, except that each child must participate in one of the activities and cannot participate in more than one at a time. Also, it tells you that the activity occurs only once. You go clue by clue.

Clue 1: Exactly twice as many children choose drawing as choose ceramics.

You can use the abbreviated letter for each activity to represent the number of children in each group. Therefore, this clue can be represented as follows:

D = 2C

Clue 2: Sharon and Usef participate in the same activity as each other.

You use the children’s initials as symbols for the children. The words “participate in the same activity” is in effect saying that if Sharon, then Usef and if Usef, then Sharon, so you symbolize this clue with two representations as follows:

S → U

U → S

Clue 3: Ezra and Karly do not participate in the same activity as each other.

The phrase “do not participate” is the same as “if Ezra then not Karly” and vice versa.

E → ~K

K → ~E

Clue 4: Barry and Pakhi do not participate in the same activity as each other.

This is similar to the third clue and looks like this:

B → ~P

P → ~B

Clue 5: Barry participates in either ceramics or origami.

This clue can be represented directly in the diagram with B appearing above ceramics and origami with a “/” after or before it to indicate that it can be in either column.

Clue 6: Jaime participates in drawing.

You can represent this clue directly in the diagram and do not need to symbolize it.

STEP 4:  Double-check your symbolizations.

To double-check your symbolizations, translate your symbolized clues back into normal English and see whether they match the original language of each clue.

Here is your visual again, now with your symbolized clues alongside it:

Images

STEP 5:  Make deductions.

Finally, before you tackle the questions, see if you can make any deductions based on the setup of the game and the clues. Go through each type of deduction.

1. Can’t-be-first-or-last deductions

This is not an ordering game, so you will not find this kind of deduction in this game.

2. Repeated-element deductions

There are two clues affecting Barry. The only thing you can deduce from these rules is that if Barry is in ceramics, then Pakhi will be in origami or drawing, and if Barry is in origami, Pakhi will be in drawing or ceramics. There are no other repeated elements that help you come to any deductions.

3. Down-to-two deductions

Since there is no constraint put on the size of each group, this sort of deduction does not come into play. The first rule, D = 2C, may cause some sort of constraint on the size of those two groups, but that constraint cannot be calculated at this time.

4. Block-splitting deductions

Clues 2, 3, and 4 create situations where blocks might cause constraints, but since you do not know the size of the groups yet, you cannot deduce anything yet. These constraints will become apparent as the questions add information to the game.

In this case, your attempt at finding deductions has not resulted in any additional clues, but you might come to further deductions as you work through your questions. You move on to answering the questions.

STEP 6:  Answer the questions in the smartest order.

Approach the questions in this order:

1. Answer the Complete and Accurate List question.

2. Answer questions that give you more information to work with.

3. Answer the remaining questions

Questions 8 and 12 are Complete and Accurate List questions and should be done first. The questions that offer more information are the following:

Question 10 (“exactly one of the children chooses origami”)

Question 11 (“Sharon chooses the same activity as Barry … ”)

The remaining questions are 7, 9, and 13 and can be answered in that order.

THE ANSWERS

7. Answer: B

This question asks you to judge which answer could be a correct matching of children to activities. The answers are about particular children and do not ask you for a complete assignment of children to activities. So, you should go through the answers and use your grid to test each one.

(A) Karly-drawing; Pakhi-drawing; Usef-origami

See the diagram that follows. Place each child in the appropriate group. You are left with B and E to assign to an activity. You know that E cannot be in the same group as K, and you know that B cannot be in the same group as P. Thus, you are left with an uneven number of children in the drawing group. This arrangement cannot be possible because of your first clue, D = 2C, which could be reformulated as C = 1/2D. The drawing group must be divisible by 2. Thus, this answer cannot be correct and you can later add your deduction to your list of clues.

Images

(B) Karly-origami; Pakhi-origami; Sharon-origami

This scenario leaves B, E, and U to be assigned. Because B cannot be assigned with P to origami, then B must be in ceramics. U is assigned to origami because of clue 2. Therefore, E must be assigned to drawing. This scenario is in accordance with your clues and the game, so it is a possible matching of children to activities. This is your answer. You can stop here, but let’s explore the other answers, for learning purposes.

Images

(C) Ezra-drawing; Pakhi-drawing; Sharon-ceramics

This scenario leaves B and K to be assigned. Neither B nor K can be assigned to drawing and you need one more in drawing for clue 1 to work, so this cannot be your answer.

Images

(D) Barry-origami; Ezra-ceramics; Sharon-ceramics

This scenario leaves K and P to be assigned. Your drawing group must include at least six children to satisfy the first clue because you already have three children in the ceramics group. Since you only have two children to assign to the drawing group, you cannot reach that number and this cannot be your answer.

Images

(E) Barry-drawing; Ezra-origami; Karly-drawing

You do not need to use your diagram for this one, since Barry cannot be assigned to the drawing group. According to clue 5 he must be assigned to the ceramics or origami group. This cannot be your answer.

The correct choice is answer B.

8. Answer: A

This question is asking for a complete and accurate list of the children who do not choose drawing. Again, look at each scenario. This means that all the other children choose drawing, so you must find the answer that allows everyone but those listed to take drawing together and still follow your clues.

(A) Barry, Ezra, Pakhi

To test this scenario, use your diagram. You only need one scenario to work, so for argument’s sake, you say that B chooses ceramics. P cannot be in the same group but also cannot choose drawing, so she chooses origami. E can go in either ceramics or origami, so you choose one, in this case ceramics. You have S and U remaining and they can go into any group, but they have to go together because according to your clues they choose the same group. Since you need to make your first rule work, you put them in drawing. This scenario works and this is your answer. Let’s explore the other options for learning purposes.

Images

(B) Barry, Sharon, Usef

This answer does not work because E and K cannot be in the drawing group together.

Images

(C) Barry, Karly, Usef

You do not even need your diagram for this one. Usef and Sharon must be in the same group. Sharon cannot be in drawing and she is not included in this answer, so this answer breaks the rule in the second clue.

(D) Barry, Karly

You do not need your diagram for this one either. Because only two choose not to draw, five children are left to the drawing group. This means the ratio of drawing to ceramics is 5 to 2 (assuming B and K both choose ceramics), which is greater than 2 to 1 and breaks with your first clue.

(E) Barry, Pakhi

This one has the same problem as answer B. E and K are forced to take drawing together and that breaks the rule in clue 3. Even if you were to ignore that clue (which you can’t), it also breaks the ratio in clue 1. This cannot be your answer.

The correct choice is answer A.

9. Answer: E

The question asks you to find the one true statement among the answers. You can evaluate these without the use of a diagram. Answer A cannot be true because it leaves five children to choose between drawing and ceramics. The ratio between these groups is 2 to 1, and there is no configuration of five children that will give you an exact ratio of 2 to 1. Answer B cannot be true because you know that S and U must be in the same group. Usef cannot be the only child in any group. Answers C and D cannot be true for the same reason as answer A: the number of children in drawing must be an even number and able to give you a 2-to-1 ratio to those in ceramics. By process of elimination, answer E becomes your choice, but you can test it to be sure. See the diagram that follows. All the other children can be arranged to match your clues and leave P in ceramics alone. The correct choice is answer E.

Images

10. Answer: B

This question tells you that exactly one child chooses origami and asks you which among the answers must be true. You have already created a scenario similar to this in an earlier question, and in a test situation you will want to use that knowledge to quickly answer any later questions that create the same scenarios. Of course, you know that with only one child in origami, that leaves six children for the two remaining groups. To make clue 1 work there must be two children in ceramics and four in drawing, which gives you the 2-to-1 ratio you need. You must look at what groups of children, going left to right, of 2-4-1 will work. The next diagram is the same as the one you created in question 8. You can use this example as a guide. Look at the possible answers and see which one must be true. Answer A says K chooses ceramics. You see in the diagram that K could take drawing, but you could also make K the child who is alone in origami and move P to drawing without upsetting the game. Answer B says that S chooses drawing. S and U must choose the same group, so S cannot be the lone origami child. Also, you cannot switch S and U with B and E because E and K cannot be in the same group. You can switch B, P, E, and K around in all kinds of configurations, but it remains true that S must choose drawing. This is your correct answer, but let’s review the remaining answers for learning purposes. Answer C says that B chooses ceramics, but this cannot be your answer because B could easily switch with P and be your lone origami participant. Answer D says P chooses drawing, but P could easily switch with B and take ceramics, so this cannot be your answer. Answer E says that Ezra chooses ceramics. You see in the diagram that is possible, but it is also possible to switch E with K in drawing, so this is not necessarily true.

Images

The correct choice is answer B.

11. Answer: A

This question tells you that S chooses the same activity as B. This fact leads you to some new deductions that are pertinent to this question and only this question. Your clues tell you that S and U choose the same activity. Therefore, you know that B, S, and U all choose the same activity. You also know that B, S, and U do not choose the same activity as P. In addition, according to clue 5, S and U participate in either ceramics or origami. Lastly, all three cannot choose ceramics because clue 1 would require that six children choose drawing, but there are only four other children, so all three must choose origami. This new deduction actually negates the previous deduction. So you now have these added clues:

Images

These rules apply only to this one question; keep your new clues separated from your previous clues so that you do not accidentally use them for future questions. The question asks you which among the answers could be true (not must). Using your new information you look to your diagram to evaluate the options. You see that there are only three children left to allocate—E, K, and P. Clue 1 says the drawing group must be twice the size of the ceramics group K or P, so the most you can add to the drawing group is one. In fact, you can only allocate one to each group. Answer A says Ezra chooses drawing, which could be true. Ezra could choose ceramics or drawing, so this is your likely answer. You know that P cannot share the same group with B, so P cannot be in origami. This eliminates answer B. Answers C, D, and E cannot be true because your deductions made all three impossible. The correct choice is answer A.

Images

12. Answer: E

The question asks you to find among the answers the one that does not work as a complete and accurate list of children in the ceramics group. Each answer has two names, so according to clue 1 you need four children in the drawing group. You already have J in the drawing group, so you need only to ask yourself whether there are three children who can be together in the drawing group with J and remain consistent with your clues. The one that fails this test is your answer. You evaluate each answer choice.

(A) Barry, Karly

This leaves E, P, S, and U. You can group E, S, U and P, S, U together in drawing. Therefore, this cannot be your answer.

(B) Karly, Pakhi

This leaves B, E, S, and U. You can group B, S, U and E, S, U together in drawing. Therefore, this cannot be your answer.

(C) Ezra, Pakhi

This leaves B, K, S, and U. You can group B, S, U and K, S, U together in drawing. Therefore, this cannot be your answer.

(D) Barry, Ezra

This leaves K, P, S, and U. You can group K, S, U and P, S, U together in drawing. Therefore, this cannot be your answer.

(E) Sharon, Usef

This leaves B, E, K, and P. There is no group of three that works, mainly because B → ~P and E → ~K. There are too many conflicts. Besides, you are out of options. This must be your answer.
The correct choice is answer E.

13. Answer: D

Your last question in this game asks which among the answers must be true. You do not need your diagram to solve this question. You just need to test each statement against your clues, and you can also use your experience with the previous questions. Of course, you could use your diagram to devise situations that test each one.

(A) E chooses a different activity than S does.

There is nothing among the rules that indicates E and S have to choose a different activity from each other. For example, the scenario below shows them sharing origami, so this cannot be your answer.

Images

(B) E, J, and S do not choose the same activity.

You can create the following scenario to disprove this one. Since E, J, and S can choose the same activity, this cannot be your answer.

Images

(C) E chooses a different activity than J.

The above scenario shows that E and J can choose the same activity, so this cannot be your answer.

(D) B, J, and K do not all choose the same activity.

This means that B and K must be with J in drawing because of clue 6. This is impossible, because it conflicts with clue 5, which requires that B choose ceramics or origami, so it must be true that B, J, and K cannot all choose the same activity. This is your answer, but let’s review the last answer for learning purposes.

(E) B chooses a different activity than E does.

You can create the following scenario to disprove this one. Since B and E can share the same activity, this option does not have to be true.

Images

The correct choice is answer D.

Questions 14–20

As with all logic games you follow the six-step process.

STEP 1:  Identify the game type.

Several clues tell you that this is a grouping game. The most obvious is that the game tells you that there are two test “groups.” Also, it tells you that no rat can be in more than one group, and it tells you a condition under which the rats should be divided (“at least one brown rat and one white rat”) into the two groups.

STEP 2:  Begin your diagram.

Create a diagram that represents the two test groups. In this case, a grid with two columns suits your purposes perfectly. Leave space to the right of your diagram for the symbolization of your clues. Also, abbreviate your rats’ names and group them according to color (B for brown and W for white).

Images

STEP 3:  Symbolize the clues.

Before you go through the clues, you must recognize what is said in the setup for the game, that each group must have at least one brown rat and one white rat. This cannot be easily symbolized, but you can represent it in your diagram by creating slots for the required rats in each group. Now, you go clue by clue and simplify them into easy-to-read equations.

Clue 1: Neither group includes both Abby and Iris.

This clue is worded in a way to confuse you, but it is basically saying that A and I cannot be together in the same group. You can symbolize it as follows:

A → ~I

I → ~A

Clue 2: Neither group includes both Elle and Horn.

This is worded the same way as clue 1, so you can symbolize it in the same way:

E → ~H

H → ~E

Clue 3: If a group includes Dennis, it includes neither Horn nor Iris.

Again, the wording is meant to make it more difficult to understand what the clue is really saying. In this case, it says that if a group includes D, then H and I cannot be in that group. This can be symbolized as follows:

D → ~H & ~I

Due to the complexity of this one it might be useful to create the contrapositive of this clue as well. That is symbolized as follows:

H/I → ~D

Clue 4: If group 1 includes Carl, group 2 includes Horn.

This is a simple “if … then” statement. If Carl is in group 1, then Horn is in group 2. It can be symbolized as follows:

C = 1 → H = 2

STEP 4:  Double-check your symbolizations.

To double-check your symbolizations, translate your symbolized clues back into normal English and see whether they match the original language of each clue. When you are through, your page should look like this:

Images

STEP 5:  Make deductions.

Before you tackle the questions, see if you can make any deductions based on the setup of the game and the clues. Go through each type of deduction.

1. Can’t-be-first-or-last deductions

Because this is not an ordering game this deduction does not apply.

2. Repeated-element deductions

You see that H is repeated in several clues. Of particular interest is the repetition between clue 4 and the contrapositive of clue 3. You can deduce that if C is in group 1, then because H is in group 2, D cannot be in group 2. This could be symbolized as follows:

C = 1 → D ≠ 2

3. Down-to-two deductions

There is a minimum of two rats to each group (one brown, one white). You notice that D is a brown rat. Because of clue 3, if D is in a group, then the only white rats it can be grouped with are E and F. You also notice that F is the only rat that is a free agent. It has no limitations on where it can be grouped and who it can be grouped with.

4. Block-splitting deductions

There are no deductions of this kind to be made.

STEP 6:  Answer the questions in the smartest order.

Answer the questions in this order:

1. Answer the Complete and Accurate List question.

2. Answer questions that give you more information to work with.

3. Answer the remaining questions

Though it is not worded as such, question 14 is a Complete and Accurate List question and should be answered first. Questions that give more information include the following:

Question 15 (“Carl is in group 1 … ”)

Question 16 (“Elle is in group 1 … ”)

Question 17 (“Abby is in the same group as Dennis … ”)

Question 20 (“Dennis is in group 2 … ”)

That leaves questions 18 and 19, which can be answered in that order.

THE ANSWERS

14. Answer: A

This question is asking for which answer gives a possible grouping of the rats. The key word in the question is could, which means that four out of the five answers are not possible. You must go through each answer and test it against your diagram and the clues listed to the right.

(A) Group 1: Abby, Fern, Horn
Group 2: Carl, Elle, Iris

As you can see by the diagram, no clues are violated and each group has one brown rat and one white rat. This is your answer and you could move on to the next question, but let’s evaluate the other answers for learning purposes.

Images

(B) Group 1: Carl, Fern, Horn
Group 2: Abby, Dennis, Elle

You can see the problem here without looking at your diagram. According to the fourth clue, if C is in group 1, then H must be in group 2. Since H is in group 1, this cannot be your answer.

(C) Group 1: Carl, Dennis, Elle
Group 2: Fern, Horn, Iris

This cannot be your answer because group 2 does not include a brown rat and each group must have at least one brown rat and one white rat.

(D) Group 1: Abby, Elle, Iris
Group 2: Carl, Fern, Horn

Abby and Iris cannot be in the same group according to the first clue, so this cannot be your answer.

(E) Group 1: Abby, Dennis, Fern
Group 2: Carl, Elle, Horn

Elle and Horn cannot be in group 2 together according to the second clue. This cannot be your answer.

The correct choice is answer A.

15. Answer: B

This question gives you a fact, that C is in group 1. Based on that fact, you must figure out which of the pairs listed in the answers must be in group 2 together. One deduction you can make before looking at the answers is that since C is in group 1, then H must be in group 2 (clue 4). From your contrapositive of clue 3, you know that D cannot be in the same group as H, so it cannot be in group 2. This leaves rat A as the only available brown rat for group 2. You look at your answers and see that B says A and H must be in group 2 together. This is your answer. There is no need to review the remaining options. The correct choice is answer B.

Images

16. Answer: E

This question tells you that E is in group 1 and asks you to find the pair that could be in group 1 along with E. This means that four out of the five pairs cannot be in Elle’s group. You can see immediately that E is a white rat and that one of the two rats must be brown or the conditions of the game will not be met. You look through your answers and evaluate them one by one.

Images

(A) Fern and Iris

Both are white rats, so this cannot be your answer.

(B) Carl and Fern

If Carl is in group 1, then H is in group 2. Because H is in group 2, rat D cannot also be in group 2, so the brown rat for group 2 will be A. This leaves H and I to fill out the rest of group 2, but clue 1 says that A and I cannot be in the same group, so this does not work. This cannot be your answer.

Images

(C) Carl and Horn

You know this cannot be your answer because clue 4 tells you if C is in group 1, H must be in group 2.

(D) Abby and Carl

Since this scenario takes two brown rats, it leaves only D to be the brown rat in group 2. Thanks to your deduction that if C is in group 1 then D cannot be in group 2, you know that this cannot be true and this option cannot be your answer.

(E) Carl and Iris

This scenario leaves A and D available to be the brown rat in group 2, but you know that D cannot be in group 2 since C is in group 1. Therefore, A automatically becomes the brown rat in group 2. You can easily put E and F in group 2 and remain consistent with the game and clues. This is your answer.

Images

The correct choice is answer E.

17. Answer: E

The question tells you that Abby is in the same group as Dennis and then asks you to find the true statement among the options. This means that four out of the five options are impossible given the game setup and clues. For the purposes of this question, you now have the clue A → D and D → A. This also tells you that C will be the brown rat for the group that A and D are not in. This creates the following clues:

C → ~D & ~A and D/A → ~C

You must test each answer to see which could be true.

(A) Both Elle and Fern are in group 2

Elle and Fern are both white rats, so you need one brown rat to round out the group. Since D and A must be together, it must be C. This scenario leaves H or I to fill the last remaining slot in group 1, but D cannot be in the same group with H, and A cannot be in the same group with I. Therefore, this cannot work and this is not your choice.

Images

(B) Horn is in group 1

According to the contrapositive of clue 3, if Horn is in group 1, then D and A must be in group 2. This forces C to be the brown rat in group 1 and this is impossible. According to clue 4, if C is in group 1, then H must be in group 2. This cannot be your answer.

Images

(C) Carl is in group 1

If C is in group 1, then H is in group 2 according to clue 4. D and A must be in group 2 as well, but this conflicts with your contrapositive—D and H cannot be in the same group—so this cannot be your answer.

Images

(D) Both Elle and Fern are in group 1

Because each group must have three rats and this arrangement leaves only one place in group 1, you can determine that D and A must be in group 2. C must fill that spot in group 1 so that there is a brown rat. This scenario leaves H or I to fill the last remaining spot in group 2, but clue 3 says that neither H nor I can be in the same group as D, so this cannot be your answer.

Images

(E) Iris is in group 2

This is your last remaining option and must be the correct answer, but let’s analyze it to make sure. With I in group 2, you know that D and A must be in group 1 and C must be in group 2. This scenario leaves E, F, and H to assign to the remaining three open spots, and there are a number of configurations that work. Below is one of them. This is your answer.

Images

The correct choice is answer E.

18. Answer: D

The question asks you to find among the answers the one pair that could not be in group 2 together. You can use your experience from question 17 and quickly choose D as your answer. Why? You discovered when working on that question that when A and D are both in group 2, it forces C to be in group 1, since it is your one last remaining brown rat. According to clue 4, when C is in group 1, H must be in group 2 and D and H cannot be in group 2 together. There is no reason to check any other answers. The correct choice is answer D.

19. Answer: C

The question asks that you find which of the answers could be true, so four out of the five answers are impossible. Answering this question is a matter of eliminating answers that are impossible to find the one that is possible.

(A) Carl is in group 1 and Elle is in group 2

This cannot be true because clue 4 tells you that if C is in group 1, then H is in group 2. Clue 2 tells you that E and H cannot be in group 2 together, so you can eliminate this choice.

(B) Abby is not in any group and Carl is in group 1.

If A is taken out altogether and C is in group 1, then D must be the brown rat for group 2. Clue 4 tells you that if C is in group 1, then H is in group 2. Clue 3 tells you that D and H cannot be in group 2 together, so this cannot be your answer.

(C) Abby is in group 1 and Elle is in group 2.

You can see in the sample diagram that you can create every possible scenario under this option’s conditions. Therefore, this is your answer. Let’s review the rest of the options for learning purposes.

Images

(D) Abby is in group 2 and Dennis is in group 2.

This option has the same problem that question 18 posed. If both A and D are in group 2, then C must be the brown rat for group 1. If C is in group 1, rat H must be in group 2. Rats D and H cannot be in group 2 together, so this cannot be your answer.

(E) Abby is in group 2 and Carl is not in any group.

This option forces an untenable situation. With A in group 2 and thus D in group 1, you must divide the remaining white rats among the two groups. Unfortunately, clue 1 tells you that A and I cannot be in the same group together and clue 3 tells you that D and I cannot be in the same group together, so you are stuck with nowhere to put I. This cannot be your answer choice.

The correct choice is answer C.

20. Answer: D

The question tells you that D is in group 2 and asks you to find among the answers the one rat that, if assigned to a group, must also be in group 2. You already know from your previous work that if Carl is in group 1 and D is in group 2, the scenario cannot work. Carl in group 1 forces H to be in group 2, and D and H cannot be in the same group. So, answer D must be correct. All the other options can be easily assigned to group 1 and do not necessarily have to be in group 2, and answers A and C are impossible because D cannot be in the same group as H or I. The correct choice is answer D.

Questions 21–26

As with all logic games you follow the six-step process.

STEP 1:  Identify the game type.

Paying close attention to the language of the game, you can see that this is a two-tiered ordering game. It includes both grouping and ordering language. For example, it requires that you group six of the eight cargoes into three trucks. This is definitely a grouping requirement. Then, it says that the cargo must be loaded in a particular order—front first, then back—and this is definitely ordering language. Each truck has two elements—front and back—which is another hallmark of the two-tiered ordering game. Lastly, the two slots in each group are specified. The front and back of each truck are definitive and not interchangeable. Therefore, you know you are facing a two-tiered ordering game.

STEP 2:  Begin your diagram.

You can imagine a loading dock at the bottom of the diagram with the trucks lined up from left to right. (This may seem confusing, but remember that dock loaders must go through the back to load the front, thus the loading dock is at the bottom.)

Images

STEP 3:  Symbolize the clues.

In this game there are only three clues. Go through them one at a time.

Clue 1: L can only go in Truck 2.

This can be represented in the diagram. The clue does not tell you whether L has to be in the front hold or back hold, so you put the letter L above the column for Truck 2.

Clue 2: T and W cannot go in the back hold of a truck.

This is really two different clues. You can represent both using the box method of representation:

Images

Clue 3: If S is loaded on a truck, then N and O go on the next truck, unless S is loaded on Truck 3.

This clue is a complex conditional statement. In essence it says that if S is on Truck 1, then N and O are loaded on Truck 2. Also, if S is loaded on Truck 2, then N and O are loaded on Truck 3. But, if S is loaded on Truck 3, N and O can be loaded on any truck. You can set this up as two separate conditionals to represent the first part of the statement:

S1 → N2 & O2

S2 → N3 & O3

The contrapositives are as follows:

~N2/~O2 → ~S1

~N3/~O3 → ~S2

The last part of the statement is implied; if S is on Truck 3, no restriction has been placed on N and O.

STEP 4:  Double-check your symbolizations.

To double-check your symbolizations, translate your symbolized clues back into normal English and see whether they match the original language of each clue. When you are through, your page should look like this:

Images

STEP 5:  Make deductions.

1. Can’t-be-first-or-last deductions

At first you might think that your third clue gives you the opportunity to make this deduction. After all, if N and O must be loaded on the next truck after S, then S cannot be last, but this is not the case. The “unless” clause on that clue takes this opportunity away. S can be loaded on Truck 3 because the clue says that if it is, N and O can be loaded anywhere. Only if S is loaded on Truck 1 or 2 do N and O have to be loaded on the next truck. Therefore, you cannot make this sort of deduction for this game.

2. Repeated-element deductions

You do not have an opportunity to make this sort of deduction since there are no repeated elements among the clues.

3. Down-to-two deductions

You cannot definitively place any cargo in a particular hold. You need more information to make this sort of deduction.

4. Block-splitting deductions

If S is located in either Truck 1 or Truck 2, then the next truck will become fully loaded with N and O. This creates a block that cannot be split and will most likely restrict the loading of cargo. Also, since T and W both cannot be loaded in the back hold of a truck, they obviously cannot be loaded on the same truck. You can add these clues to your list:

T → ~W

W → ~T

Finally, before you tackle the questions, see if you can make any deductions based on the setup of the game and the clues. Unfortunately, there are not many clues in this particular two-tiered ordering game and none of the clues, when taken together, lead to any further deductions.

STEP 6:  Answer the questions in the smartest order.

Answer the questions in this order:

1. Answer the Complete and Accurate List question.

2. Answer questions that give you more information to work with.

3. Answer the remaining questions.

Question 21 is the Complete and Accurate List question for this game and should be answered first. Questions with more information include the following:

Question 22 (“P and R are not loaded … ”)

Question 24 (“L, S, and W are loaded in different trucks … ”)

Question 25 (“N, P, and R … loaded in the fronts … ”)

Question 26 (“P is on the back of Truck 2, and N is on the back of Truck 3 … ”)

That leaves question 23 to be answered last.

THE ANSWERS

21. Answer: D

The question is asking you to determine which answer gives a valid loading register. You do not need your diagram to help you answer this one. You can use your clues to test each answer. Thankfully, you only need the first clue to find the correct answer on this question, as the only answer with L in the second truck is D. You do not have to look further and there is no reason to check the answer against the other clues. The correct choice is answer D.

22. Answer: A

The question tells you that P and R are not loaded, then asks for a possible listing of those cargoes that are loaded into the front holds of the three trucks. The facts given tell us that L, N, O, S, T, and W are left as possible cargoes, and you must have six filled holds. Thus, all these items must be loaded. Clue 1 tells you that T and W must be loaded in the front, so the only answer that will work will include T and W. Luckily, answer A is the only option that includes both T and W. The correct choice is answer A.

23. Answer: C

This question asks that you determine which pair of cargoes, if loaded together, must go on Truck 3. Notice that the question does not say the pairs are in any particular order, front and back, so order is not the issue, only pairing. This question also does not require your diagram. You know from your third clue that if S is paired with either N or O, then they must be on Truck 3. Answer C pairs S and N, and they can only be on Truck 3. Thus, the correct choice must be answer C.

24. Answer: C

The question tells you that L, S, and W are each loaded on a different truck, then asks you to choose which among the answers could be true. This means that four out of the five answers have to be false. Use your diagram to check each scenario. You know that L is on Truck 2, so S and W must be on Trucks 1 and 2, but you don’t know which is on which truck. You must test each answer and see what happens.

(A) T is on Truck 1

Clue 2 tells you that T must be loaded in the front of Truck 1. Therefore, W cannot be on Truck 1. W cannot be on Truck 2 because according to clue 1, L must be on that truck, so W must be on Truck 3. That leaves Truck 1 for S. According to clue 3, N and O must be loaded on Truck 2, but there is not enough room and you cannot split that block. Therefore, this cannot be your answer.

Images

(B) S is on Truck 2.

The question says that L, S, and W are loaded on different trucks, but clue 1 tells you that L must be loaded on Truck 2, so if S is on Truck 2 the conditions given by the question cannot be met and this cannot be your answer.

(C) N is on Truck 3.

This situation does not trigger any conditions set out by your clues, and as you can see in the diagram, you can create a viable loading scenario, so this is your answer. But let’s go through the remainder of the answers for learning purposes.

Images

(D) W is on Truck 3.

Again, L must be on Truck 2 and S must be on Truck 1, which leaves no room on Truck 2 for N and O. Since you cannot satisfy clue 3 this cannot be your answer.

Images

(E) S is on Truck 1.

As with the previous answers you know that L must be on Truck 2. If S is on Truck 1, then N and O must be on Truck 2, but L makes it impossible for N and O to be loaded together. This cannot be your answer.

The correct choice is answer C.

25. Answer: E

This question tells you that N, P, and R are loaded into the fronts of the three trucks, but the order is indeterminate. You must find the answer that must be true no matter what order they are in. You know immediately that T and W cannot be loaded since there are no more front holds available. Therefore, only L, O, and S are left to load in the three remaining holds. Clue 1 tells you that L must be in the back hold of Truck 2. You look at your diagram to figure out which answer must be true under the conditions. To test the answers, you put N, P, and R in no particular order to see what might happen. You quickly see that S cannot be in Truck 1 because there is no room for N and O together in Truck 2. There is no room for S in Truck 2 either, so it must be in Truck 3. Thus, O must be on Truck 1. You can quickly see answer E must be the correct choice. You can rearrange N, P, and R, but O must be on Truck 1 regardless of the order, because Truck 2 and Truck 3 become filled by L and S respectively. The correct choice is answer E.

Images

26. Answer: B

You are given that P is in the back hold of Truck 2 and N is in the back hold of Truck 3. You are to choose the answer that includes a possible list of cargoes in the fronts of Trucks 1, 2, and 3. This means that four out of the five options are impossible given the conditions of the question. The list of cargoes in each answer is in order (1, 2, 3). You look at your diagram and test each scenario.

(A) W, O, and R

This scenario leaves L, S, and T to load and only one hold, the back of Truck 1. None of these cargoes can go into that hold. Clue 1 says L must be on Truck 2. Loading S requires that N and O be loaded together on Truck 2 and that is not possible. Finally, clue 2 tells you that T cannot be loaded in the back of any truck.

Images

(B) W, L, and O

This scenario works. All the cargoes in the front holds are consistent with your clues and you are left with R, which can be loaded in the back of Truck 1 without conflicting with your clues. This is your answer, but let’s go through the remaining answers for learning purposes.

Images

(C) R, L, and O

This option leaves S, T, and W as possible cargoes to be loaded in the one remaining hold on the back of Truck 1. All three fail your tests. S requires N and O to be loaded on Truck 2, but there is no room for them. According to clue 2, neither T nor W can be loaded in the back hold of any truck. Note that just because N and O are together on Truck 3 does not necessarily mean that S must be on Truck 2. The “if … then” conditional goes from left to right but not the reverse. Either way, this cannot be your answer.

(D) O, R, and S

This scenario leaves L, T, and W as possible cargoes to be loaded in the one remaining hold on the back of Truck 1. All three fail your tests. Clue 1 tells you that L must be on Truck 2. Clue 2 tells you that both T and W cannot be loaded on the back hold of any truck, so this cannot be your answer.

(E) S, O, and R

This is merely a reordering of answer D and the same conditions apply. This is not your answer.

The correct choice is answer B.

SECTION IV

PASSAGE 1

1. Answer: D

STEP 1:  Read the question and identify your task.

This is a variation of a Main Idea question—a Primary Purpose question. You are required to select from among the choices the answer that best describes what the passage is attempting to achieve.

STEP 2:  Go back to the passage to find the answer.

Since this is a Main Idea question type, refer to your summary of the passage. Then describe that content in slightly more general terms, and find the answer choice that matches this description.

STEP 3:  Read every word of every answer choice.

Answer A concentrates too much on the Supreme Court’s role. While the passage discusses a Supreme Court decision, Wesberry v. Sanders, the passage focuses much more on the propositions approved by the state referendums. Answer B talks about mistakes, but the passage does not make any judgments as to whether the states have made a mistake or not. LSAT passages almost always avoid making such extreme judgments on any person or subject. Answer C speaks of “unique” problems, but the passage makes it clear that these states are addressing problems that exist across the nation and are not unique at all. Answer D says that the passage is about the legislation approved by voters to solve a problem, which is exactly what the passage does in the first two paragraphs. The passage uses those paragraphs to discuss the propositions to solve the problem of political gerrymandering. The answer then says the passage discusses arguments brought by the opposition that might be used in future such propositions raised in other states, which is exactly what the second and third paragraphs discuss. This must be your answer, but you must review the remaining choices to be sure. Answer E has the same problem as answer B. It attributes a very biased intent to the passage, saying that it judges the reform efforts to be “costly and unwise.” The correct choice is answer D.

2. Answer: C

STEP 1:  Read the question and identify your task.

This is an Inference question. The question is asking about a specific part of the passage and what is implied by the language.

STEP 2:  Go back to the passage to find the answer.

In this case, the key word or term is the year “1964.” You must look back at the passage and reread the sentences that discuss that year. You discover that 1964 was the year of the Supreme Court decision in Wesberry v. Sanders, which said that all congressional districts must contain an equal number of persons and established the “one person, one vote” requirement in designating congressional districts. Your question asks what might have been the situation before this case was decided by the Supreme Court. Most likely it will indicate an imbalance in voting among districts.

STEP 3:  Read every word of every answer choice.

Answer A says it is implied that political parties had no influence before 1964, but the opposite is actually implied by the passage. Political parties had even more control before 1964, which is why the decision was necessary in the first place. Answer B says that some citizens’ votes counted for less than a full vote. The passage says nothing about the value of a vote being less than full. Instead, it discusses districting and its effect on elections. Answer C seems promising since it talks about the design of a district and how it could create an imbalance in favor of a particular group. This could be your answer, but you should assess the remaining choices. For answer D, there is no indication that the Supreme Court case dealt with an incongruity between states or that one state had an advantage over another in this regard. Answer E has things reversed. It was after 1964 that the Supreme Court gained the ability to overrule redistricting planes, not the other way around. The correct choice is answer C.

3. Answer: D

STEP 1:  Read the question and identify your task.

This is a Line ID question. The question is asking you to put a particular section of the passage in context and find among the answers the best description of how it fits into that context.

STEP 2:  Go back to the passage to find the answer.

In this case, the section concerns one you investigated already in question 2, the Wesberry v. Sanders decision. You can use your knowledge from question 2 to answer this question. Go through each answer and assess its potential.

STEP 3:  Read every word of every answer choice.

Answer A cannot be your correct because “one person, one vote” was created by the decision. It was not a problem that the decision intended to solve. Answer B is tempting because the Supreme Court decision was an early legal action that led to the recent propositions, but the direct connection is tenuous. Still, you will keep this answer in mind as you review the remaining options, and if a better answer does not come up then this may be your selection. Answer C cannot be correct because there is no mention of the Supreme Court decision in relation to any effort by the executive or legislative branches of the United States government. Answer D is fairly accurate. The section of the passage does give some historical perspective. There are several key phrases that make this choice optimal, such as “Since the early years,” “In 1964,” and “continued to influence.” These are historical-sounding phrases. Answer E cannot be correct because the Supreme Court was trying to solve gerrymandering, not create it. While answer B seemed promising, answer D is much better, so the correct answer is D.

4. Answer: C

STEP 1:  Read the question and identify your task.

This is a Line ID question. This question tells you to refer to specific lines (53–59) in the passage and asks you to find among the answers an inference you can derive from those lines in relation to Proposition 20.

STEP 2:  Go back to the passage to find the answer.

You find the lines in the passage and discover that they discuss Proposition 27 and its attempt to eliminate the Citizens Redistricting Commission. With this in mind you look at your choices.

STEP 3:  Read every word of every answer choice.

Answer A gives what might have been the opinion of those supporters of Proposition 27, but it has nothing to do with Proposition 20. Answer B attributes a motive to the supporters of Proposition 27 that is not in evidence in the lines. Answer C first says that the commission existed before the election. Since Proposition 27, if approved, was to eliminate the commission then it must have existed before the election. Also, Proposition 20 was intended to make the commission able to draw districts for the congressional map, so you can definitely infer that they did not have this authority before the election. Since both elements of answer C are implied by the lines, this is most likely your answer, but you should review the rest of the options to be sure. Nothing in the lines implies in any way what the voters might have done in any other scenario, so D cannot be correct. Finally, in answer E, nothing in the lines implies the unconstitutionality of the commission or how the courts would react if Proposition 27 was not approved. Also, this answer has nothing to do with Proposition 20, which is what the question is asking for. The correct choice is answer C.

5. Answer: E

STEP 1:  Read the question and identify your task.

This is a Weaken question. You must find among the answers the statement that most undermines support for Proposition 20.

STEP 2:  Go back to the passage to find the answer.

Refer to the passage and skim over the third paragraph and its arguments in favor of Proposition 20. In this case, the paragraph discusses the fact that without Proposition 20, politicians have secure incumbency due to gerrymandering. The population of their district is narrowly defined, and thus they have to satisfy only that specific constituency to be reelected. The correct answer will state a fact proving that even with such gerrymandering the politicians are still not secure in their incumbency.

STEP 3:  Read every word of every answer choice.

Answer A is tempting. Why would people want to change the system if most voters are happy with their representatives? But Proposition 20 is meant to help the minority and underrepresented, not the majority. While this option might be tempting, it is not the strongest. Regarding answer B, compromise and cross-party-line success in a few cases does not make much of a case against the proposition. Answer C actually seems to make a stronger case for the proposition by saying that it will be income, race, and gender blind. For answer D, this is a technical issue and even if every member of the commission is a member of a political party, this does not mean that it cannot be balanced to avoid one party having more influence over the other. Finally, answer E indicates that population movements may make the proposition impossible to execute. This would definitely undermine the arguments for Proposition 20 because, if answer E is true, even if approved it would not be effective. The correct choice is answer E.

6. Answer: B

STEP 1:  Read the question and identify your task.

This is a Line ID question. The question is asking you to find among the options a synonym for the word entrenched that gives the meaning as it is used in the passage.

STEP 2:  Go back to the passage to find the answer.

Refer to the passage and read the sentence that includes the word entrenched to determine how it is being used. In this case, you notice that entrenched politicians do not have to compromise on policy since they are assured of keeping their job. The correct answer will probably be a word similar to having such a secure position.

STEP 3:  Read every word of every answer choice.

Answer A could be seen as the opposite of what the word means since the politicians are elected over and over to the same office. Answer B is promising since the politicians are able to keep their position despite any failures on their part. They are “deep-seated” in their position. You review the rest of the answers to see if there is a better option. Answer C seems more like an adjective for a person in a pressure situation. If the politicians were more vulnerable but principled in their stances on issues, they might be described as unshakeable, but this is not the case here. Answer D is similar to answer C, but the politicians are not stubborn, nor do they face any opposition. Answer E would mean that the politicians are taking action and are unable to be persuaded to change their minds. This is also an inexact match to your definition. The correct choice is answer B.

PASSAGE 2

7. Answer: D

STEP 1:  Read the question and identify your task.

This is a Main Idea question. However, in the case of paired passages, look for answer choices that articulate the main ideas of both passages, as they relate to each other.

STEP 2:  Go back to the passage to find the answer.

Refer to your passage summary.

STEP 3:  Read every word of every answer choice.

Answer A discusses solutions to problems, which is never addressed in either passage. Although the two theater movements discussed here both arose in response to societal ills, there is no sense from the passages that these were “solutions” to the problem of the Great Depression. For similar reasons, answer choice B can be eliminated. Passages A and B both deal with the same circumstance: the Great Depression. And both discuss theater movements arising from these social and economic circumstances. But passage B is not a solution to a problem in passage A. There is also no contradiction between the passages, so eliminate answer choice C. Answer choice D, however, in its broad strokes, gets the relationship correct. The passages are very similar, with both describing theater groups that came about as a response to the circumstances of the Great Depression. This is very likely the correct answer. Answer choice E cannot be correct. There is no suggestion that the Shock Troupe is a refinement of Group Theatre. If anything, passage B suggests that the opposite is true, due to the Shock Troupe’s later adoption of some of the Group Theatre’s methods. The correct answer must be D.

8. Answer: C

STEP 1:  Read the question and identify your task.

This is a Inference question. The question asks that you assess each answer to see whether it can be inferred from the passage based on what you read.

STEP 2:  Go back to the passage to find the answer.

You refer to the passage and look specifically where it discusses the new theater groups. By understanding the new groups, you can make certain assumptions about what came before, in the early twentieth century. Since the new groups were more serious and more interested in social change, the earlier groups must have been less serious and less interested in bringing about social change.

STEP 3:  Read every word of every answer choice.

Looking back at the passage you can see that Answer A cannot be correct since it says that agit-prop theatre was an evolution from an earlier form of theater. Thus agit-prop was not a wholly new concept. Answer B makes an unwarranted connection between a political movement and Strasburg’s efforts, which were artistically motivated, not politically motivated. Answer C is an excellent candidate for your choice. Since the passage is almost entirely how the Great Depression transformed Broadway into offering more serious fare, it is likely that in 1920 you would more likely find a musical or lighthearted show since this was before the Great Depression. This is most likely your answer, but you should review the remaining options. Answers D and E cannot be inferred from the passage. The correct answer is C.

9. Answer: B

STEP 1:  Read the question and identify your task.

This is a Line ID question. The answer choices will contain four points that are explicitly mentioned as being points of difference in the passage. The correct answer will not be mentioned at all or will be a point of similarity.

STEP 2:  Go back to the passage to find the answer.

In this case, go back to passages A and B, and look for discussion about the differences between these theater groups and others.

STEP 3:  Read every word of every answer choice.

Passages A and B both explicitly mention that fact that the Group Theatre used trained actors while the Shock Troupe did not, so eliminate answer choice A. Answer choice B, however, immediately looks promising. Both groups were concerned with presenting plays that depicted societal ills. This will most likely be your correct answer. Answer choice C is another point of difference between the two. The passages mention that the Group Theatre used conventional theatrical staging, whereas the Shock Troupe sometimes performed in the street. The names of the characters are another point of difference, explicitly mentioned in both passages, so eliminate answer choice D. Finally, answer E may be tempting, but passage B states that the Group Theatre merely wanted to depict societal ills whereas the Shock Troupe wanted to move further and actually advocate societal change. The correct answer is B.

10. Answer: C

STEP 1:  Read the question and identify your task.

This is an Inference question. This question is asking you to use the information about the Shock Troup as the basis for a judgment, mainly which play the theater would least likely perform.

STEP 2:  Go back to the passage to find the answer.

You go back to the part of passage B that describes the theatrical philosophy of the Shock Troup.

STEP 3:  Read every word of every answer choice.

The Shock Troup was a politically oriented theater group, so the answer that is least political will be the correct choice. Answer A, B, D, and E are all very politically or socially strident plays. The play described in answer C seems rooted in the drama of everyday life. There is little social or political content in it. The correct answer is C.

11. Answer: A

STEP 1:  Read the question and identify your task.

This is an Inference question. The correct answer will correctly identify the attitude of the author of passage B toward the Group Theatre as that attitude is presented in the passages.

STEP 2:  Go back to the passage to find the answer.

Passage B only mentioned the Group Theatre in passing, noting at the beginning of the passage that the Group Theater was concerned only with societal ills whereas the Shock Troupe wanted to also advocate change. At the end, the author also mentions that the Shock Troupe wanted to train its actors using the Group Theatre’s method, that it might better connect with audiences.

STEP 3:  Read every word of every answer choice.

Answer A is promising right away. The Group Theatre was brought in to train the Shock Troupe’s actors to allow those actors to better connect with audiences. Answer choice A is most likely correct. Answer choice B discusses which theater group was better known, which is never discussed. Eliminate answer choice B. Answer choice C discusses comparative idealism, which is not discussed here. Additionally, the information in the passage would seem to support the opposite contention, since the Shock Troupe thought theater could change society, not merely present the ills of society. Much like answer choice B, you have no way of knowing which play was more popular. Finally, although you know that the Shock Troupe believed that “The Method” was an effective means of teaching acting, you cannot know that they believed it to be the most effective means of teaching acting. The correct answer must be A.

12. Answer: C

STEP 1:  Read the question and identify your task.

This is a Line ID question. This question takes a technical term and asks you to choose the answer that most closely represents what it means.

STEP 2:  Go back to the passage to find the answer.

You look back at the passage and derive that agit-prop theaters were politically motivated, and that those theaters took extreme measures to put their political ideas in front of the masses. Thus, the answer will be oriented toward this sort of description. All the answers extend the shortened words on each side of the hyphen so you must assess both words.

STEP 3:  Read every word of every answer choice.

For answer A, both words are inadequate. Agit-prop theater was not concerned with agriculture and it did not make propositions through its performances. Answer B does well with the first part of the term. Agit-prop theater definitely used its performances to agitate, especially when they performed at picket lines or in front of workers as they went to work. But, the second term, “proposal,” seems too weak a description of what they did. Answer C is much better. The first term, as discussed, fits well. The second term works as well, because they definitely used their performances to spread propaganda or politically charged ideas. This is most likely your answer, but you must assess the remaining choices as well. The second term in answer D is problematic. Propagation is a term that could be used to describe the dissemination of ideas, but it also has a more common meaning related to breeding. Such ambiguity hurts this option’s chances. Answer E is way off the mark since the word “proper” does not fit agit-prop theater at all. They were not interested in being a proper theater, especially since they performed their plays outside the theater. The correct answer is C.

PASSAGE 3

13. Answer: B

STEP 1:  Read the question and identify your task.

This is a Main Idea question, which in essence is asking you to find among the answers the one statement that sums up the author’s argument or thesis.

STEP 2:  Go back to the passage to find the answer.

Refer to your passage summary.

STEP 3:  Read every word of every answer choice.

Answer A focuses on Bo Rothstein’s essay but ignores the rest of the passage, which puts Rothstein’s claims in context of a wider discussion of the merits of a welfare state. Answer B starts with Rothstein’s essay, then adds the context, the evaluation of happiness as a measure of success and perspective on whether the studies help you with the larger argument afoot. Even though this answer is not perfect, it is your best candidate so far. You must evaluate the remaining options. Answer C gives a good description of the political part of the passage but ignores the essays and studies discussed, so this is most likely not your choice. Answer D is problematic because the passage does not say the politicians, social scientists, and economists are unsupported by data, just that they are not focused on the happiness factor. Most likely they do have their own data, just not Rothstein, Pacek, and Radcliff’s data that takes happiness into account. Answer E starts out well. The passage does note some flaws in the SWB measurement, but the passage makes no claim that an improvement in the data fueling this measurement would make it more acceptable to the “thinkers of today.” You are left with the only answer that works, answer B, flawed though it may be. The correct choice is answer B.

14. Answer: E

STEP 1:  Read the question and identify your task.

This is an Arguments-style question. It asks that you choose the answer that best describes the structure used in the passage’s argument.

STEP 2:  Go back to the passage to find the answer.

To do this, you look back at the passage and read the first sentence or two of each paragraph to get a sense of how the passage moves from one subject to the next. You notice that the first paragraph starts with a very general discussion of the current conflict over the welfare state. The second paragraph moves to a discussion of measuring happiness. The third paragraph goes deeper into the basis of Rothstein’s work on happiness. Finally, the concluding paragraph brings the discussion back to the United States. Now, you assess your choices.

STEP 3:  Read every word of every answer choice.

Answer A is a very good description of the passage, and you might stop here, but the description does not seem to do justice to paragraph three. Paragraph three does not simply “assess” the happiness measurement. Still, you will hold on to this option as a possibility and read through the remaining options to see if there is a better one. In answer B the word “solution” in the second sentence is inappropriate for this passage. The author of the passage offers no solutions since the existence of the welfare state is not a social problem (such as crime) that must be “solved.” Instead, the passage discusses the merits of the universal welfare state and how to evaluate its success or failure. Answer C is incorrect because its organization does not match the passage. The third paragraph actually goes on to provide more information on the basis of Rothstein’s work on happiness. It does not “discredit” that work. Answer D also says that a solution to the problem is proposed and there is no solution offered, so this cannot be your choice. Finally, answer E offers answer A some competition. In this case, the answer seems more descriptive, especially with regard to paragraph three, saying that it discusses the validity of the happiness measurement, something answer A ignores. Between answers A and E, answer E is the better description, so the correct choice is answer E.

15. Answer: C

STEP 1:  Read the question and identify your task.

This is a Line ID question. This question asks that you look at a specific sentence or comment within the passage and explain the author’s purpose for using it.

STEP 2:  Go back to the passage to find the answer.

Look back at the identified line and read the sentence and possibly its context to understand the author’s reasons for including it. Your options will most likely be related to how that comment fits within that context.

STEP 3:  Read every word of every answer choice.

Answer A restates the author’s comment in more explicit terms, but it does not explain why this comment appears where it does in the passage. Answer B may be true, but this relates to the next paragraph about Pacek and Radcliff and not to the paragraph in which the comment appears. Answer C is very promising. The sentence before the bias comment discusses how Rothstein focused his study on northern European countries, which you learn have long-standing welfare states, and you have already learned that he is Swedish, that is, from a northern European country. This explanation fits within your context and explains the comment very well. You should evaluate the remaining options just in case there is a better answer. Answer D may be true, but this explanation refers to a very different bias from the one the author is referring to in this context. Finally, answer E gives a totally unrelated fact and again refers to a different type of bias that does not fit into the context of where the comment appears in the passage. The correct choice is answer C.

16. Answer: B

STEP 1:  Read the question and identify your task.

This is a Weaken question. This question is very similar to a question you might find in the Arguments section of the LSAT. It asks which answer would most undermine or weaken a particular argument Rothstein is advancing.

STEP 2:  Go back to the passage to find the answer.

Rothstein feels that the SWB measurement is a better measurement of society because it is a “widespread” arbiter rather than an elitist one that the experts create.

STEP 3:  Read every word of every answer choice.

Answer A makes a case that the experts, due to their own biases, make judgments that are not much different from the happiness measurement. This weakens Rothstein’s statement somewhat, but it is not a very strong attempt. You should expect that there might be a stronger effort in the other options. Answer B actually makes a case that the experts are superior because they filter out cultural factors that cloud one’s ability to focus on the political problem at hand. This is stronger than answer A because it makes a case that the experts are better, but you should continue to review the remaining options. Answer C restates Rothstein’s argument to a certain extent and does nothing to weaken it. Answer D may be true, but this also supports Rothstein’s argument that his measurement is superior at assessing the success of the welfare state. If the experts are, in fact, unaffected by economic changes, they might remain an elitist arbiter and continue to make poor assessments of political progress. Finally, answer E also supports rather than weakens Rothstein’s argument. If the SWB measurement is truly more accurate at making such predictions than the models created by the experts, then it is also a better arbiter. The correct choice is answer B.

17. Answer: C

STEP 1:  Read the question and identify your task.

This is a Strengthen question. This question is very similar to one you might find in the Arguments section of the LSAT.

STEP 2:  Go back to the passage to find the answer.

You must find a statement that supports the arguments presented in the first paragraph, advanced by those who support universal welfare programs in the United States.

STEP 3:  Read every word of every answer choice.

Answer A actually supports those in the opposing camp who believe that social welfare is harmful to the nation, so this cannot be your answer. Answer B also gives support to the opposition to welfare programs, saying that welfare recipients do not use the funds the federal government gives them. Answer C says that new welfare programs increase political participation. This very much supports the argument advanced in the passage that welfare programs are important to maintaining an active democracy. Therefore, this is most likely the correct answer. You should review the remaining answers to be sure. Answer D may be true, but whether recipients like or dislike the government does not seem relevant to either the supporters or the critics of social welfare programs. Answer E would definitely be helpful to the supporters of welfare programs, but it discusses an argument advanced by Rothstein later in the passage. The question asks that you base the answer just on paragraph one. Since this is not really addressing the specific arguments in the first paragraph, it is not as strong as answer C. The correct choice is answer C.

18. Answer: D

STEP 1:  Read the question and identify your task.

This is a Tone question. For this question you are to use your own judgment at assessing the author’s tone or attitude toward Rothstein’s approach.

STEP 2:  Go back to the passage to find the answer.

Specific words in the passage give you clues to this attitude. For example, the comment regarding Rothstein’s bias in the second paragraph indicates that the author might have some doubts about Rothstein’s conclusions. Consider the phrases “He even goes as far as to say” and “the SWB is the most controversial basis of these studies.” These and other phrases, though tepid, attest to the author’s doubts regarding Rothstein. You assess your options with this in mind.

STEP 3:  Read every word of every answer choice.

These observations regarding the author’s language make it impossible to think “cautious neutrality” can be correct, so answer A cannot be your choice. The author does not use strong enough language for you to consider answer B. Regarding answer C, the author acknowledges that Rothstein’s work might be helpful to the discourse regarding the welfare state, but “moderate advocacy” would mean the author is recommending it and this is not the case. Answer D is very likely your choice for the author does show a fair amount of skepticism with regard to Rothstein’s work. Finally, in answer E, nothing in the language shows a grudging acceptance of Rothstein’s ideas. This would require the author to ultimately admit an error in judgment and words of contrition as Rothstein’s ideas won over. You see none of that in the passage. Thus, the correct choice is answer D.

19. Answer: E

STEP 1:  Read the question and identify your task.

This is an Information Retrieval question. You must find among the answers the one statement Rothstein would disagree with.

STEP 2:  Go back to the passage to find the answer.

You refer to the passage and read the sections that give Rothstein’s views. Paragraphs two, three, and four all discuss his views. The one answer that is not consistent with those views concerning social happiness will be the correct answer.

STEP 3:  Read every word of every answer choice.

In paragraph three you learn that Rothstein admits that cultural factors like the one in answer A could affect happiness. Even though he dismisses such concerns, he might agree that the abilities listed in answer A would cloud the relationship between the SWB and the establishment of a universal welfare state. For answer B, Rothstein would surely agree that a program protecting children would increase national happiness. Answer C relates to Rothstein’s bias, which you have already seen favors the northern European states, so he would most likely agree with this statement. Next look at answer D. As the passage states, Rothstein believes that people need a long-standing trust in their institutions to achieve happiness, so he would definitely agree that such a long-standing program will increase their SWB. Finally, in answer E, Rothstein would never agree that a misappropriation proved the failure of a welfare state in general. He would say only that the particular welfare state had failed one of the key elements necessary to bring happiness to its recipients. The correct choice is answer E.

PASSAGE 4

20. Answer: B

STEP 1:  Read the question and identify your task.

This is a Main Idea question, which asks you to find among the answers the one statement that encapsulates the passage.

STEP 2:  Go back to the passage to find the answer.

Refer to your passage summary.

STEP 3:  Read every word of every answer choice.

Read through the answers paying close attention to the language. Answer A attributes a motive to the corporations not supported by the passage. The passage says nothing about a “stigma” associated with cash donations or that they are seen as cheap marketing tactics. Actually, the passage states that they are no different from each other. Answer B is promising. The passage does argue that whether corporations integrate philanthropic activities into their operations or they just cut a check, the effect to all parties is the same, and the passage does explore the reasons for corporate philanthropy without coming to any particular conclusion. This is most likely your answer, but you need to review the remaining options to be sure. Answer C is too strong an opinion to be summarizing the main idea of this passage, which retains a certain amount of objectivity. The passage does not make such a strong recommendation to do both or either one. For answer D, extortion is one idea explored in the passage, but it is not the entire idea of the passage. This answer makes it too central, and the passage definitely does not make an outright recommendation that corporations just make a charitable donation. Finally, answer E is not bad, but it is fairly general in its discussion. There is no mention of the integration of philanthropy into operations and sales versus just cutting a check, which is central to the passage’s argument. Thus, the correct choice is answer B.

21. Answer: C

STEP 1:  Read the question and identify your task.

This is a Line ID question. You need to look at the lines specified in this question and assess the author’s attitude or tone with regard to the philanthropy model discussed.

STEP 2:  Go back to the passage to find the answer.

In this case, the lines discuss the philanthropic model in which corporations integrate philanthropy into their “supply, production, and sales operations such that they become part and parcel with their corporate and product identity.” Reading the lines you discover that the author feels that this model is “no different than that of simple donations.”

STEP 3:  Read every word of every answer choice.

With this in mind you read through your answers. You learn later in the passage that answer A is not true, but you do not need the rest of the passage to know that nothing in the lines specified indicates such a positive perspective on the philanthropic mode. Answer B also discusses elements later in the passage that are contradictory to what they say. Still, the lines under consideration give no indication that the author views this model as empowering to the customers. Answer C is most likely your answer. The author definitely does not seem to believe that the new model is really anything new at all and also offers no insights into why corporations engage in philanthropy. You should review the remaining options to be sure. Answer D ventures into areas that are explored later in the passage and are out of the scope of this questions. And regarding answer E, the author does not seem very pleased with the new way of being philanthropic since the language indicates that the author does not see a difference between it and cutting a check. Therefore, the correct choice is answer C.

22. Answer: B

STEP 1:  Read the question and identify your task.

This is an Inference question. This question is one of the rare ones that ask you to imagine the passage continuing into another paragraph, and you must select the answer that fills that role best. In essence, it is asking you to infer what will come after the last paragraph.

STEP 2:  Go back to the passage to find the answer.

Return to the passage and scan the last paragraph. The last few sentences give you a clue as to where the author might continue with his or her thoughts. The last paragraph discusses the justifications for corporate philanthropy, and the last few sentences compare a corporation to an individual in society and how that view affects the analysis.

STEP 3:  Read every word of every answer choice.

Answer A might be a good paragraph to be inserted after the second paragraph, which discusses integration of philanthropy into supply lines, but not at the end of the passage, which has moved on to other considerations. Answer B might very well be the next topic, moving on from corporate philanthropy to address whether the dollars they devote to charity do any good and which method works best. This is not a perfect option, but it might work. You will need to read through the other options to see if there is a better one. Answer C starts out well, but the second sentence veers off to make an odd logical leap into government intervention. Nothing in the passage mentions government involvement, so this is most likely not the correct choice. If the passage had ended with paragraph three, answer D might be correct, but the fourth paragraph makes this one impossible to choose, since it has moved on to address the larger question of whether corporations should be engaged in philanthropy at all. Finally, answer E is tempting, but it actually belongs as another point within the last paragraph rather than a new paragraph, which should move on to a new topic altogether but one that follows the previous paragraph logically. Answer B is the correct choice.

23. Answer: E

STEP 1:  Read the question and identify your task.

This is a Line ID question. You are asked to compare two different sections of the passage and choose the answer that most accurately describes the relationship between them.

STEP 2:  Go back to the passage to find the answer.

You look at the specified sections of the passage and discover that the latter passage is almost a restatement of the earlier passage, except that the later passage seems more explicit and thus could be considered a clarification.

STEP 3:  Read every word of every answer choice.

You can quickly eliminate answer A because you can see that both sections discuss the lack of difference between the two options and they both discuss the financial equation involved in the integration of philanthropy into operations and sales. It cannot be true that there is no relationship. There is no other opinion than what the author has revealed in either section, so answer B can be eliminated. There also is no hypothetical situation offered in either section. No particular examples are discussed at all, so answer C cannot be the correct choice. Regarding answer D, there is agreement with the position but no disagreement over particulars. You have eliminated all the other answers, so answer E must be correct, but you must read it to be sure. The sections are equivalent assertions that integration versus just donating are not much different, and the latter section gives clarification to the financial aspect of why they are not different, specifically how both decrease profitability and shareholder value. The correct choice is answer E.

24. Answer: C

STEP 1:  Read the question and identify your task.

This is an Inference question. This question asks that you make an inference based on the author’s views on corporate philanthropy.

STEP 2:  Go back to the passage to find the answer.

This question is asking for an inference based on your impression of the content of the entire passage and the author’s opinion regarding corporate philanthropy. You must have read the entire passage to answer this question, but if you need to you can skim the passage for clues that might help you as you review the answers.

STEP 3:  Read every word of every answer choice.

Regarding answer A, informing the customer that the corporation is getting a tax deduction is not exactly useful to the customer or the corporation. It is unlikely the author would agree with such an idea. For answer B, such a proposition might discourage the corporation from engaging in the philanthropic activity altogether, and it is unlikely the author wants that to happen since there are indications that the author supports corporate philanthropy even if some practices are questionable. Answer C seems like a valid idea that the author might agree with. The author implies that the customer is not informed that the increase in purchase price is a result of philanthropic efforts, so the author might support informing the customer about the effects of integrating philanthropy into operations and sales. This is probably your answer, but you should review the remaining options to be sure. Answer D is a nice idea, but this seems out of the scope of the passage. The author is not concerned about misuse of funds but in philanthropic efforts in general. Answer E is an interesting possibility as well, but the author is not really concerned about morality or ethics within corporations or that customers know about the company’s morals. The correct choice is answer C.

25. Answer: D

STEP 1:  Read the question and identify your task.

This is a variation of a Main Idea question—a Primary Purpose question, which asks you to identify a more general description of what the passage is doing logically.

STEP 2:  Go back to the passage to find the answer.

Read the first sentence of each paragraph and possibly the final sentence to understand any conclusion that is reached by the author. Then, evaluate the options and look for the one that seems most accurate.

STEP 3:  Read every word of every answer choice.

Reading answer A, certain words become problematic. The author does not question the reasons for integrating philanthropy into operations and sales. The author only explores what effect such behavior has and assesses its differences from simply donating money. Also, the question posed at the end is asking why they participate in philanthropy in the first place, so this answer has several clues that make it incorrect. You cannot recommend answer B because the author never makes an argument that corporate philanthropy should stop. In fact, the author supports the activity even if he or she questions certain practical aspects of the behavior. The first part of answer C seems right but the second part does not. The author does not raise any moral reason against corporate philanthropy. The author questions the basis for it but never comes to a definite conclusion or recommendation against it. Regarding answer D, the author does explain a trend in corporate behavior, namely integration of philanthropy into operations and sales, and compares it to the previous behavior of just making a donation. Finally, the author poses a question directed at the idea of corporate philanthropy in general. This answer is right on the mark. Answer E proposes that the author discusses an unethical behavior, but this is not the case. The author merely says that the corporations are not giving full disclosure. The correct choice is answer D.